SlideShare a Scribd company logo
1 of 22
Download to read offline
MATHEMATICS
11
MULTIPLE CHOICE
QUESTIONS WITH
ANSWERS
MUHAMMAD SALMAN
SHERAZI
M.Sc(Math) , B.ed
03337727666/03067856232
2 | P a g e
Contents
UNIT #
Title Page #
1 Number System 3
2 Sets, Functions and Groups 4
3 Matrices and Determinants 6
4 Quadratic Equations 8
5 Partial Fractions 9
6 Sequence and Series 10
7 Permutation, Combinations and
Probability
11
8 Mathematical Induction and
Binomial Theorem
13
9 Fundamental of Trigonometry 13
10 Trigonometric Identities 15
11 Trigonometric Functions and their
Graphs
17
12 Application of Trigonometry 18
13 Inverse Trigonometric Functions 19
14 Solution of Trigonometric Equations 21
3 | P a g e
UNIT # 01 Number Systems
Each question has four possible answer. Tick the correct answer.
1. For any complex number ๐’›, it is always true that |๐’›| is equal to:
(a) |๐‘ง| (b) | โˆ’ ๐‘ง| (c) | โˆ’ ๐‘ง| (d) โœ” all of these
2. The numbers which can be written in the form of
๐’‘
๐’’
, ๐’‘, ๐’’ โˆˆ ๐’ , ๐’’ โ‰  ๐ŸŽ are :
(a) โœ”Rational number (b) Irrational number (c) Complex number (d) Whole number
3. A decimal which has a finite numbers of digits in its decimal part is called______ decimal.
(a) โœ”Terminating (b) Non-Terminating (c) Recurring (d) Non recurring
4. ๐Ÿ“. ๐Ÿ‘๐Ÿ‘๐Ÿ‘ โ€ฆ. Is
(a) Rational (b) Irrational (c) an integer (d) a prime number
5. ๐… is
(a) Rational (b) โœ” Irrational (c) Natural number (d) None
6.
๐Ÿ๐Ÿ
๐Ÿ•
is
(a) โœ”Rational (b) Irrational (c) an integer (d) a whole number
7. Multiplicative inverse of โ€ฒ๐ŸŽโ€ฒ is
(a) 0 (b) any real number (c) โœ” not defined (d) 1
8. If ๐’‚ is any non-zero real number, then multiplicative inverse is
(a) โ€“ ๐‘Ž (b) โœ”
1
๐‘Ž
(c) โˆ’
1
๐‘Ž
(d) not defined
9. For all ๐’‚ โˆˆ ๐‘น , ๐’‚ = ๐’‚ is โ€ฆ. Property.
(a) โœ”Reflexive (b) Symmetric (c) Transitive (d) Trichotomy
10. For all ๐’‚, ๐’ƒ โˆˆ ๐‘น , ๐’‚ = ๐’ƒ โŸน ๐’ƒ = ๐’‚ is calledโ€ฆ.. property.
(a) Reflexive (b) โœ” Symmetric (c) Transitive (d) Trichotomy
11. Golden rule of fraction is that for ๐’Œ โ‰  ๐ŸŽ,
๐’‚
๐’ƒ
=
(a) โœ”
๐‘˜๐‘Ž
๐‘˜๐‘
(b)
๐‘Ž๐‘
๐‘™
(c)
๐‘˜๐‘Ž
๐‘
(d)
๐‘˜๐‘
๐‘
12. The set {๐Ÿ, โˆ’๐Ÿ} possesses closure property ๐’˜. ๐’“. ๐’•
(a) โ€ฒ + โ€ฒ (b) โœ” โ€ฒ ร— โ€ฒ (c) โ€ฒ รท โ€ฒ (d) โ€ฒ โˆ’ โ€ฒ
13. If ๐’‚ < ๐‘ then
(a) ๐‘Ž < ๐‘ (b)
1
๐‘Ž
<
1
๐‘
(c) โœ”
1
๐‘Ž
>
1
๐‘
(d) ๐‘Ž โˆ’ ๐‘ > 0
14. The additive identity in set of complex number is
(a) โœ”(0,0) (b) (0,1) (c) (1,0) (d) (1,1)
15. The multiplicative identity of complex number is
(a) (0,0) (b) (0,1) (c) โœ” (1,0) (d) (1,1)
16. The modulus of ๐’› = ๐’‚ + ๐’Š๐’ƒ is
(a) โˆš๐‘Ž + ๐‘ (b) โœ”โˆš๐‘Ž2 + ๐‘2
(c) ๐‘Ž โˆ’ ๐‘ (d) โˆš๐‘Ž2 โˆ’ ๐‘2
17. ๐’Š๐Ÿ๐Ÿ‘
equals:
(a) โœ”๐‘– (b) โ€“ ๐‘– (c) 1 (d) -1
18. The multiplicative inverse of (๐Ÿ’, โˆ’๐Ÿ•) is:
(a) (โˆ’
4
65
, โˆ’
7
65
) (b) (โˆ’
4
65
,
7
65
) (c) (
4
65
, โˆ’
7
65
) (d) โœ”(
4
65
,
7
65
)
19. (๐ŸŽ, ๐Ÿ‘)(๐ŸŽ, ๐Ÿ“) =
(a) 15 (b) โœ”-15 (c) โˆ’8๐‘– (d) 8๐‘–
20. (โˆ’๐Ÿ)โˆ’
๐Ÿ๐Ÿ
๐Ÿ =
(a) ๐‘– (b) โœ” โ€“ ๐‘– (c) 1 (d) -1
21. โˆš๐Ÿ‘ is __________
(a) Rational (b) โœ” Irrational (c) Integer (d) Prime
22. Product โˆšโˆ’๐Ÿ ร— โˆšโˆ’๐Ÿ is equal to _____
(a) -2 (b) โœ”2 (c) 0 (d) 4
23. The imaginary part of the complex number (๐’ƒ , ๐’‚) is _________
(a) โœ”๐‘Ž (b) ๐‘ (c) ๐‘–๐‘Ž (d) None of these
24. If ๐’› = โˆ’๐Ÿ โˆ’ ๐’Š then ๐’› =______
(a) (โˆ’1, โˆ’1) (b) โœ”(โˆ’1,1) (c) (1, โˆ’1) (d) (1,1)
25. The property ๐Ÿ•. ๐Ÿ– + (โˆ’๐Ÿ•. ๐Ÿ–) = ๐ŸŽ is______
(a) Commutative (b)โ€˜.โ€™ inverse (c) โœ” โ€˜ + โ€™ inverse (d) Associative
4 | P a g e
26. If ๐’™ = ๐ŸŽ then multiplicative inverse of ๐’™ is
(a) 0 (b) 1 (c)
1
๐‘ฅ
(d) โœ” None of these
27. (โˆ’๐’Š)๐Ÿ๐Ÿ“
=_______
(a) 1 (b) -1 (c) โœ” ๐‘– (d) โ€“ ๐‘–
28. If ๐’›1 and ๐’›2 are complex numbers then |๐’›1+๐’›2| is _______
(a) <|๐‘ง1+๐‘ง2| (b) โœ” โ‰ค|๐‘ง1 |+|๐‘ง2| (c) โ‰ฅ |๐‘ง1+๐‘ง2| (d) None of these
29. A recurring decimal represents_____
(a) Real number (b) Natural number (c) โœ”Rational number (d) None of these
30. Every recurring decimal is
(a) a rational number (b) an irrational number (c) a prime number (d) a whole number
31. Imaginary part of (โˆ’๐Ÿ + ๐Ÿ‘๐’Š๐Ÿ‘
) is
(a) -2 (b) 9 (c) 26 (d) โœ” -3
32. The product of two conjugate complex numbers is
(a) โœ”A real number (b) an imaginary number (c) may be an irrational number (d) not defined
33. (๐ŸŽ, ๐Ÿ)๐Ÿ‘
is equal to:
(a) โœ”1 (b) -1 (c) ๐‘– (d) โ€“ ๐‘–
34. If ๐’ is an even integer, then (๐’Š)๐’
is equal to:
(a) โœ”๐‘– (b) โ€“ ๐‘– (c) ยฑ1 (d) 1
35. Factors of ๐Ÿ‘(๐’™๐Ÿ
+ ๐’š๐Ÿ
) are:
(a) 3(๐‘ฅ + ๐‘ฆ)(๐‘ฅ โˆ’ ๐‘ฆ) (b) โœ” 3(๐‘ฅ + ๐‘–๐‘ฆ)(๐‘ฅ โˆ’ ๐‘–๐‘ฆ) (c) โˆš3(๐‘ฅ + ๐‘–๐‘ฆ)(๐‘ฅ โˆ’ ๐‘–๐‘ฆ) (d) None of these
36. Real part of
๐Ÿ+๐’Š
๐’Š
is:
(a) 1 (b) โœ”- 2 (c) -1 (d)
1
2
UNIT # 02 Sets, Functions and Groups
Each question has four possible answer. Tick the correct answer.
1. If ๐’™ โˆˆ ๐‘ณ โˆช ๐‘ด then
(a) ๐‘ฅ โˆ‰ ๐ฟ ๐‘œ๐‘Ÿ ๐‘ฅ โˆ‰ ๐‘€ (b) ๐‘ฅ โˆ‰ ๐ฟ ๐‘œ๐‘Ÿ ๐‘ฅ โˆˆ ๐‘€ (c) ๐‘ฅ โˆˆ ๐ฟ ๐‘œ๐‘Ÿ ๐‘ฅ โˆ‰ ๐‘€ (d) โœ”๐‘ฅ โˆˆ ๐ฟ ๐‘œ๐‘Ÿ ๐‘ฅ โˆˆ ๐‘€
2. Total number of subsets that can be formed from the set {๐’™, ๐’š, ๐’›} is
(a) 1 (b) โœ” 8 (c) 5 (d)2
3. If ๐’™ โˆˆ ๐‘ฉโ€ฒ
= ๐‘ผ โˆ’ ๐‘ฉ then
(a) ๐‘ฅ โˆˆ ๐ต ๐‘Ž๐‘›๐‘‘ ๐‘ฅ โˆˆ ๐‘ˆ (b) โœ”๐‘ฅ โˆ‰ ๐ต ๐‘Ž๐‘›๐‘‘ ๐‘ฅ โˆˆ ๐‘ˆ (c) ๐‘ฅ โˆ‰ ๐ต ๐‘Ž๐‘›๐‘‘ ๐‘ฅ โˆ‰ ๐‘ˆ (d)๐‘ฅ โˆˆ ๐ต ๐‘Ž๐‘›๐‘‘ ๐‘ฅ โˆ‰ ๐‘ˆ
4. ๐‘ณ โˆช ๐‘ด = ๐‘ณ โˆฉ ๐‘ดthen ๐‘ณ is equal to
(a) โœ”๐‘€ (b) ๐ฟ (c) ๐œ‘ (d) ๐‘€โ€ฒ
5. A set is a collection of objects which are
(a) Well defined (b) โœ”Well defined and distinct (c) identical (d) not defined
6. The set of odd numbers between 1 and 9 are
(a) {1,3,5,7} (b) {3,5,7,9} (c) {1,3,5,7,9} (d) โœ”{3,5,7}
7. Every recurring non terminating decimal represents
(a) โœ”๐‘„ (b) ๐‘„โ€ฒ (c) ๐‘… (d) None
8. A diagram which represents a set is called______
(a) โœ”Vennโ€™s (b) Argand (c) Plane (d) None
9. ๐‘น โˆ’ {๐ŸŽ} is a group ๐’˜. ๐’“. ๐’• the binary operation
(a) + (b) โœ” ร— (c) รท (d)โˆ’
10. In a proposition if ๐’‘ โ†’ ๐’’ ๐’•๐’‰๐’†๐’ ๐’’ โ†’ ๐’‘ is called
(a) Inverse of ๐‘ โ†’ ๐‘ž (b) โœ” converse of ๐‘ โ†’ ๐‘ž (c) contrapositive of ๐‘ โ†’ ๐‘ž(d) None
11. If ๐‘จ and ๐‘ฉ are disjoint sets , then ๐‘จ โˆฉ ๐‘ฉ =
(a) 0 (b) 1 (c) 2 (d) โœ” ๐œ‘
12. ๐’‘ โ†’ ๐’’ is called converse of
(a) ~๐‘๐‘ž (b) โœ” ๐‘ โ†’ ๐‘ž (c) ๐‘ž โ†’ ๐‘ž (d) ~๐‘ž โ†’ ๐‘
13. If ๐‘จ โŠ† ๐‘ฉ then ๐’(๐‘จ โˆฉ ๐‘ฉ) =
(a) โœ”๐‘›(๐ด) (b) ๐‘›(๐ต) (c) ๐‘›(๐ด) + ๐‘›(๐ต) (d) ๐‘›(๐ด). ๐‘›(๐ต)
14. Inverse of any element of a group is:
(a) Not unique (b) โœ”unique (c) has many inverses (d) none of these
15. Every function is:
(a) โœ”Relation (b) inverse function (c) one to one (d) none of these
16. The number of all subsets of a set having three elements is:
(a) 4 (b) 6 (c) โœ” 8 (d) 10
5 | P a g e
17. The graph of linear function is :
(a) Circle ( b) โœ”straight line (c) parabola (d) triangle
18. Identity element in (๐‘ช, . ) is:
(a) (0,0) (b) (0,1) (c) โœ” (1,0) (d) (1,1)
19. Squaring a number is a:
(a) โœ”Unary operation (b) binary operation (c) relation (d) Function
20. Which of the following is true:
(a) ๐‘ โŠ‚ ๐‘ (b) ๐‘ โŠ‚ ๐‘„ (c) ๐‘„ โŠ‚ ๐‘… (d) โœ”all of these
21. Set of integers is a group ๐’˜. ๐’“. ๐’•
(a) โœ”Addition (b) multiplication (c) subtraction (d) division
22. If ๐‘จ = ๐‹ then ๐‘ท(๐‘จ) =
(a) Empty set (b) {0} (c) โœ” {๐œ‘} (d) none of these
23. If ๐‘จ and ๐‘ฉ are disjoint sets then :
(a) โœ”๐ด โˆฉ ๐ต = ๐œ‘ (b) ๐ด โˆฉ ๐ต โ‰  ๐œ‘ (c) ๐ด โŠ‚ ๐ต (d) ๐ด โˆ’ ๐ต = ๐œ‘
24. The set of non-zero real numbers ๐’˜. ๐’“. ๐’• multiplication is:
(a) Groupied (b) Semi-group (c) Monoied (d) โœ” Group
25. Which of the following is not a binary operation :
(a) + (b) ร— (c) โœ” โˆš (d) โ€“
26. If ๐’š = โˆš๐’™ , ๐’™ โ‰ฅ ๐ŸŽ is a function, then its inverse is:
(a) A line (b) a parabola (c) a point (d) โœ” not a function
27. An onto function is also called:
(a) Injective (b) โœ” Surjective (c) Bijective (d) Inverse
28. A (๐Ÿ โˆ’ ๐Ÿ) function is also called:
(a) โœ”Injective (b) Surjective (c) Bijective (d) Inverse
29. For the propositions ๐’‘ and ๐’’, (๐’‘ โˆง ๐’’) โ†’ ๐’‘ is:
(a) โœ”Tautology (b) Absurdity (c) contingency (d) None of these
30. For the propositions ๐’‘ and ๐’’ , ๐’‘ โ†’ (๐’‘ โˆจ ๐’’) is:
(a) โœ”Tautology (b) Absurdity (c) Contingency (d) None of these
31. If set ๐‘จ has 2 elements and ๐‘ฉ has 4 elements , then number of elements in ๐‘จ ร— ๐‘ฉ is :
(a) 6 (b) โœ” 8 (c) 16 (d) None of these
32. Inverse of a line is :
(a) โœ”A line (b) a parabola (c) a point (d) not defined
33. The function ๐’‡ = {(๐’™, ๐’š), ๐’š = ๐’™} is :
(a) โœ”Identity function (b) Null function (c) not a function (d) similar function
34. If ๐’š = โˆš๐’™, ๐’™ โ‰ฅ ๐ŸŽ is a function, then its inverse is :
(a) A line (b) a parabola (c) a point (d) โœ” not defined
35. Truth set of a tautology is
(a) โœ”Universal set (b) ๐œ‘ (c) True (d) False
36. A compound proposition which is always true is called:
(a) โœ”Tautology (b) contradiction (c) absurdity (d) contingency
37. A compound proposition which is always neither true nor false is called:
(a) Tautology (b) contradiction (c) absurdity (d) โœ” contingency
38. A compound proposition which is always wrong is called:
(a) Tautology (b) โœ” contradiction (c) absurdity (d) contingency
39. If ๐‘บ = { } , then order of set ๐‘บ is:
(a) โœ”0 (b) 1 (c) Infinite set (d) not defined
40. The symbol which is used to denote negation of a proposition is
(a) โœ”~ (b) โ†’ (c) โˆง (d) โˆจ
41. Which of the following is true:
(a) ๐œ‘ โˆ’ ๐ด = ๐œ‘ (b) ๐ด โˆช ๐ด = ๐ด (c) ๐ด โˆฉ ๐ด = ๐ด (d) โœ”All of these
42. Which of the following is true:
(a) ๐ด โˆช ๐œ‘ = ๐ด (b) ๐ด โˆฉ ๐œ‘ = ๐œ‘ (c) ๐ด โˆ’ ๐œ‘ = ๐ด (d) โœ”All of these
43. If ๐‘ฉ โŠ† ๐‘จ, then ๐’(๐‘ฉ โˆ’ ๐‘จ) is equal to:
(a) ๐‘›(๐ด) (b) ๐‘›(๐ต) (c) ๐‘›(๐ด โˆฉ ๐ต) (d) โœ” 0
6 | P a g e
UNIT # 03 Matrices and Determinants
Each question has four possible answer. Tick the correct answer.
1. A rectangular array of numbers enclosed by a square brackets is called:
(a) โœ”Matrix (b) Row (c) Column (d) Determinant
2. The horizontal lines of numbers in a matrix are called:
(a) Columns (b) โœ” Rows (c) Column matrix (d) Row matrix
3. The vertical lines of numbers in a matrix are called:
(a) โœ”Columns (b) Rows (c) Column matrix (d) Row matrix
4. If a matrix A has ๐’Ž rows and ๐’ columns , then order of A is :
(a) โœ”๐‘š ร— ๐‘› (b) ๐‘› ร— ๐‘š (c) ๐‘š + ๐‘› (d) ๐‘š๐‘›
5. The element ๐’‚๐’Š๐’‹ of any matrix A is present in:
(a) โœ”๐‘–๐‘กโ„Ž
row and ๐‘—๐‘กโ„Ž
column (b)๐‘–๐‘กโ„Ž
column and ๐‘—๐‘กโ„Ž
row
(c) (๐‘– + ๐‘—)๐‘กโ„Ž
row and column (d) (๐‘– + ๐‘—)๐‘กโ„Ž
row and column
6. A matrix A is called real if all ๐’‚๐’Š๐’‹ are
(a) โœ”Real numbers (b) complex numbers (c) 0 (d) 1
7. If a matrix A has only one row then it is called:
(a) โœ”Row vector (b) Column vector (c) square matrix (d) Rectangle matrix
8. If a matrix A has only one column then it is called:
(a) Row vector (b) โœ” Column vector (c) square matrix (d) Rectangle matrix
9. If a matrix A has same number of rows and columns then A is called:
(a) Row vector (b) Column vector (c) โœ”square matrix (d) Rectangular matrix
10. If a matrix A has different number of rows and columns then A is called:
(a) Row vector (b) Column vector (c) square matrix (d) โœ” Rectangular matrix
11. For the square matrix ๐‘จ = [๐’‚๐’Š๐’‹]๐’ร—๐’ then ๐’‚๐Ÿ๐Ÿ, ๐’‚๐Ÿ๐Ÿ, ๐’‚๐Ÿ‘๐Ÿ‘โ€ฆ๐’‚๐’๐’ are:
(a) โœ”Main diagonal (b) primary diagonal (c) proceding diagonal (d) secondary diagonal
12. For a square matrix ๐‘จ = [๐’‚๐’Š๐’‹]if all ๐’‚๐’Š๐’‹ = ๐ŸŽ, ๐’Š โ‰  ๐’‹ and all ๐’‚๐’Š๐’‹ = ๐’Œ(๐’๐’๐’ โˆ’ ๐’›๐’†๐’“๐’) for ๐’Š = ๐’‹ then A
is called:
(a) Diagonal matrix (b) โœ” Scalar Matrix (c) Scalar Matrix (d) Null matrix
13. For a square matrix ๐‘จ = [๐’‚๐’Š๐’‹]if all ๐’‚๐’Š๐’‹ = ๐ŸŽ, ๐’Š โ‰  ๐’‹ and at least ๐’‚๐’Š๐’‹ โ‰  ๐ŸŽ, ๐’Š = ๐’‹ then A is called:
(a) โœ”Diagonal matrix (b) Scalar Matrix (c) Scalar Matrix (d) Null matrix
14. The matrix [๐Ÿ•] is :
(a) Square matrix (b) row matrix (c) column matrix (d) โœ” all of these
15. If a matrix of order ๐’Ž ร— ๐’ then the matrix of order ๐’ ร— ๐’Ž is :
(a) โœ”Transpose of A (b) Inverse of A (c) Main diagonal of A (d) Adjoint of A
16. For any two matrices A and B ,(๐‘จ + ๐‘ฉ)๐’•
=
(a) ๐ต๐‘ก
๐ด๐‘ก
(b) ๐ด๐‘ก
๐ต๐‘ก
(c) โœ”๐ด๐‘ก
+ ๐ต๐‘ก
(d) ๐ต๐‘ก
+ ๐ด๐‘ก
17. Let A be a matrix and ๐’ is an integer then ๐‘จ + ๐‘จ + ๐‘จ + โ‹ฏ to ๐’ terms
(a) โœ”๐‘›๐ด (b) ๐‘› + ๐ด (c) ๐‘› โˆ’ ๐ด (d) ๐ด๐‘›
18. If order of ๐‘จ is ๐’Ž ร— ๐’ and order of B is ๐’ ร— ๐’’ then order of AB is
(a) ๐‘š ร— ๐‘› (b) ๐‘› ร— ๐‘š (d) ๐‘› ร— ๐‘ž (d) โœ”๐‘š ร— ๐‘ž
19. If ๐’‚๐’‹๐’… ๐‘จ = [
โˆ’๐Ÿ โˆ’๐Ÿ
๐Ÿ‘ ๐Ÿ’
] then matrix ๐‘จ is
(a) [
โˆ’1 โˆ’2
4 3
] (b) [
4 2
3 โˆ’1
] (c) [
โˆ’4 3
โˆ’2 โˆ’1
] (d) โœ” [
4 2
โˆ’3 โˆ’1
]
20. If ๐‘จ is non-singular matrix then ๐‘จโˆ’๐Ÿ
=
(a) โœ”
1
|๐ด|
๐‘Ž๐‘‘๐‘—๐ด (b) โˆ’
1
|๐ด|
๐‘Ž๐‘‘๐‘—๐ด (c)
|๐ด|
๐‘Ž๐‘‘๐‘—๐ด
(d)
1
|๐ด|๐‘Ž๐‘‘๐‘—๐ด
21. If ๐‘จ๐‘ฟ = ๐‘ฉ then ๐‘ฟ is equal to:
(a) ๐ด๐ต (b) โœ” ๐ดโˆ’1
๐ต (c) ๐ตโˆ’1
๐ด (d) ๐ต๐ด
22. Inverse of a matrix exists if it is:
(a) Singular (b) Null (c) Rectangular (d) โœ” Non-singular
23. Which of the property does not hold matrix multiplication?
(a) Associative (b) โœ” Commutative (c) Closure (d) Inverse
24. For any matrix A , it is always true that
(a) ๐ด = ๐ด๐‘ก
(b) โ€“ ๐ด = ๐ดฬ… (c) โœ”|๐ด| = |๐ด๐‘ก
| (d) ๐ดโˆ’1
=
1
๐ด
25. If all entries of a square matrix of order ๐Ÿ‘ is multiplied by ๐’Œ, then value of |๐’Œ๐‘จ| is equal to:
(a) ๐‘˜|๐ด| (b) ๐‘˜2
|๐ด| (c) โœ”๐‘˜3
|๐ด| (d) |๐ด|
7 | P a g e
26. For a non-singular matrix it is true that :
(a) (๐ดโˆ’1)โˆ’1
= ๐ด (b) (๐ด๐‘ก)๐‘ก
= ๐ด (c) ๐ดฬฟ = ๐ด (d) โœ”all of these
27. For any non-singular matrices A and B it is true that:
(a) (๐ด๐ต)โˆ’1
= ๐ตโˆ’1
๐ดโˆ’1
(b) (๐ด๐ต)๐‘ก
= ๐ต๐‘ก
๐ด๐‘ก
(c) ๐ด๐ต โ‰  ๐ต๐ด (d) โœ” all of these
28. A square matrix ๐‘จ = [๐’‚๐’Š๐’‹] for which ๐’‚๐’Š๐’‹ = ๐ŸŽ, ๐’Š > ๐‘— then A is called:
(a) โœ”Upper triangular (b) Lower triangular (c) Symmetric (d) Hermitian
29. A square matrix ๐‘จ = [๐’‚๐’Š๐’‹] for which ๐’‚๐’Š๐’‹ = ๐ŸŽ, ๐’Š < ๐‘— then A is called:
(a) Upper triangular (b) โœ”Lower triangular (c) Symmetric (d) Hermitian
30. Any matrix A is called singular if:
(a) โœ”|๐ด| = 0 (b) |๐ด| โ‰  0 (c) ๐ด๐‘ก
= ๐ด (d) ๐ด๐ดโˆ’1
= ๐ผ
31. Which of the following Sets is a field.
(a) R (b) Q (c) C (d) โœ”all of these
32. Which of the following Sets is not a field.
(a) R (b) Q (c) C (d) โœ”Z
33. A square matrix A is symmetric if:
(a) โœ”๐ด๐‘ก
= ๐ด (b) ๐ด๐‘ก
= โˆ’๐ด (c) (๐ด)
๐‘ก
= ๐ด (d) (๐ด)
๐‘ก
= โˆ’๐ด
34. A square matrix A is skew symmetric if:
(a) ๐ด๐‘ก
= ๐ด (b) โœ” ๐ด๐‘ก
= โˆ’๐ด (c) (๐ด)
๐‘ก
= ๐ด (d) (๐ด)
๐‘ก
= โˆ’๐ด
35. A square matrix A is Hermitian if:
(a) ๐ด๐‘ก
= ๐ด (b) ๐ด๐‘ก
= โˆ’๐ด (c) โœ”(๐ด)
๐‘ก
= ๐ด (d) (๐ด)
๐‘ก
= โˆ’๐ด
36. A square matrix A is skew- Hermitian if:
(a) ๐ด๐‘ก
= ๐ด (b) ๐ด๐‘ก
= โˆ’๐ด (c) (๐ด)
๐‘ก
= ๐ด (d) โœ”(๐ด)
๐‘ก
= โˆ’๐ด
37. The main diagonal elements of a skew symmetric matrix must be:
(a) 1 (b) โœ” 0 (c) any non-zero number (d) any complex number
38. The main diagonal elements of a skew hermitian matrix must be:
(a) 1 (b) โœ” 0 (c) any non-zero number (d) any complex number
39. In echelon form of matrix, the first non zero entry is called:
(a) โœ”Leading entry (b) first entry (c) preceding entry (d) Diagonal entry
40. The additive inverse of a matrix exist only if it is:
(a) Singular (b) non singular (c) null matrix (d) โœ” any matrix of order ๐‘š ร— ๐‘›
41. The multiplicative inverse of a matrix exist only if it is:
(a) Singular (b) โœ” non singular (c) null matrix (d) any matrix of order ๐‘š ร— ๐‘›
42. The number of non zero rows in echelon form of a matrix is called:
(a) Order of matrix (b) Rank of matrix (c) leading (d) leading row
43. If A is any square matrix then ๐‘จ + ๐‘จ๐’•
is a
(a) โœ”Symmetric (b) skew symmetric (c) hermitian (d) skew hermitian
44. If A is any square matrix then ๐‘จ โˆ’ ๐‘จ๐’•
is a
(a) Symmetric (b) โœ”skew symmetric (c) hermitian (d) skew hermitian
45. If A is any square matrix then ๐‘จ + (๐‘จ)
๐’•
is a
(a) Symmetric (b) skew symmetric (c) โœ” hermitian (d) skew hermitian
46. If A is any square matrix then ๐‘จ + (๐‘จ)
๐’•
is a
(a) Symmetric (b) skew symmetric (c) hermitian (d) โœ” skew hermitian
47. If A is symmetric (Skew symmetric), then ๐‘จ๐Ÿ
must be
(a) Singular (b) non singular (c) โœ”symmetric (d) non trivial solution
48. In a homogeneous system of linear equations , the solution (0,0,0) is:
(a) โœ”Trivial solution (b) non trivial solution (c) exact solution (d) anti symmetric
49. If ๐‘จ๐‘ฟ = ๐‘ถ then ๐‘ฟ =
(a) ๐ผ (b) โœ” ๐‘‚ (c) ๐ดโˆ’1
(d) Not possible
50. If the system of linear equations have no solution at all, then it is called a/an
(a) Consistent system (b)โœ” Inconsistent system (c) Trivial System (d) Non Trivial
System
51. The value of ๐€ for which the system ๐’™ + ๐Ÿ๐’š = ๐Ÿ’; ๐Ÿ๐’™ + ๐€๐’š = โˆ’๐Ÿ‘ does not possess the unique
solution
(a) โœ”4 (b) -4 (c) ยฑ4 (d) any real number
52. If the system ๐’™ + ๐Ÿ๐’š = ๐ŸŽ; ๐Ÿ๐’™ + ๐€๐’š = ๐ŸŽ has non-trivial solution, then ๐€ is:
(a) โœ”4 (b) -4 (c) ยฑ4 (d) any real number
53. The inverse of unit matrix is:
(a) โœ”Unit (b) Singular (c) Skew Symmetric (d) rectangular
8 | P a g e
54. Transpose of a row matrix is:
(a) Diagonal matrix (b) zero matrix (c) โœ” column matrix (d) scalar matrix
55. If |
๐’™ ๐Ÿ’
๐Ÿ“ ๐Ÿ๐ŸŽ
| = ๐ŸŽ โ‡’ ๐’™ equals
(a) โœ”2 (b) 4 (c) 6 (d) 8
UNIT # 04 Quadratic Equations
Each question has four possible answer. Tick the correct answer.
1. The equation ๐’‚๐’™๐Ÿ
+ ๐’ƒ๐’™ + ๐’„ = ๐ŸŽ will be quadratic if:
(a) ๐‘Ž = 0, ๐‘ โ‰  0 (b) โœ” ๐‘Ž โ‰  0 (c) ๐‘Ž = ๐‘ = 0 (d) ๐‘ = any real number
2. Solution set of the equation ๐’™๐Ÿ
โˆ’ ๐Ÿ’๐’™ + ๐Ÿ’ = ๐ŸŽ is:
(a) {2, โˆ’2} (b) โœ” {2} (c) {โˆ’2} (d) {4, โˆ’4}
3. The quadratic formula for solving the equation ๐’‚๐’™๐Ÿ
+ ๐’ƒ๐’™ + ๐’„ = ๐ŸŽ; ๐’‚ โ‰  ๐ŸŽ is
(a) โœ”๐‘ฅ =
โˆ’๐‘ยฑโˆš๐‘2
โˆ’4๐‘Ž๐‘
2๐‘Ž
(b) ๐‘ฅ =
โˆ’๐‘ยฑโˆš๐‘Ž2โˆ’4๐‘Ž๐‘
2๐‘
(c) ๐‘ฅ =
โˆ’๐‘Žยฑโˆš๐‘2โˆ’4๐‘Ž๐‘
2
(d) None of these
4. To convert ๐’‚๐’™๐Ÿ๐’
+ ๐’ƒ๐’™๐’
+ ๐’„ = ๐ŸŽ(๐’‚ โ‰  ๐ŸŽ) into quadratic form , the correct substitution is:
(a) โœ”๐‘ฆ = ๐‘ฅ๐‘›
(b) ๐‘ฅ = ๐‘ฆ๐‘›
(c) ๐‘ฆ = ๐‘ฅโˆ’๐‘›
(d) ๐‘ฆ =
1
๐‘ฅ
5. The equation in which variable occurs in exponent , called:
(a) Exponential function (b) Quadratic equation (c) Reciprocal equation (d) โœ”Exponential equation
6. To convert ๐Ÿ’๐Ÿ+๐’™
+ ๐Ÿ’๐Ÿโˆ’๐’™
= ๐Ÿ๐ŸŽ into quadratic , the substitution is:
(a) ๐‘ฆ = ๐‘ฅ1โˆ’๐‘ฅ
(b) ๐‘ฆ = 41+๐‘ฅ
(c) ๐‘ฆ = 4๐‘ฅ
(d) ๐‘ฆ = 4โˆ’๐‘ฅ
7. The equations involving redical expressions of the variable are called:
(a) Reciprocal equations(b) โœ” Redical equations (c) Quadratic functions (d) exponential equations
8. The cube roots of unity are :
(a) โœ”1,
โˆ’1+โˆš3๐‘–
2
,
โˆ’1+โˆš3๐‘–
2
(b) 1,
1+โˆš3๐‘–
2
,
1+โˆš3๐‘–
2
(c) โˆ’1,
โˆ’1+โˆš3๐‘–
2
,
โˆ’1+โˆš3๐‘–
2
(d) โˆ’1,
1+โˆš3๐‘–
2
,
1+โˆš3๐‘–
2
9. Sum of all cube roots of 64 is :
(a) โœ”0 (b) 1 (c) 64 (d) -64
10. Product of cube roots of -1 is:
(a) 0 (b) -1 (c) โœ”1 (d) None
11. ๐Ÿ๐Ÿ”๐Ž๐Ÿ–
+ ๐Ÿ๐Ÿ”๐’˜๐Ÿ’
=
(a) 0 (b) โœ” -16 (c) 16 (d) -1
12. The sum of all four fourth roots of unity is:
(a) Unity (b) โœ”0 (c) -1 (d) None
13. The product of all four fourth roots of unity is:
(a) Unity (b) 0 (c) โœ”-1 (d) None
14. The sum of all four fourth roots of 16 is:
(a) 16 (b) -16 (c) โœ” 0 (d) 1
15. The complex cube roots of unity areโ€ฆโ€ฆโ€ฆโ€ฆโ€ฆโ€ฆ. each other.
(a) Additive inverse (b) Equal to (c) โœ”Conjugate (d) None of these
16. The complex cube roots of unity areโ€ฆโ€ฆโ€ฆโ€ฆโ€ฆโ€ฆ. each other.
(a) Multiplicative inverse (b) reciprocal (c) square (d) โœ”None of these
17. The complex fourth roots of unity are โ€ฆโ€ฆ. of each other.
(a) โœ”Additive inverse (b) equal to (c) square of (d) None of these
18. If sum of all cube roots of unity is equal to ๐’™๐Ÿ
+ ๐Ÿ , then ๐’™ is equal to:
(a) โˆ’1 (b) 0 (c) โœ”ยฑ๐‘– (d) 1
19. If product of all cube roots of unity is equal to ๐’‘๐Ÿ
+ ๐Ÿ , then ๐’‘ is equal to:
(a) โˆ’1 (b) โœ” 0 (c) ยฑ๐‘– (d) 1
20. The expression ๐’™๐Ÿ
+
๐Ÿ
๐’™
โˆ’ ๐Ÿ‘ is polynomial of degree:
(a) 2 (b) 3 (c) 1 (d) โœ” not a polynomial
21. If ๐’‡(๐’™) is divided by ๐’™ โˆ’ ๐’‚ , then dividend = (Divisor)(โ€ฆโ€ฆ.)+ Remainder.
(a) Divisor (b) Dividend (c) โœ” Quotient (d) ๐‘“(๐‘Ž)
22. If ๐’‡(๐’™) is divided by ๐’™ โˆ’ ๐’‚ by remainder theorem then remainder is:
(a) โœ” ๐‘“(๐‘Ž) (b) ๐‘“(โˆ’๐‘Ž) (c) ๐‘“(๐‘Ž) + ๐‘… (d) ๐‘ฅ โˆ’ ๐‘Ž = ๐‘…
23. The polynomial (๐’™ โˆ’ ๐’‚) is a factor of ๐’‡(๐’™) if and only if
(a) โœ” ๐‘“(๐‘Ž) = 0 (b) ๐‘“(๐‘Ž) = ๐‘… (c) Quotient = ๐‘… (d) ๐‘ฅ = โˆ’๐‘Ž
24. ๐’™ โˆ’ ๐Ÿ is a factor of ๐’™๐Ÿ
โˆ’ ๐’Œ๐’™ + ๐Ÿ’, if ๐’Œ is:
(a) 2 (b) โœ” 4 (c) 8 (d) -4
25. If ๐’™ = โˆ’๐Ÿ is the root of ๐’Œ๐’™๐Ÿ’
โˆ’ ๐Ÿ๐Ÿ‘๐’™๐Ÿ
+ ๐Ÿ‘๐Ÿ” = ๐ŸŽ, then ๐’Œ =
9 | P a g e
(a) 2 (b) -2 (c) 1 (d) โœ” -1
26. ๐’™ + ๐’‚ is a factor of ๐’™๐’
+ ๐’‚๐’
when ๐’ is
(a) Any integer (b) any positive integer (c) โœ” any odd integer (d) any real number
27. ๐’™ โˆ’ ๐’‚ is a factor of ๐’™๐’
โˆ’ ๐’‚๐’
when ๐’ is
(a) โœ” Any integer (b) any positive integer (c) any odd integer (d) any real number
28. Sum of roots of ๐’‚๐’™๐Ÿ
โˆ’ ๐’ƒ๐’™ โˆ’ ๐’„ = ๐ŸŽ is (๐’‚ โ‰  ๐ŸŽ)
(a) โœ”
๐‘
๐‘Ž
(b) โ€“
๐‘
๐‘Ž
(c)
๐‘
๐‘Ž
(d) โ€“
๐‘
๐‘Ž
29. Sum of roots of ๐’‚๐’™๐Ÿ
โˆ’ ๐’ƒ๐’™ โˆ’ ๐’„ = ๐ŸŽ is (๐’‚ โ‰  ๐ŸŽ)
(a)
๐‘
๐‘Ž
(b) โ€“
๐‘
๐‘Ž
(c)
๐‘
๐‘Ž
(d) โœ” โ€“
๐‘
๐‘Ž
30. If 2 and -5 are roots of a quadratic equation , then equation is:
(a) ๐‘ฅ2
โˆ’ 3๐‘ฅ โˆ’ 10 = 0 (b) ๐‘ฅ2
โˆ’ 3๐‘ฅ + 10 = 0 (c) โœ” ๐‘ฅ2
+ 3๐‘ฅ โˆ’ 10 = 0 (d) ๐‘ฅ2
+ 3๐‘ฅ + 10 = 0
31. If ๐œถ and ๐œท are the roots of ๐Ÿ‘๐’™๐Ÿ
โˆ’ ๐Ÿ๐’™ + ๐Ÿ’ = ๐ŸŽ, then the value of ๐œถ + ๐œท is:
(a) โœ”
2
3
(b) โˆ’
2
3
(c)
4
3
(d) โˆ’
4
3
32. If roots of ๐’‚๐’™๐Ÿ
+ ๐’ƒ๐’™ + ๐’„ = ๐ŸŽ, (๐’‚ โ‰  ๐ŸŽ) are real , then
(a) Discโ‰ฅ 0 (b) Disc< 0 (c) Discโ‰  0 (d) Discโ‰ค 0
33. If roots of ๐’‚๐’™๐Ÿ
+ ๐’ƒ๐’™ + ๐’„ = ๐ŸŽ, (๐’‚ โ‰  ๐ŸŽ) are complex , then
(a) โœ”Discโ‰ฅ 0 (b) โœ” Disc< 0 (c) Discโ‰  0 (d) Discโ‰ค 0
34. If roots of ๐’‚๐’™๐Ÿ
+ ๐’ƒ๐’™ + ๐’„ = ๐ŸŽ, (๐’‚ โ‰  ๐ŸŽ) are equal , then
(a) โœ”Disc= 0 (b) Disc< 0 (c) Discโ‰  0 (d) None of these
35. Graph of quadratic equation is:
(a) Straight line (b) Circle (c) square (d) โœ”Parabola
36. ๐Ž๐Ÿ๐Ÿ–
+ ๐Ž๐Ÿ๐Ÿ—
+ ๐Ÿ =
(a) โœ”0 (b) 1 (c) -1 (d) ๐œ”
37. Synthetic division is a process of:
(a) Addition (b) multiplication (c) subtraction (d) โœ” division
38. Degree of quadratic equation is:
(a) 0 (b)1 (c) โœ”2 (d) 3
39. Basic techniques for solving quadratic equations are
(a) 1 (b) 2 (c) โœ”3 (d) 4
40. ๐Ÿ๐Ÿ”๐Ž๐Ÿ’
+ ๐Ÿ๐Ÿ”๐Ž๐Ÿ–
=
(a) 0 (b) โœ” -16 (c) 16 (d) -1
UNIT # 05 Partial Fractions
Each question has four possible answer. Tick the correct answer.
1. An open sentence formed by using sign of โ€œ = โ€ is called a/an
(a) โœ”Equation (b) Formula (c) Rational fraction (d) Theorem
2. If an equation is true for all values of the variable, then it is called:
(a) a conditional equation (b) โœ” an identity (c) proper rational fraction (d) All of these
3. (๐’™ + ๐Ÿ‘)(๐’™ + ๐Ÿ’) = ๐’™๐Ÿ
+ ๐Ÿ•๐’™ + ๐Ÿ๐Ÿ is a/an:
(a) Conditional equation (b) โœ”an identity (c) proper rational fraction (d) a formula
4. The quotient of two polynomials
๐‘ท(๐’™)
๐‘ธ(๐’™)
, ๐‘ธ(๐’™) โ‰  ๐ŸŽ is called :
(a) โœ”Rational fraction (b) Irrational fraction (c) Partial fraction (d) Proper fraction
5. A fraction
๐‘ท(๐’™)
๐‘ธ(๐’™)
, ๐‘ธ(๐’™) โ‰  ๐ŸŽ is called proper fraction if :
(a) โœ”Degree of ๐‘ƒ(๐‘ฅ) < Degree of ๐‘„(๐‘ฅ) (b) Degree of ๐‘ƒ(๐‘ฅ) = Degree of ๐‘„(๐‘ฅ)
(c) Degree of ๐‘ƒ(๐‘ฅ) > Degree of ๐‘„(๐‘ฅ) (d) Degree of ๐‘ƒ(๐‘ฅ) โ‰ฅ Degree of ๐‘„(๐‘ฅ)
6. A fraction
๐‘ท(๐’™)
๐‘ธ(๐’™)
, ๐‘ธ(๐’™) โ‰  ๐ŸŽ is called proper fraction if :
(a) Degree of ๐‘ƒ(๐‘ฅ) < Degree of ๐‘„(๐‘ฅ) (b) Degree of ๐‘ƒ(๐‘ฅ) = Degree of ๐‘„(๐‘ฅ)
(c) Degree of ๐‘ƒ(๐‘ฅ) > Degree of ๐‘„(๐‘ฅ) (d) โœ” Degree of ๐‘ƒ(๐‘ฅ) โ‰ฅ Degree of ๐‘„(๐‘ฅ)
7. A mixed form of fraction is :
(a) An integer+ improper fraction (b) a polynomial+improper fraction
(c) โœ”a polynomial+proper fraction (d) a polynomial+rational fraction
8. When a rational fraction is separated into partial fractions, then result is always :
(a) A conditional equations (b) โœ”an identity (c) a partial fraction (d) an improper fraction
10 | P a g e
9. The number of Partial fraction of
๐’™๐Ÿ‘
๐’™(๐’™+๐Ÿ)(๐’™๐Ÿโˆ’๐Ÿ)
are:
(a) 2 (b) 3 (c) โœ”4 (d) None of these
10. The number of Partial fraction of
๐’™๐Ÿ“
๐’™(๐’™+๐Ÿ)(๐’™๐Ÿโˆ’๐Ÿ’)
are:
(a) 2 (b) 3 (c) 4 (d) โœ” 6
11. Partial fractions of
๐Ÿ
๐’™๐Ÿโˆ’๐Ÿ
are:
(a)
1
2(๐‘ฅโˆ’1)
+
1
2(๐‘ฅ+1)
(b) โœ”
1
2(๐‘ฅโˆ’1)
โˆ’
1
2(๐‘ฅ+1)
(c) โˆ’
1
2(๐‘ฅโˆ’1)
+
1
2(๐‘ฅ+1)
(d) โˆ’
1
2(๐‘ฅโˆ’1)
โˆ’
1
2(๐‘ฅ+1)
12. Conditional equation ๐Ÿ๐’™ + ๐Ÿ‘ = ๐ŸŽ holds when ๐’™ is equal to:
(a) โœ”โˆ’
3
2
(b)
3
2
(c)
1
3
(d) 1
13. Which is a reducible factor:
(a) ๐‘ฅ3
โˆ’ 6๐‘ฅ2
+ 8๐‘ฅ (b) ๐‘ฅ2
+ 16๐‘ฅ (c) ๐‘ฅ2
+ 5๐‘ฅ โˆ’ 6 (d) โœ”all of these
14. A quadratic factor which cannot written as a product of linear factors with real coefficients is
called:
(a) โœ”An irreducible factor (b) reducible factor (c) an irrational factor (d) an improper factor
15.
๐Ÿ—๐’™๐Ÿ
๐’™๐Ÿ‘โˆ’๐Ÿ
is an
(a) Improper fraction (b) โœ” Proper fraction (c) Polynomial (d) equation
UNIT # 06 Sequence and Series
Each question has four possible answer. Tick the correct answer.
1. An arrangement of numbers according to some definite rule is called:
(a) โœ”Sequence (b) Combination (c) Series (d) Permutation
2. A sequence is also known as:
(a) Real sequence (b) โœ”Progression (c) Arrangement (d) Complex sequence
3. A sequence is function whose domain is
(a) ๐‘ (b) ๐‘ (c) ๐‘„ (d) โœ” ๐‘…
4. As sequence whose range is ๐‘น ๐’Š. ๐’†., set of real numbers is called:
(a) โœ”Real sequence (b) Imaginary sequence (c) Natural sequence (d) Complex sequence
5. If ๐’‚๐’ = {๐’ + (โˆ’๐Ÿ)๐’}, then ๐’‚๐Ÿ๐ŸŽ =
(a) 10 (b) โœ” 11 (c) 12 (d) 13
6. The last term of an infinite sequence is called :
(a) ๐‘›๐‘กโ„Ž term (b) ๐‘Ž๐‘› (c) last term (d) โœ” does not exist
7. The next term of the sequence ๐Ÿ, ๐Ÿ, ๐Ÿ๐Ÿ, ๐Ÿ’๐ŸŽ, โ€ฆ is
(a) โœ”112 (b) 120 (c) 124 (d) None of these
8. A sequence {๐’‚๐’} in which ๐’‚๐’ โˆ’ ๐’‚๐’โˆ’๐Ÿ is the same number for all ๐’ โˆˆ ๐‘ต,๐’ > 1 is called:
(a) โœ”A.P (b) G.P (c) H.P (d) None of these
9. ๐’๐’•๐’‰ term of an A.P is ๐Ÿ‘๐’ โˆ’ ๐Ÿ then 10th
term is :
(a) 9 (b) โœ” 29 (c) 12 (d) cannot determined
10. ๐’๐’•๐’‰ term of the series (
๐Ÿ
๐Ÿ‘
)
๐Ÿ
+ (
๐Ÿ“
๐Ÿ
)
๐Ÿ
+ (
๐Ÿ•
๐Ÿ‘
)
๐Ÿ
+ โ‹ฏ
(a) โœ” (
2๐‘›โˆ’1
3
)
2
(b) (
2๐‘›โˆ’1
3
)
2
(c) (
2๐‘›
3
)
2
(d) cannot determined
11. If ๐’‚๐’โˆ’๐Ÿ, ๐’‚๐’, ๐’‚๐’+๐Ÿ are in A.P, then ๐’‚๐’ is
(a) โœ”A.M (b) G.M (c) H.M (d) Mid point
12. Arithmetic mean between ๐’„ and ๐’… is:
(a) โœ”
๐‘+๐‘‘
2
(b)
๐‘+๐‘‘
2๐‘๐‘‘
(c)
2๐‘๐‘‘
๐‘+๐‘‘
(d)
2
๐‘+๐‘‘
13. The arithmetic mean between โˆš๐Ÿ and ๐Ÿ‘โˆš๐Ÿ is:
(a) 4โˆš2 (b) โœ”
4
โˆš2
(c) โˆš2 (d) none of these
14. The sum of terms of a sequence is called:
(a) Partial sum (b) โœ” Series (c) Finite sum (d) none of these
15. Forth partial sum of the sequence {๐’๐Ÿ
} is called:
(a) 16 (b) โœ” 1+4+9+16 (c) 8 (d) 1+2+3+4
16. Sum of ๐’ โˆ’term of an Arithmetic series ๐‘บ๐’ is equal to:
(a) โœ”
๐‘›
2
[2๐‘Ž + (๐‘› โˆ’ 1)๐‘‘] (b)
๐‘›
2
[๐‘Ž + (๐‘› โˆ’ 1)๐‘‘] (c)
๐‘›
2
[2๐‘Ž + (๐‘› + 1)๐‘‘] (d)
๐‘›
2
[2๐‘Ž + ๐‘™]
17. For any ๐‘ฎ. ๐‘ท., the common ratio ๐’“ is equal to:
(a)
๐‘Ž๐‘›
๐‘Ž๐‘›+1
(b)
๐‘Ž๐‘›โˆ’1
๐‘Ž๐‘›
(c) โœ”
๐‘Ž๐‘›
๐‘Ž๐‘›โˆ’1
(d) ๐‘Ž๐‘›+1 โˆ’ ๐‘Ž๐‘›, ๐‘› โˆˆ ๐‘, ๐‘› > 1
11 | P a g e
18. No term of a ๐‘ฎ. ๐‘ท., is:
(a) โœ”0 (b) 1 (c) negative (d) imaginary number
19. The general term of a ๐‘ฎ. ๐‘ท., is :
(a) โœ”๐‘Ž๐‘› = ๐‘Ž๐‘Ÿ๐‘›โˆ’1
(b) ๐‘Ž๐‘› = ๐‘Ž๐‘Ÿ๐‘›
(c) ๐‘Ž๐‘› = ๐‘Ž๐‘Ÿ๐‘›+1
(d) None of these
20. The sum of infinite geometric series is valid if
(a) |๐‘Ÿ| > 1 (b) |๐‘Ÿ| = 1 (c) |๐‘Ÿ| โ‰ฅ 1 (d) โœ” |๐‘Ÿ| < 1
21. For the series ๐Ÿ + ๐Ÿ“ + ๐Ÿ๐Ÿ“ + ๐Ÿ๐Ÿ๐Ÿ“ + โ‹ฏ + โˆž , the sum is
(a) -4 (b) 4 (c)
1โˆ’5๐‘›
โˆ’4
(d) โœ” not defined
22. An infinite geometric series is convergent if
(a) |๐‘Ÿ| > 1 (b) |๐‘Ÿ| = 1 (c) |๐‘Ÿ| โ‰ฅ 1 (d) โœ” |๐‘Ÿ| < 1
23. An infinite geometric series is divergent if
(a) |๐‘Ÿ| < 1 (b) |๐‘Ÿ| โ‰  1 (c) ๐‘Ÿ = 0 (dโœ”) |๐‘Ÿ| > 1
24. If sum of series is defined then it is called:
(a) โœ”Convergent series (b) Divergent series (c) finite series (d) Geometric series
25. If sum of series is not defined then it is called:
(a) Convergent series (b) โœ” Divergent series (c) finite series (d) Geometric series
26. The interval in which series ๐Ÿ + ๐Ÿ๐’™ + ๐Ÿ’๐’™๐Ÿ
+ ๐Ÿ–๐’™๐Ÿ‘
+ โ‹ฏ is convergent if :
(a) โˆ’2 < ๐‘ฅ < 2 (b) โœ” โˆ’
1
2
< ๐‘ฅ <
1
2
(c) |2๐‘ฅ| > 1 (d) |๐‘ฅ| < 1
27. If the reciprocal of the terms a sequence form an ๐‘จ. ๐‘ท., then it is called:
(a) โœ”๐ป. ๐‘ƒ (b) ๐บ. ๐‘ƒ (c) ๐ด. ๐‘ƒ (d) sequence
28. The ๐’๐’•๐’‰ term of
๐Ÿ
๐Ÿ
,
๐Ÿ
๐Ÿ“
,
๐Ÿ
๐Ÿ–
, โ€ฆ is
(a) โœ”
1
3๐‘›โˆ’1
(b) 3๐‘› โˆ’ 1 (c) 2๐‘› + 1 (d)
1
3๐‘›+1
29. Harmonic mean between ๐Ÿ and ๐Ÿ– is:
(a) โœ”5 (b)
16
5
(c) ยฑ4 (d)
5
16
30. If ๐‘จ, ๐‘ฎ and ๐‘ฏ are Arithmetic , Geometric and Harmonic means between two positive numbers
then
(a) ๐บ2
= ๐ด๐ป (b) โœ”๐ด, ๐บ, ๐ป ๐‘Ž๐‘Ÿ๐‘’ ๐‘–๐‘› ๐บ. ๐‘ƒ (c) ๐ด > ๐บ > ๐ป (d) all of these
31. If ๐‘จ, ๐‘ฎ and ๐‘ฏ are Arithmetic , Geometric and Harmonic means between two negative numbers
then
(a) ๐บ2
= ๐ด๐ป (b) ๐ด, ๐บ, ๐ป ๐‘Ž๐‘Ÿ๐‘’ ๐‘–๐‘› ๐บ. ๐‘ƒ (c) ๐ด < ๐บ < ๐ป (d) โœ”all of these
32. If ๐’‚ and ๐’ƒ are two positive number then
(a) ๐ด < ๐บ < ๐ป (b) โœ” ๐ด > ๐บ > ๐ป (c) ๐ด = ๐บ = ๐ป (d) ๐ด โ‰ฅ ๐บ โ‰ฅ ๐ป
33. If ๐’‚ and ๐’ƒ are two negative number then
(a) โœ”๐ด < ๐บ < ๐ป (b) ๐ด > ๐บ > ๐ป (c) ๐ด = ๐บ = ๐ป (d) ๐ด โ‰ฅ ๐บ โ‰ฅ ๐ป
34. If
๐’‚๐’+๐Ÿ+๐’ƒ๐’+๐Ÿ
๐’‚๐’+๐’ƒ๐’ is A.M between ๐’‚ and ๐’ƒ then ๐’ is equal to:
(a) โœ”0 (b) -1 (c) 1 (d)
1
2
35. If
๐’‚๐’+๐’ƒ๐’
๐’‚๐’โˆ’๐Ÿ+๐’ƒ๐’โˆ’๐Ÿ is G.M between ๐’‚ and ๐’ƒ then ๐’ is equal to:
(a) 0 (b) -1 (c) 1 (d) โœ”
1
2
36. If
๐’‚๐’+๐Ÿ+๐’ƒ๐’+๐Ÿ
๐’‚๐’+๐’ƒ๐’ is H.M between ๐’‚ and ๐’ƒ then ๐’ is equal to:
(a) 0 (b) โœ” -1 (c) 1 (d)
1
2
37. If ๐‘บ๐’ = (๐’ + ๐Ÿ)๐Ÿ
, then ๐‘บ๐Ÿ๐’ is equal to:
(a) 2๐‘› + 1 (b) โœ” 4๐‘›2
+ 4๐‘› + 1 (c) (2๐‘› โˆ’ 1)2
(d) cannot be determined
38. โˆ‘ ๐’๐Ÿ‘
=
(a)
๐‘›(๐‘›+1)
2
(b)
๐‘›(๐‘›+1)(๐‘›+2)
6
(c) โœ”
๐‘›2(๐‘›+1)2
4
(d)
๐‘›(๐‘›+1)2
2
UNIT # 07 Permutation, Combination and
Probability
Each question has four possible answer. Tick the correct answer.
1. ๐Ÿ๐ŸŽ๐‘ท๐Ÿ‘=
(a) 6890 (b) 6810 (c) โœ”6840 (d) 6880
2. If ๐’๐‘ท๐Ÿ=30 then ๐’ =
(a) 4 (b) 5 (c) โœ”6 (d) 10
12 | P a g e
3. The number of diagonals in 10-sided figure is
(a) 10 (b) 10๐ถ2
(c) โœ”10๐ถ2
โˆ’ 10 (d) 45
4. ๐’๐‘ช๐ŸŽ=
(a) 0 (b) โœ” 1 (c) ๐‘› (d) ๐‘›!
5. How many arrangement of the word โ€œMATHEMATICSโ€ can be made
(a) 11! (b) (
11
3,2,1,1,1,1,1
) (c) โœ” (
11
2,2,2,1,1,1,1,1
) (d) None
6. If ๐’ is negative then ๐’! is
(a) 1 (b) 0 (c) unique (d) โœ”Not defined
7. ๐’ โˆ’ ๐Ÿ๐‘ช๐’“
+ ๐’ โˆ’ ๐Ÿ๐‘ช๐’“โˆ’๐Ÿ
equals
(a) โœ”๐‘›๐ถ๐‘Ÿ
(b) ๐‘›๐ถ๐‘Ÿโˆ’1
(c) ๐‘› โˆ’ 1๐ถ๐‘Ÿ
(d) ๐‘› + 1๐ถ๐‘Ÿ
8. How many signals can be given by 5 flags of different colors , using 3 at a time
(a) 120 (b) โœ”60 (c) 24 (d) 15
9. ๐’๐‘ท๐’=
(a) 1 (b) ๐‘› (c) โœ” ๐‘›! (d) None
10.
๐Ÿ–!
๐Ÿ•!
=
(a) โœ”8 (b) 7 (c) 56 (d)
8
7
11. If an event ๐‘จ can occur in ๐’‘ ways and ๐‘ฉ can occur ๐’’ ways , then number of ways that both
events occur is:
(a) ๐‘ + ๐‘ž (b) โœ” ๐‘. ๐‘ž (c) (๐‘๐‘ž)! (d) (๐‘ + ๐‘ž)!
12. If ( ๐’
๐Ÿ๐Ÿ
) = (๐’
๐Ÿ–
) then the value of ๐’,
(a) 15 (b) 16 (c) 18 (d) โœ”20
13. Probability of non-occurrence of an event E is equal to :
(a) โœ”1 โˆ’ ๐‘ƒ(๐ธ) (b) ๐‘ƒ(๐ธ) +
๐‘›(๐‘†)
๐‘›(๐ธ)
(c)
๐‘›(๐‘†)
๐‘›(๐ธ)
(d) 1 + ๐‘ƒ(๐ธ)
14. For independent events ๐‘ท(๐‘จ โˆฉ ๐‘ฉ) =
(a) ๐‘ƒ(๐ด) + ๐‘ƒ(๐ต) (b) ๐‘ƒ(๐ด) โˆ’ ๐‘ƒ(๐ต) (c) โœ” ๐‘ƒ(๐ด). ๐‘ƒ(๐ต) (d)
๐‘ƒ(๐ด)
๐‘ƒ(๐ต)
15. A card is drawn from a deck of 52 playing cards. The probability of card that it is an ace
card is:
(a)
2
13
(b)
4
13
(c) โœ”
1
13
(d)
17
13
16. Four persons wants to sit in a circular sofa, the total ways are:
(a) 24 (b) โœ”6 (c) 4 (d) None of these
17. Two teams ๐‘จ and ๐‘ฉ are playing a match, the probability that team ๐‘จ does not lose is:
(a)
1
3
(b)
2
3
(c) 1 (d) 0
18. Let ๐‘บ = {๐Ÿ, ๐Ÿ, ๐Ÿ‘, โ€ฆ , ๐Ÿ๐ŸŽ} the probability that a number is divided by 4 is :
(a)
2
5
(b) โœ”
1
5
(c)
1
10
(d)
1
2
19. A die is rolled , the probability of getting 3 or 5 is:
(a)
2
3
(b) โœ”
15
36
(c)
15
36
(d)
1
36
20. A coin is tossed 5 times , then ๐’(๐‘บ) is equal to:
(a) โœ”32 (b) 25 (c) 10 (d) 20
21. The number of ways for sitting 4 persons in a train on a straight sofa is:
(a) โœ”24 (b) 6 (c) 4 (d) None of these
22. Sample space for tossing a coin is:
(a) {๐ป} (b) {๐‘‡} (c) {๐ป, ๐ป} (d) โœ” {๐ป, ๐‘‡}
23. If ๐‘ท(๐‘ฌ) =
๐Ÿ•
๐Ÿ๐Ÿ
, ๐’(๐‘บ) = ๐Ÿ–๐Ÿ’๐ŸŽ๐ŸŽ , ๐’(๐‘ฌ) =
(a) 108 (b) โœ”4900 (c) 144 (d) 14400
24. In a permutation ๐’๐‘ท๐’“
๐’๐’“ ๐‘ท(๐’, ๐’“) , it is always true that
(a) โœ”๐‘› โ‰ฅ ๐‘Ÿ (b) ๐‘› < ๐‘Ÿ (c) ๐‘› โ‰ค ๐‘Ÿ (d) ๐‘› < 0, ๐‘Ÿ < 0
25. If an event always occurs , then it is called:
(a) Null Event (b) Possible Event (c) โœ”Certain event (d) Independent Event
26. If ๐‘ฌ is a certain event , then
(a) ๐‘ƒ(๐ธ) = 0 (b) โœ”๐‘ƒ(๐ธ) = 1 (c) 0 < ๐‘ƒ(๐ธ) < 1 (d) ๐‘ƒ(๐ธ) > 1
27. If ๐‘ฌ is an impossible event ,then
(a) โœ”๐‘ƒ(๐ธ) = 0 (b) ๐‘ƒ(๐ธ) = 1 (c) ๐‘ƒ(๐ธ) โ‰  0 (d) 0 < ๐‘ƒ(๐ธ) < 1
13 | P a g e
28. Non occurrence of an event E is denoted by:
(a) โˆผ ๐ธ (b) โœ” ๐ธ (c) ๐ธ๐‘
(d) All of these
UNIT # 08 Mathematical Induction and
Binomial Theorem
Each question has four possible answer. Tick the correct answer.
1. The statement ๐Ÿ’๐’
+ ๐Ÿ‘๐’
+ ๐Ÿ’ is true when :
(a) ๐‘› = 0 (b) ๐‘› = 1 (c) โœ” ๐‘› โ‰ฅ 2 (d) ๐‘› is any +iv integer
2. The number of terms in the expansion of (๐’‚ + ๐’ƒ)๐’
are:
(a) ๐‘› (b) โœ” ๐‘› + 1 (c) 2๐‘›
(d) 2๐‘›โˆ’1
3. Middle term/s in the expansion of (๐’‚ โˆ’ ๐Ÿ‘๐’™)๐Ÿ๐Ÿ’
is/are :
(a) ๐‘‡7 (b) โœ” ๐‘‡8 (c) ๐‘‡6&๐‘‡7 (d) ๐‘‡7&๐‘‡8
4. The coefficient of the last term in the expansion of (๐Ÿ โˆ’ ๐’™)๐Ÿ•
is :
(a) 1 (b) โœ” โˆ’1 (c) 7 (d) โˆ’7
5. (๐Ÿ๐’
๐ŸŽ
) + (๐Ÿ๐’
๐Ÿ
) + (๐Ÿ๐’
๐Ÿ
) + โ‹ฏ + (๐Ÿ๐’
๐Ÿ๐’
) is equal to:
(a) 2๐‘›
(b) โœ”22๐‘›
(c) 22๐‘›โˆ’1
(d) 22๐‘›+1
6. ๐Ÿ + ๐’™ + ๐’™๐Ÿ
+ ๐’™๐Ÿ‘
+ โ‹ฏ
(a) (1 + ๐‘ฅ)โˆ’1
(b) โœ”(1 โˆ’ ๐‘ฅ)โˆ’1
(c) (1 + ๐‘ฅ)โˆ’2
(d) (1 โˆ’ ๐‘ฅ)โˆ’2
7. The middle term in the expansion of (๐’‚ + ๐’ƒ)๐’
is (
๐’
๐Ÿ
+ ๐Ÿ) ; then ๐’ is
(a) Odd (b) โœ” even (c) prime (d) none of these
8. The number of terms in the expansion of (๐’‚ + ๐’ƒ)๐Ÿ๐ŸŽ
is:
(a) 18 (b) 20 (c) โœ” 21 (d) 19
9. The expansion (๐Ÿ โˆ’ ๐Ÿ’๐’™)โˆ’๐Ÿ
is valid if:
(a) โœ”|๐‘ฅ| <
1
4
(b) |๐‘ฅ| >
1
4
(c) โˆ’1 < ๐‘ฅ < 1 (d) |๐‘ฅ| < โˆ’1
10. The statement ๐Ÿ‘๐’
< ๐‘›! is true, when
(a) ๐‘› = 2 (b) ๐‘› = 4 (c) ๐‘› = 6 (d) โœ”๐‘› > 6
11. General term in the expansion of (๐’‚ + ๐’ƒ)๐’
is:
(a) (๐‘›+1
๐‘Ÿ
)๐‘Ž๐‘›โˆ’๐‘Ÿ
๐‘ฅ^๐‘Ÿ (b)โœ”( ๐‘›
๐‘Ÿโˆ’1
)๐‘Ž๐‘›โˆ’๐‘Ÿ
๐‘ฅ๐‘Ÿ
(c) ( ๐‘›
๐‘Ÿ+1
)๐‘Ž๐‘›โˆ’๐‘Ÿ
๐‘ฅ๐‘Ÿ
(d) (๐‘›
๐‘Ÿ
)๐‘Ž๐‘›โˆ’๐‘Ÿ
๐‘ฅ๐‘Ÿ
12. The method of induction was given by Francesco who lived from:
(a) โœ”1494-1575 (b) 1500-1575 (c) 1498-1575 (d) 1494-1570
UNIT # 09 Fundamentals of Trigonometry
Each question has four possible answer. Tick the correct answer.
1. Two rays with a common starting point form:
(a) Triangle (b) โœ” Angle (c) Radian (d) Minute
2. The common starting point of two rays is called:
(a) Origin (b) Initial Point (c) โœ” Vertex (d)All of these
3. If the rotation of the angle is counter clock wise, then angle is:
(a) Negative (b) โœ” Positive (c) Non-Negative (d) None of these
4. If the initial ray ๐‘ถ๐‘จ
โƒ—โƒ—โƒ—โƒ—โƒ—โƒ— rates in anti-clockwise direction in such a way that it coincides with itself,
the angle then formed is:
(a) 180ยฐ (b) 270ยฐ (c) 300ยฐ (d) โœ” 360ยฐ
5. One rotation in anti-clock wise direction is equal to:
(a) 180ยฐ (b) 270ยฐ (c) โœ”360ยฐ (d) 90ยฐ
6. Straight line angle is equal to
(a)
1
2
๐‘Ÿ๐‘œ๐‘ก๐‘Ž๐‘ก๐‘–๐‘œ๐‘› (b) ๐œ‹ radian (c) 180ยฐ (d) โœ” All of these
7. One right angle is equal to
(a)
๐œ‹
2
๐‘Ÿ๐‘Ž๐‘‘๐‘–๐‘Ž๐‘› (b) 90ยฐ (c)
1
4
๐‘Ÿ๐‘œ๐‘ก๐‘Ž๐‘ก๐‘–๐‘œ๐‘› (d) โœ” All of these
8. ๐Ÿยฐ is equal to
(a) 30 minutes (b) โœ” 60 minutes (c)
1
60
minutes (d)
1
2
minutes
9. ๐Ÿยฐ is equal to
(a) 360โ€ฒโ€ฒ (b) โœ” 3600โ€ฒโ€ฒ (c) (
1
360
)โ€ฒ (d) 60โ€ฒโ€ฒ
14 | P a g e
10. 60th
part of ๐Ÿยฐ is equal to
(a) One second (b) โœ” One minute (c) 1 Radian (d) ๐œ‹ radian
11. 60th
part of ๐Ÿโ€ฒ is equal to
(a) 1โ€™ (b) โœ” 1โ€™โ€™ (c) 60โ€™โ€™ (d) 3600โ€™โ€™
12. 3600th
part of ๐Ÿยฐ is equal to
(a) 1โ€™ (b) โœ” 1โ€™โ€™ (c) 60โ€™โ€™ (d) 3600โ€™โ€™
13. Sexagesimal system is also called
(a) German System (b) โœ” English System (c) C.G.S System (d) SI System
14. ๐Ÿ๐Ÿ”ยฐ๐Ÿ‘๐ŸŽโ€ฒ equal to
(a) โœ”16.5ยฐ (b)
32ยฐ
2
(c) 16.05ยฐ (d) 16.2ยฐ
15. Conversion of ๐Ÿ๐Ÿ. ๐Ÿ๐Ÿ“๐Ÿ”ยฐ to ๐‘ซยฐ๐‘ดโ€ฒ
๐‘บโ€ฒ
โ€ฒ form is:
(a) 21ยฐ25โ€™6โ€™โ€™ (b) 21ยฐ40โ€ฒ27โ€ฒโ€ฒ (c) โœ” 21ยฐ15โ€ฒ22โ€ฒโ€ฒ (d) 21ยฐ30โ€ฒ2โ€ฒโ€ฒ
16. The angle subtended at the center of a circle by an arc whose length is equal to the radius of
the circle is called:
(a) 1 Degree (b) 1โ€™ (c) โœ” 1 Radian (d) 1โ€™โ€™
17. The system of angular measurement in which the angle is measured in radian is called:
(a) Sexagesimal System (b) โœ” Circular System (c) English System (d) Gradient System
18. Relation between the length of arc of a circle and the circular measure of it central angle is:
(a) ๐‘™ =
๐‘Ÿ
๐œƒ
(b) ๐œƒ = ๐‘™๐‘Ÿ (c) โœ” ๐œƒ =
๐‘™
๐‘Ÿ
(d) ๐‘™ =
1
2
๐‘Ÿ2
๐œƒ
19. With usual notation , if ๐’ = ๐Ÿ”๐’„๐’Ž, ๐’“ = ๐Ÿ๐’„๐’Ž, then unit of ๐œฝ is:
(a) ๐‘๐‘š (b) ๐‘๐‘š2
(c) โœ” No unit (d) ๐‘๐‘š3
20. ๐Ÿยฐ is equal to:
(a) (
๐œ‹
180
) ยฐ (b)
180
๐œ‹
๐‘Ÿ๐‘Ž๐‘‘ (c) (
180
๐œ‹
) ๐‘Ÿ๐‘Ž๐‘‘ (d) โœ”
๐œ‹
180
๐‘Ÿ๐‘Ž๐‘‘
21. ๐Ÿยฐ is equal to:
(a) 0.175 ๐‘Ÿ๐‘Ž๐‘‘ (b) โœ”0.0175๐‘Ÿ๐‘Ž๐‘‘ (c) 1.75 ๐‘Ÿ๐‘Ž๐‘‘ (d) 0.00175๐‘Ÿ๐‘Ž๐‘‘
22. 1 radian is equal to
(a) (
๐œ‹
180
) ยฐ (b)
180
๐œ‹
๐‘Ÿ๐‘Ž๐‘‘ (c) โœ” (
180
๐œ‹
)
ยฐ
(d)
๐œ‹
180
๐‘Ÿ๐‘Ž๐‘‘
23. 1 radian is equal to:
(a) โœ”57.296ยฐ (b) 5.7296ยฐ (c) 175.27ยฐ (d) 17.5276
24. 3 radian is:
(a) โœ”171.888ยฐ (b) 120ยฐ (c) 300ยฐ (d) 270ยฐ
25. ๐Ÿ๐ŸŽ๐Ÿ“ยฐ = __________๐’“๐’‚๐’…๐’Š๐’‚๐’
(a) โœ”
7๐œ‹
12
(b)
2๐œ‹
3
(c)
5๐œ‹
12
(d)
5๐œ‹
6
26. 3โ€™โ€™= ___________ ๐’“๐’‚๐’…๐’Š๐’‚๐’
(a)
53๐œ‹
270
(b) โœ”
๐œ‹
216000
(c)
41๐œ‹
720
(d)
27721๐œ‹
32400
27.
๐…
๐Ÿ’
๐’“๐’‚๐’…๐’Š๐’‚๐’ = ________๐’…๐’†๐’ˆ๐’“๐’†๐’†
(a) โœ”45ยฐ (b) 30ยฐ (c) 60ยฐ (d) 75ยฐ
28. Circular measure of angle between the hands of a watch at ๐Ÿ’โ€ฒ
๐‘ถ ๐’„๐’๐’๐’„๐’Œ is
(a) 45ยฐ (b) โœ” 120ยฐ (c)
3๐œ‹
2
(d) 270ยฐ
29. If ๐’ = ๐Ÿ. ๐Ÿ“ ๐’„๐’Ž & ๐’“ = ๐Ÿ. ๐Ÿ“ ๐’„๐’Ž then ๐œฝ is equal to:
(a) โœ”
3
5
(b)
5
3
(c) 3.75 (d) ๐‘๐‘œ๐‘›๐‘’
30. If ๐œฝ = ๐Ÿ’๐Ÿ“ยฐ , ๐’“ = ๐Ÿ๐Ÿ–๐’Ž๐’Ž , then ๐’ =
(a) โœ”
9
2
๐œ‹ (b)
2
9
๐œ‹ (c) 812mm (d) 810mm
31. Area of sector of circle of radius ๐’“ is:
(a) โœ”
1
2
๐‘Ÿ2
๐œƒ (b)
1
2
๐‘Ÿ๐œƒ2
(c)
1
2
(๐‘Ÿ๐œƒ)2
(d)
1
2๐‘Ÿ2๐œƒ
32. Angles with same initial and terminal sides are called:
(a) Acute angles (b) Allied Angles (c) โœ”Coterminal angles(d) Quadrentel angles
33. If angle ๐œฝ is in degree, then the angle coterminal with ๐œฝ is:
(a) ๐œƒ + 180ยฐ๐‘˜, ๐‘˜ โˆˆ ๐‘ (b) โœ” ๐œƒ + 360ยฐ๐‘˜, ๐‘˜ โˆˆ ๐‘ (c) ๐œƒ + 90ยฐ๐‘˜, ๐‘˜ โˆˆ ๐‘ (d) ๐œƒ + 60ยฐ๐‘˜, ๐‘˜ โˆˆ ๐‘
34. If angle ๐œฝ is in degree, then the angle coterminal with ๐œฝ is:
(a) โœ”๐œƒ + 2๐œ‹๐‘˜, ๐‘˜ โˆˆ ๐‘ (b) ๐œƒ + ๐œ‹๐‘˜, ๐‘˜ โˆˆ ๐‘ (c) ๐œƒ +
๐œ‹
2
๐‘˜, ๐‘˜ โˆˆ ๐‘ (d) ๐œƒ +
๐œ‹
3
๐‘˜, ๐‘˜ โˆˆ ๐‘
35. An angle is in standard position , if its vertex is
(a) โœ”At origin (b) at ๐‘ฅ โˆ’ ๐‘Ž๐‘ฅ๐‘–๐‘  (c) ๐‘Ž๐‘ก๐‘ฆ โˆ’ ๐‘Ž๐‘ฅ๐‘–๐‘  (d) in 1st
Quad Only
15 | P a g e
36. If initial and the terminal side of an angle falls on ๐’™ โˆ’ ๐’‚๐’™๐’Š๐’” ๐’๐’“ ๐’š โˆ’ ๐’‚๐’™๐’Š๐’” then it is called:
(a) Coterminal angle (b) โœ” Quadrantal angl (c) Allied angle (d) None of these
37. 0ยฐ, 90ยฐ, 180ยฐ, 270ยฐ and 360ยฐ are called
(a) Coterminal angle (b) โœ” Quadrantal angl (c) Allied angle (d) None of these
38. ๐’”๐’Š๐’๐Ÿ
๐œฝ + ๐’„๐’๐’”๐Ÿ
๐œฝ is equal to:
(a) 0 (b) -1 (c) 2 (d) โœ” 1
39. ๐Ÿ + ๐’•๐’‚๐’๐Ÿ
๐œฝ is equal to:
(a) csc2
๐œƒ (b) sin2
๐œƒ (c) โœ”sec2
๐œƒ (d) tan2
๐œƒ
40. ๐œ๐ฌ๐œ๐Ÿ
๐œฝ โˆ’ ๐œ๐จ๐ญ๐Ÿ
๐œฝ is equal to:
(a) 0 (b) โœ” 1 (c) -1 (d) 2
41. If ๐’”๐’Š๐’๐œฝ < 0 and ๐’„๐’๐’”๐œฝ > 0 then the terminal arm of angle lies in โ€ฆโ€ฆโ€ฆโ€ฆ.. Quad.
(a) I (b) II (c) III (d) โœ” IV
42. If ๐’„๐’๐’•๐œฝ > 0 and ๐’„๐’๐’”๐’†๐’„๐œฝ > 0 then the terminal arm of angle lies in โ€ฆโ€ฆโ€ฆโ€ฆ.. Quad.
(a) โœ”I (b) II (c) III (d) IV
43. If ๐’•๐’‚๐’๐œฝ < 0 and ๐’„๐’๐’”๐’†๐’„๐œฝ > 0 then the terminal arm of angle lies in โ€ฆโ€ฆโ€ฆโ€ฆ.. Quad.
(a) I (b) โœ” II (c) III (d) IV
44. If ๐’”๐’†๐’„๐œฝ < 0 and ๐’”๐’Š๐’๐œฝ < 0 then the terminal arm of angle lies in โ€ฆโ€ฆโ€ฆโ€ฆ.. Quad.
(a) I (b) II (c) โœ” III (d) IV
45. In right angle triangle, the measure of the side opposite to ๐Ÿ‘๐ŸŽยฐ is:
(a) โœ”Half of Hypotenuse (b) Half of Base (c) Double of base (d) None of these
46. The point (๐ŸŽ, ๐Ÿ) lies on the terminal side of angle:
(a) 0ยฐ (b) โœ” 90ยฐ (c) 180ยฐ (d) 270ยฐ
47. The point (โˆ’๐Ÿ, ๐ŸŽ) lies on the terminal side of angle:
(a) 0ยฐ (b) 90ยฐ (c) โœ” 180ยฐ (d) 270ยฐ
48. The point (๐ŸŽ, โˆ’๐Ÿ) lies on the terminal side of angle:
(a) 0ยฐ (b) 90ยฐ (c) 180ยฐ (d) โœ” 270ยฐ
49. ๐Ÿ๐‘บ๐’Š๐’๐Ÿ’๐Ÿ“ยฐ +
๐Ÿ
๐Ÿ
๐‘ช๐’๐’”๐’†๐’„๐Ÿ’๐Ÿ“ยฐ =
(a) โˆš
2
3
(b) โœ”
3
โˆš2
(c) โˆ’1 (d) 1
50. Domain of ๐’”๐’Š๐’๐œฝ is:
(a) โœ”๐‘… (b) ๐œƒ โˆˆ ๐‘… ๐‘๐‘ข๐‘ก ๐œƒ โ‰  ๐‘›๐œ‹, ๐‘› โˆˆ ๐‘ (c) ๐œƒ โˆˆ ๐‘… ๐‘๐‘ข๐‘ก ๐œƒ โ‰ 
(2๐‘›+1)๐œ‹
2
, ๐‘› โˆˆ ๐‘ (d) None of these
51. Domain of ๐’„๐’๐’”๐œฝ is:
(a) โœ”๐‘… (b) ๐œƒ โˆˆ ๐‘… ๐‘๐‘ข๐‘ก ๐œƒ โ‰  ๐‘›๐œ‹, ๐‘› โˆˆ ๐‘ (c) ๐œƒ โˆˆ ๐‘… ๐‘๐‘ข๐‘ก ๐œƒ โ‰ 
(2๐‘›+1)๐œ‹
2
, ๐‘› โˆˆ ๐‘ (d) None of these
52. Domain of ๐’•๐’‚๐’๐œฝ is:
(a) ๐‘… (b) ๐œƒ โˆˆ ๐‘… ๐‘๐‘ข๐‘ก ๐œƒ โ‰  ๐‘›๐œ‹, ๐‘› โˆˆ ๐‘ (c) โœ” ๐œƒ โˆˆ ๐‘… ๐‘๐‘ข๐‘ก ๐œƒ โ‰ 
(2๐‘›+1)๐œ‹
2
, ๐‘› โˆˆ ๐‘ (d) None of these
53. Domain of ๐’”๐’†๐’„๐œฝ is:
(a) ๐‘… (b) ๐œƒ โˆˆ ๐‘… ๐‘๐‘ข๐‘ก ๐œƒ โ‰  ๐‘›๐œ‹, ๐‘› โˆˆ ๐‘ (c) โœ” ๐œƒ โˆˆ ๐‘… ๐‘๐‘ข๐‘ก ๐œƒ โ‰ 
(2๐‘›+1)๐œ‹
2
, ๐‘› โˆˆ ๐‘ (d) None of these
54. ๐’„๐’๐’”๐’†๐’„๐œฝ๐’”๐’†๐’„๐œฝ๐’”๐’Š๐’๐œฝ๐’„๐’๐’”๐œฝ =
(a) โœ”1 (b) 0 (c) ๐‘ ๐‘–๐‘›๐œƒ (d) ๐‘๐‘œ๐‘ ๐œƒ
55. (๐’”๐’†๐’„๐œฝ + ๐’•๐’‚๐’๐œฝ)(๐’”๐’†๐’„๐œฝ โˆ’ ๐’•๐’‚๐’๐œฝ) =
(a) โœ”1 (b) 0 ๐‘ ๐‘’๐‘๐œƒ (d) ๐‘ก๐‘Ž๐‘›๐œƒ
56.
๐Ÿโˆ’๐’”๐’Š๐’๐œฝ
๐’„๐’๐’”๐œฝ
=
(a)
๐‘๐‘œ๐‘ 
1โˆ’๐‘ ๐‘–๐‘›๐œƒ
(b) โœ”
๐‘๐‘œ๐‘ ๐œƒ
1+๐‘ ๐‘–๐‘›๐œƒ
(c)
๐‘ ๐‘–๐‘›๐œƒ
1โˆ’๐‘๐‘œ๐‘ ๐œƒ
(d)
๐‘ ๐‘–๐‘›๐œƒ
1+๐‘๐‘œ๐‘ ๐œƒ
UNIT # 10 Trigonometric Identities
Each question has four possible answer. Tick the correct answer.
1. Distance between the points ๐‘จ(๐Ÿ‘, ๐Ÿ–) & ๐ต(5,6) is:
(a) โœ”2โˆš2 (b) 3 (c) 4 (d) โˆš2
2. Fundamental law of trigonometry is , ๐’„๐’๐’”(๐œถ โˆ’ ๐œท)
(a) โœ”๐‘๐‘œ๐‘ ๐›ผ๐‘๐‘œ๐‘ ๐›ฝ + ๐‘ ๐‘–๐‘›๐›ผ๐‘ ๐‘–๐‘›๐›ฝ (b) ๐‘๐‘œ๐‘ ๐›ผ๐‘๐‘œ๐‘ ๐›ฝ โˆ’ ๐‘ ๐‘–๐‘›๐›ผ๐‘ ๐‘–๐‘›๐›ฝ
(c) ๐‘ ๐‘–๐‘›๐›ผ๐‘๐‘œ๐‘ ๐›ฝ + ๐‘๐‘œ๐‘ ๐›ผ๐‘ ๐‘–๐‘›๐›ฝ (d) ๐‘ ๐‘–๐‘›๐›ผ๐‘๐‘œ๐‘ ๐›ฝ โˆ’ ๐‘๐‘œ๐‘ ๐›ผ๐‘ ๐‘–๐‘›๐›ฝ
3. ๐’„๐’๐’”(๐œถ + ๐œท) is equal to:
(a) ๐‘๐‘œ๐‘ ๐›ผ๐‘๐‘œ๐‘ ๐›ฝ + ๐‘ ๐‘–๐‘›๐›ผ๐‘ ๐‘–๐‘›๐›ฝ (b) โœ” ๐‘๐‘œ๐‘ ๐›ผ๐‘๐‘œ๐‘ ๐›ฝ โˆ’ ๐‘ ๐‘–๐‘›๐›ผ๐‘ ๐‘–๐‘›๐›ฝ
(c) ๐‘ ๐‘–๐‘›๐›ผ๐‘๐‘œ๐‘ ๐›ฝ + ๐‘๐‘œ๐‘ ๐›ผ๐‘ ๐‘–๐‘›๐›ฝ (d) ๐‘ ๐‘–๐‘›๐›ผ๐‘๐‘œ๐‘ ๐›ฝ โˆ’ ๐‘๐‘œ๐‘ ๐›ผ๐‘ ๐‘–๐‘›๐›ฝ
16 | P a g e
4. ๐’”๐’Š๐’(๐œถ + ๐œท) is equal to:
(a) ๐‘๐‘œ๐‘ ๐›ผ๐‘๐‘œ๐‘ ๐›ฝ + ๐‘ ๐‘–๐‘›๐›ผ๐‘ ๐‘–๐‘›๐›ฝ (b) ๐‘๐‘œ๐‘ ๐›ผ๐‘๐‘œ๐‘ ๐›ฝ โˆ’ ๐‘ ๐‘–๐‘›๐›ผ๐‘ ๐‘–๐‘›๐›ฝ
(c) โœ” ๐‘ ๐‘–๐‘›๐›ผ๐‘๐‘œ๐‘ ๐›ฝ + ๐‘๐‘œ๐‘ ๐›ผ๐‘ ๐‘–๐‘›๐›ฝ (d) ๐‘ ๐‘–๐‘›๐›ผ๐‘๐‘œ๐‘ ๐›ฝ โˆ’ ๐‘๐‘œ๐‘ ๐›ผ๐‘ ๐‘–๐‘›๐›ฝ
5. ๐’”๐’Š๐’(๐œถ โˆ’ ๐œท) is equal to:
(a) ๐‘๐‘œ๐‘ ๐›ผ๐‘๐‘œ๐‘ ๐›ฝ + ๐‘ ๐‘–๐‘›๐›ผ๐‘ ๐‘–๐‘›๐›ฝ (b) ๐‘๐‘œ๐‘ ๐›ผ๐‘๐‘œ๐‘ ๐›ฝ โˆ’ ๐‘ ๐‘–๐‘›๐›ผ๐‘ ๐‘–๐‘›๐›ฝ
(c) ๐‘ ๐‘–๐‘›๐›ผ๐‘๐‘œ๐‘ ๐›ฝ + ๐‘๐‘œ๐‘ ๐›ผ๐‘ ๐‘–๐‘›๐›ฝ (d) โœ” ๐‘ ๐‘–๐‘›๐›ผ๐‘๐‘œ๐‘ ๐›ฝ โˆ’ ๐‘๐‘œ๐‘ ๐›ผ๐‘ ๐‘–๐‘›๐›ฝ
6. ๐’„๐’๐’” (
๐…
๐Ÿ
โˆ’ ๐œท) =
(a) ๐‘๐‘œ๐‘ ๐›ฝ (b) โ€“ ๐‘๐‘œ๐‘ ๐›ฝ (c) โœ” ๐‘ ๐‘–๐‘›๐›ฝ (d) โ€“ ๐‘ ๐‘–๐‘›๐›ฝ
7. ๐’„๐’๐’” (๐œท +
๐…
๐Ÿ
) =
(a) ๐‘๐‘œ๐‘ ๐›ฝ (b) โ€“ ๐‘๐‘œ๐‘ ๐›ฝ (c) ๐‘ ๐‘–๐‘›๐›ฝ (d) โœ” โ€“ ๐‘ ๐‘–๐‘›๐›ฝ
8. ๐’”๐’Š๐’ (๐œท โˆ’
๐…
๐Ÿ
) =
(a) ๐‘๐‘œ๐‘ ๐›ฝ (b) โœ” โ€“ ๐‘๐‘œ๐‘ ๐›ฝ (c) ๐‘ ๐‘–๐‘›๐›ฝ (d) โœ” โ€“ ๐‘ ๐‘–๐‘›๐›ฝ
9. ๐’„๐’๐’”(๐Ÿ๐… โˆ’ ๐œฝ) =
(a) โœ”๐‘๐‘œ๐‘ ๐œƒ (b) โ€“ ๐‘๐‘œ๐‘ ๐œƒ (c) ๐‘ ๐‘–๐‘›๐œƒ (d) โ€“ ๐‘ ๐‘–๐‘›๐œƒ
10. ๐’”๐’Š๐’(๐Ÿ๐… โˆ’ ๐œฝ) =
(a) ๐‘๐‘œ๐‘ ๐œƒ (b) โ€“ ๐‘๐‘œ๐‘ ๐œƒ (c) ๐‘ ๐‘–๐‘›๐œƒ (d) โœ” โ€“ ๐‘ ๐‘–๐‘›๐œƒ
11. ๐’•๐’‚๐’(๐œถ + ๐œท) =
(a)
๐‘ก๐‘Ž๐‘›๐›ผโˆ’๐‘ก๐‘Ž๐‘›๐›ฝ
1+๐‘ก๐‘Ž๐‘›๐›ผ๐‘ก๐‘Ž๐‘›๐›ฝ
(b) โœ”
๐‘ก๐‘Ž๐‘›๐›ผ+๐‘ก๐‘Ž๐‘›๐›ฝ
1โˆ’๐‘ก๐‘Ž๐‘›๐›ผ๐‘ก๐‘Ž๐‘›๐›ฝ
(c)
๐‘ก๐‘Ž๐‘›๐›ผโˆ’๐‘ก๐‘Ž๐‘›๐›ฝ
1โˆ’๐‘ก๐‘Ž๐‘›๐›ผ๐‘ก๐‘Ž๐‘›๐›ฝ
(d)
๐‘ก๐‘Ž๐‘›๐›ผ+๐‘ก๐‘Ž๐‘›๐›ฝ
1โˆ’๐‘ก๐‘Ž๐‘›๐›ผ๐‘ก๐‘Ž๐‘›๐›ฝ
12. ๐’•๐’‚๐’(๐œถ โˆ’ ๐œท) =
(a) โœ”
๐‘ก๐‘Ž๐‘›๐›ผโˆ’๐‘ก๐‘Ž๐‘›๐›ฝ
1+๐‘ก๐‘Ž๐‘›๐›ผ๐‘ก๐‘Ž๐‘›๐›ฝ
(b)
๐‘ก๐‘Ž๐‘›๐›ผ+๐‘ก๐‘Ž๐‘›๐›ฝ
1โˆ’๐‘ก๐‘Ž๐‘›๐›ผ๐‘ก๐‘Ž๐‘›๐›ฝ
(c)
๐‘ก๐‘Ž๐‘›๐›ผโˆ’๐‘ก๐‘Ž๐‘›๐›ฝ
1โˆ’๐‘ก๐‘Ž๐‘›๐›ผ๐‘ก๐‘Ž๐‘›๐›ฝ
(d)
๐‘ก๐‘Ž๐‘›๐›ผ+๐‘ก๐‘Ž๐‘›๐›ฝ
1โˆ’๐‘ก๐‘Ž๐‘›๐›ผ๐‘ก๐‘Ž๐‘›๐›ฝ
13. Angles associated with basic angles of measure ๐œฝ to a right angle or its multiple are called:
(a) Coterminal angle (b) angle in standard position (c) โœ” Allied angle (d) obtuse angle
14. ๐’•๐’‚๐’ (
๐…
๐Ÿ
โˆ’ ๐œฝ) =
(a) โœ”๐‘๐‘œ๐‘ก๐œƒ (b) ๐‘ก๐‘Ž๐‘›๐œƒ (c) โ€“ ๐‘๐‘œ๐‘ก๐œƒ (d) โ€“ ๐‘ก๐‘Ž๐‘›๐œƒ
15. ๐’•๐’‚๐’ (
๐…
๐Ÿ
+ ๐œฝ) =
(a) ๐‘๐‘œ๐‘ก๐œƒ (b) ๐‘ก๐‘Ž๐‘›๐œƒ (c) โœ”โ€“ ๐‘๐‘œ๐‘ก๐œƒ (d) โ€“ ๐‘ก๐‘Ž๐‘›๐œƒ
16. ๐’”๐’Š๐’ (
๐Ÿ‘๐…
๐Ÿ
+ ๐œฝ) =
(a) ๐‘ ๐‘–๐‘›๐œƒ (b) ๐‘๐‘œ๐‘ ๐œƒ (c) โˆ’๐‘ ๐‘–๐‘›๐œƒ (d) โœ” โ€“ ๐‘๐‘œ๐‘ ๐œƒ
17. ๐’„๐’๐’” ๐Ÿ‘๐Ÿ๐Ÿ“ยฐ is equal to:
(a) 1 (b) 0 (c) โœ”
1
โˆš2
(d)
โˆš3
2
18. ๐’•๐’‚๐’(โˆ’๐Ÿ๐Ÿ‘๐Ÿ“ยฐ) is equal to:
(a) โœ” 1 (b) 0 (c)
1
โˆš3
(d) -1
19. ๐’”๐’†๐’„(โˆ’๐Ÿ‘๐ŸŽ๐ŸŽยฐ) =
(a) 1 (b) โœ” 0 (c) 2 (d) -1
20. ๐’”๐’Š๐’(๐Ÿ๐Ÿ–๐ŸŽยฐ + ๐œถ)๐’”๐’Š๐’(๐Ÿ—๐ŸŽยฐ โˆ’ ๐œถ) =
(a) โœ” ๐‘ ๐‘–๐‘›๐›ผ๐‘๐‘œ๐‘ ๐›ผ (b) โ€“ ๐‘ ๐‘–๐‘›๐›ผ๐‘๐‘œ๐‘ ๐›ผ (c) ๐‘๐‘œ๐‘ ๐›พ (d) โ€“ ๐‘๐‘œ๐‘ ๐›พ
21. If ๐œถ, ๐œท and ๐œธ are the angles of a triangle ABC then ๐’”๐’Š๐’(๐œถ + ๐œท) =
(a) โœ” ๐‘ ๐‘–๐‘›๐›พ (b) โ€“ ๐‘ ๐‘–๐‘›๐›พ (c) ๐‘๐‘œ๐‘ ๐›พ (d) โ€“ ๐‘๐‘œ๐‘ ๐›พ
22. If ๐œถ, ๐œท and ๐œธ are the angles of a triangle ABC then ๐’„๐’๐’” (
๐œถ+๐œท
๐Ÿ
) =
(a) โœ” sin
๐›พ
2
(b) โ€“ sin
๐›พ
2
(c) cos
๐›พ
2
(d) โ€“ cos
๐›พ
2
23. If ๐œถ, ๐œท and ๐œธ are the angles of a triangle ABC then ๐’„๐’๐’”(๐œถ + ๐œท) =
(a) ๐‘ ๐‘–๐‘›๐›พ (b) โ€“ ๐‘ ๐‘–๐‘›๐›พ (c) ๐‘๐‘œ๐‘ ๐›พ (d) โœ” โ€“ ๐‘๐‘œ๐‘ ๐›พ
24.
๐’„๐’๐’”๐Ÿ๐Ÿยฐ+๐’”๐’Š๐’๐Ÿ๐Ÿยฐ
๐’„๐’๐’”๐Ÿ๐Ÿยฐโˆ’๐’”๐’Š๐’๐Ÿ๐Ÿยฐ
=
(a) โœ” ๐‘ก๐‘Ž๐‘›56ยฐ (b) ๐‘ก๐‘Ž๐‘›34ยฐ (c) ๐‘๐‘œ๐‘ก56ยฐ (d) ๐‘๐‘œ๐‘ก34ยฐ
25. ๐’”๐’Š๐’๐Ÿ๐œถ is equal to:
(a) cos2
๐›ผ โˆ’ sin2
๐›ผ (b) 1 + ๐‘๐‘œ๐‘ 2๐›ผ (c) โœ” 2๐‘ ๐‘–๐‘›๐›ผ๐‘๐‘œ๐‘ ๐›ผ (d) 2๐‘ ๐‘–๐‘›2๐›ผ๐‘๐‘œ๐‘ 2๐›ผ
26. ๐’„๐’๐’”๐Ÿ๐œถ =
(a) cos2
๐›ผ โˆ’ sin2
๐›ผ (b) 1 โˆ’ 2 sin2
๐›ผ (c) 2 cos2
๐›ผ โˆ’ 1 (d) โœ” All of these
27. ๐’•๐’‚๐’๐Ÿ๐œถ =
(a)
2๐‘ก๐‘Ž๐‘›๐›ผ
1+๐‘ก๐‘Ž๐‘›2 ๐›ผ
(b) โœ”
2๐‘ก๐‘Ž๐‘›๐›ผ
1โˆ’tan2 ๐›ผ
(c)
2 tan2 ๐›ผ
1โˆ’tan2 ๐›ผ
(d)
tan2 ๐›ผ
1โˆ’tan2 ๐›ผ
28. ๐’”๐’Š๐’๐œถ + ๐’”๐’Š๐’๐œท is equal to:
(a) โœ” 2 sin (
๐›ผ+๐›ฝ
2
) cos (
๐›ผโˆ’๐›ฝ
2
) (b) 2 cos (
๐›ผ+๐›ฝ
2
) sin(
๐›ผโˆ’๐›ฝ
2
)
(c) โˆ’2 sin (
๐›ผ+๐›ฝ
2
) sin(
๐›ผโˆ’๐›ฝ
2
) (d) 2 cos(
๐›ผ+๐›ฝ
2
) cos (
๐›ผโˆ’๐›ฝ
2
)
17 | P a g e
29. ๐’”๐’Š๐’๐œถ โˆ’ ๐’”๐’Š๐’๐œท is equal to:
(b) 2 sin(
๐›ผ+๐›ฝ
2
) cos (
๐›ผโˆ’๐›ฝ
2
) (b) โœ” 2 cos (
๐›ผ+๐›ฝ
2
) sin (
๐›ผโˆ’๐›ฝ
2
)
(c) โˆ’2 sin (
๐›ผ+๐›ฝ
2
) sin(
๐›ผโˆ’๐›ฝ
2
) (d) 2 cos(
๐›ผ+๐›ฝ
2
) cos (
๐›ผโˆ’๐›ฝ
2
)
30. ๐’„๐’๐’”๐œถ + ๐’„๐’๐’”๐œท is equal to:
(c) 2 sin(
๐›ผ+๐›ฝ
2
) cos (
๐›ผโˆ’๐›ฝ
2
) (b) 2 cos (
๐›ผ+๐›ฝ
2
) sin(
๐›ผโˆ’๐›ฝ
2
)
(c) -2 sin(
๐›ผ+๐›ฝ
2
) sin (
๐›ผโˆ’๐›ฝ
2
) (d) โœ” 2 cos (
๐›ผ+๐›ฝ
2
)cos (
๐›ผโˆ’๐›ฝ
2
)
31. ๐’„๐’๐’”๐œถ โˆ’ ๐’„๐’๐’”๐œท is equal to:
(d) 2 sin(
๐›ผ+๐›ฝ
2
) cos (
๐›ผโˆ’๐›ฝ
2
) (b) 2 cos (
๐›ผ+๐›ฝ
2
) sin(
๐›ผโˆ’๐›ฝ
2
)
(c) โœ” โˆ’ 2 sin (
๐›ผ+๐›ฝ
2
) sin (
๐›ผโˆ’๐›ฝ
2
) (d) 2 cos(
๐›ผ+๐›ฝ
2
) cos (
๐›ผโˆ’๐›ฝ
2
)
32. Which is the allied angle
(a) โœ” 90ยฐ + ๐œƒ (b) 60ยฐ + ๐œƒ (c) 45ยฐ + ๐œƒ (d) 30ยฐ + ๐œƒ
33. ๐Ÿ๐’”๐’Š๐’๐Ÿ•๐œฝ๐’„๐’๐’”๐Ÿ‘๐œฝ =
(a) โœ” ๐‘ ๐‘–๐‘›10๐œƒ + ๐‘ ๐‘–๐‘›4๐œƒ (b) ๐‘ ๐‘–๐‘›5๐œƒ โˆ’ ๐‘ ๐‘–๐‘›2๐œƒ (c) ๐‘๐‘œ๐‘ 10๐œƒ + ๐‘๐‘œ๐‘ 4๐œƒ (d) ๐‘๐‘œ๐‘ 5๐œƒ โˆ’ ๐‘๐‘œ๐‘ 2๐œƒ
34. ๐Ÿ๐’„๐’๐’”๐Ÿ“๐œฝ๐’”๐’Š๐’๐Ÿ‘๐œฝ =
(b) โœ” ๐‘ ๐‘–๐‘›8๐œƒ โˆ’ ๐‘ ๐‘–๐‘›2๐œƒ (b) ๐‘ ๐‘–๐‘›8๐œƒ + ๐‘ ๐‘–๐‘›2๐œƒ (c) ๐‘๐‘œ๐‘ 8๐œƒ + ๐‘๐‘œ๐‘ 2๐œƒ (d) ๐‘๐‘œ๐‘ 8๐œƒ โˆ’ ๐‘๐‘œ๐‘ 2๐œƒ
UNIT # 11 Trigonometric Functions and
their Graphs
Each question has four possible answer. Tick the correct answer.
1. Domain of ๐’š = ๐’”๐’Š๐’๐’™ is
(a) โœ”โˆ’โˆž < ๐‘ฅ < โˆž (b) โˆ’1 โ‰ค ๐‘ฅ โ‰ค 1 (c) โˆ’โˆž < ๐‘ฅ < โˆž , ๐‘ฅ โ‰  ๐‘›๐œ‹ , ๐‘› โˆˆ ๐‘ (d) ๐‘ฅ โ‰ฅ 1, ๐‘ฅ โ‰ค โˆ’1
2. Domain of ๐’š = ๐’„๐’๐’”๐’™ is
(a) โœ”โˆ’โˆž < ๐‘ฅ < โˆž (b) โˆ’1 โ‰ค ๐‘ฅ โ‰ค 1 (c) โˆ’โˆž < ๐‘ฅ < โˆž , ๐‘ฅ โ‰  ๐‘›๐œ‹ , ๐‘› โˆˆ ๐‘ (d) ๐‘ฅ โ‰ฅ 1, ๐‘ฅ โ‰ค โˆ’1
3. Domain of ๐’š = ๐’•๐’‚๐’๐’™ is
(a) โˆ’โˆž < ๐‘ฅ < โˆž (b) โˆ’1 โ‰ค ๐‘ฅ โ‰ค 1 (c) โœ” โˆ’โˆž < ๐‘ฅ < โˆž , ๐‘ฅ โ‰ 
2๐‘›+1
2
๐œ‹ , ๐‘› โˆˆ ๐‘ (d) ๐‘ฅ โ‰ฅ 1, ๐‘ฅ โ‰ค โˆ’1
4. Domain of ๐’š = ๐’”๐’†๐’„๐’™ is
(a) โˆ’โˆž < ๐‘ฅ < โˆž (b) โˆ’1 โ‰ค ๐‘ฅ โ‰ค 1 (c) โœ” โˆ’โˆž < ๐‘ฅ < โˆž , ๐‘ฅ โ‰ 
2๐‘›+1
2
๐œ‹ , ๐‘› โˆˆ ๐‘ (d) ๐‘ฅ โ‰ฅ 1, ๐‘ฅ โ‰ค โˆ’1
5. Domain of ๐’š = ๐’„๐’”๐’„๐’™ is
(a) โˆ’โˆž < ๐‘ฅ < โˆž (b) โœ” โˆ’1 โ‰ค ๐‘ฅ โ‰ค 1 (c) โˆ’โˆž < ๐‘ฅ < โˆž , ๐‘ฅ โ‰ 
2๐‘›+1
2
๐œ‹ , ๐‘› โˆˆ ๐‘ (d) ๐‘ฅ โ‰ฅ 1, ๐‘ฅ โ‰ค โˆ’1
6. Domain of ๐’š = ๐’„๐’๐’•๐’™ is
(a) โˆ’โˆž < ๐‘ฅ < โˆž (b) โœ” โˆ’1 โ‰ค ๐‘ฅ โ‰ค 1 (c) โˆ’โˆž < ๐‘ฅ < โˆž , ๐‘ฅ โ‰ 
2๐‘›+1
2
๐œ‹ , ๐‘› โˆˆ ๐‘ (d) ๐‘ฅ โ‰ฅ 1, ๐‘ฅ โ‰ค โˆ’1
7. Range of ๐’š = ๐’”๐’Š๐’๐’™ is
(a) ๐‘… (b) โœ”โˆ’1 โ‰ค ๐‘ฆ โ‰ค 1 (c) (โˆ’โˆž, 1) โˆช (1, โˆž) (d) โˆ’1 < ๐‘ฆ < 1
8. Range of ๐’š = ๐’„๐’๐’”๐’™ is
(a) ๐‘… (b) โœ” โˆ’1 โ‰ค ๐‘ฆ โ‰ค 1 (c) (โˆ’โˆž, 1) โˆช (1, โˆž) (d) โˆ’1 < ๐‘ฆ < 1
9. Range of ๐’š = ๐’•๐’‚๐’๐’™ is
(a) โœ”๐‘… (b) โˆ’1 โ‰ค ๐‘ฆ โ‰ค 1 (c) ๐‘„ (d) ๐‘… โˆ’ {0}
10. Range of ๐’š = ๐’„๐’๐’•๐’™ is
(a) โœ”๐‘… (b) ๐‘… โˆ’ [โˆ’1,1] (c) ๐‘… โˆ’ {0} (d) ๐‘
11. Range of ๐’š = ๐’”๐’†๐’„๐’™ is
(a) ๐‘… (b) โœ” ๐‘ฆ โ‰ฅ 1๐‘œ๐‘Ÿ ๐‘ฆ โ‰ค โˆ’1 (c) โˆ’1 โ‰ค ๐‘ฆ โ‰ค 1 (d) ๐‘… โˆ’ [โˆ’1,1]
12. Range of ๐’š = ๐’„๐’๐’”๐’†๐’„๐’™ is
(a) ๐‘… (b) โœ” ๐‘ฆ โ‰ฅ 1๐‘œ๐‘Ÿ ๐‘ฆ โ‰ค โˆ’1 (c) โˆ’1 โ‰ค ๐‘ฆ โ‰ค 1 (d) ๐‘… โˆ’ [โˆ’1,1]
13. Smallest +๐’Š๐’—๐’† number which when added to the original circular measure of the angle
gives the same value of the function is called:
(a) Domain (b) Range (c) Co domain (d) โœ”Period
14. Period of ๐’”๐’Š๐’๐œฝ is
(a) ๐œ‹ (b) โœ” 2๐œ‹ (c) โˆ’2๐œ‹ (d)
๐œ‹
2
15. Period of ๐’„๐’๐’”๐’†๐’„๐œฝ is
(a) ๐œ‹ (b) โœ”2๐œ‹ (c) โˆ’2๐œ‹ (d)
๐œ‹
2
18 | P a g e
16. Period of ๐’„๐’๐’”๐œฝ is
(a) ๐œ‹ (b) โœ”2๐œ‹ (c) โˆ’2๐œ‹ (d)
๐œ‹
2
17. Period of ๐’•๐’‚๐’๐œฝ is
(a) โœ”๐œ‹ (b) 2๐œ‹ (c) โˆ’2๐œ‹ (d)
๐œ‹
2
18. Period of ๐’„๐’๐’•๐œฝ is
(a) โœ”๐œ‹ (b) 2๐œ‹ (c) โˆ’2๐œ‹ (d)
๐œ‹
2
19. Period of ๐’”๐’†๐’„๐œฝ is
(a) ๐œ‹ (b) โœ”2๐œ‹ (c) โˆ’2๐œ‹ (d)
๐œ‹
2
20. Period of ๐’•๐’‚๐’๐Ÿ’๐’™ is
(a) ๐œ‹ (b) 2๐œ‹ (c) โˆ’2๐œ‹ (d) โœ”
๐œ‹
4
21. Period of ๐’„๐’๐’•๐Ÿ‘๐’™ is
(a) ๐œ‹ (b) โœ”
๐œ‹
3
(c) โˆ’2๐œ‹ (d)
๐œ‹
4
22. Period of ๐Ÿ‘๐’„๐’๐’”
๐’™
๐Ÿ“
is
(a) 2๐œ‹ (b)
๐œ‹
2
(c) ๐œ‹ (d) โœ” 10๐œ‹
23. The graph of trigonometric functions have:
(a) Break segments (b) Sharp corners (c) Straight line segments (d) โœ” smooth curves
24. Curves of the trigonometric functions repeat after fixed intervals because trigonometric
functions are
(a) Simple (b) linear (c) quadratic (d) โœ” periodic
25. The graph of ๐’š = ๐’„๐’๐’”๐’™ lies between the horizontal line ๐’š = โˆ’๐Ÿ and
(a) โœ” +1 (b) 0 (c) 2 (d) -2
UNIT # 12 Application of Trigonometry
Each question has four possible answer. Tick the correct answer.
1. A โ€œTriangleโ€ has :
(a) Two elements (b) 3 elements (c) 4 elements (d) โœ” 6 elments
2. When we look an object above the horizontal ray, the angle formed is called angle of:
(a) โœ”Elevation (b) depression (c) incidence (d) reflects
3. When we look an object below the horizontal ray, the angle formed is called angle of:
(a) Elevation (b) โœ” depression (c) incidence (d) reflects
4. A triangle which is not right is called:
(a) โœ”Oblique triangle (b) Isosceles triangle (c) Scalene triangle (d) Right isosceles triangle
5. To solve an oblique triangle we use:
(a) โœ”Law of Sine (b) Law of Cosine (c) Law of Tangents (d) All of these
6. In any triangle ๐‘จ๐‘ฉ๐‘ช,
๐’ƒ๐Ÿ+๐’„๐Ÿโˆ’๐’‚๐Ÿ
๐Ÿ๐’ƒ๐’„
=
(a) โœ”๐‘๐‘œ๐‘ ๐›ผ (b) ๐‘ ๐‘–๐‘›๐›ผ (c) ๐‘๐‘œ๐‘ ๐›ฝ (d) ๐‘๐‘œ๐‘ ๐›พ
7. Which can be reduced to Pythagoras theorem,
(a) Law of sine (b) โœ” law of cosine (c) law of tangents (d) Half angle formulas
8. In any triangle ๐‘จ๐‘ฉ๐‘ช, if ๐œท = ๐Ÿ—๐ŸŽยฐ , then ๐’ƒ๐Ÿ
= ๐’‚๐Ÿ
+ ๐’„๐Ÿ
โˆ’ ๐Ÿ๐’‚๐’„๐’„๐’๐’”๐œท becomes:
(a) Law of sine (b) Law of tangents (c) โœ”Law of cosine (d) None of these
9. In any triangle ๐‘จ๐‘ฉ๐‘ช, law of tangent is :
(a)
๐‘Žโˆ’๐‘
๐‘Ž+๐‘
=
tan(๐›ผโˆ’๐›ฝ)
tan(๐›ผ+๐›ฝ)
(b)
๐‘Ž+๐‘
๐‘Žโˆ’๐‘
=
tan(๐›ผ+๐›ฝ)
tan(๐›ผโˆ’๐›ฝ)
(c) โœ”
๐‘Žโˆ’๐‘
๐‘Ž+๐‘
=
tan
๐›ผโˆ’๐›ฝ
2
tan
๐›ผ+๐›ฝ
2
(d)
๐‘Žโˆ’๐‘
๐‘Ž+๐‘
=
tan
๐›ผ+๐›ฝ
2
tan
๐›ผโˆ’๐›ฝ
2
10. In any triangle ๐‘จ๐‘ฉ๐‘ช, โˆš
(๐‘บโˆ’๐’‚)(๐’”โˆ’๐’ƒ)
๐’‚๐’ƒ
=
(a) sin
๐›ผ
2
(b) sin
๐›ฝ
2
(c) โœ” sin
๐›พ
2
(d) cos
๐›ผ
2
11. In any triangle ๐‘จ๐‘ฉ๐‘ช, โˆš
(๐‘บโˆ’๐’ƒ)(๐’”โˆ’๐’„)
๐’ƒ๐’„
=
(a) โœ” sin
๐›ผ
2
(b) sin
๐›ฝ
2
(c) sin
๐›พ
2
(d) cos
๐›ผ
2
12. In any triangle ๐‘จ๐‘ฉ๐‘ช, โˆš
(๐‘บโˆ’๐’‚)(๐’”โˆ’๐’„)
๐’‚๐’„
=
(a) sin
๐›ผ
2
(b) โœ” sin
๐›ฝ
2
(c) sin
๐›พ
2
(d) cos
๐›ผ
2
13. In any triangle ๐‘จ๐‘ฉ๐‘ช, ๐’„๐’๐’”
๐œถ
๐Ÿ
=
(a) โˆš
๐‘ (๐‘ โˆ’๐‘Ž)
๐‘Ž๐‘
(b) โˆš
๐‘ (๐‘ โˆ’๐‘)
๐‘Ž๐‘
(c) โœ” โˆš
๐‘ (๐‘ โˆ’๐‘Ž)
๐‘๐‘
(d) โˆš
๐‘ (๐‘ โˆ’๐‘)
๐‘Ž๐‘
19 | P a g e
14. In any triangle ๐‘จ๐‘ฉ๐‘ช, ๐’„๐’๐’”
๐œท
๐Ÿ
=
(a) โˆš
๐‘ (๐‘ โˆ’๐‘Ž)
๐‘Ž๐‘
(b) โœ”โˆš
๐‘ (๐‘ โˆ’๐‘)
๐‘Ž๐‘
(c) โˆš
๐‘ (๐‘ โˆ’๐‘Ž)
๐‘๐‘
(d) โˆš
๐‘ (๐‘ โˆ’๐‘)
๐‘Ž๐‘
15. In any triangle ๐‘จ๐‘ฉ๐‘ช, ๐’„๐’๐’”
๐œธ
๐Ÿ
=
(a) โˆš
๐‘ (๐‘ โˆ’๐‘Ž)
๐‘Ž๐‘
(b) โˆš
๐‘ (๐‘ โˆ’๐‘)
๐‘Ž๐‘
(c) โˆš
๐‘ (๐‘ โˆ’๐‘Ž)
๐‘๐‘
(d) โœ” โˆš
๐‘ (๐‘ โˆ’๐‘)
๐‘Ž๐‘
16. In any triangle ๐‘จ๐‘ฉ๐‘ช, with usual notations , ๐’” is equal to
(a) ๐‘Ž + ๐‘ + ๐‘ (b)
๐‘Ž+๐‘+๐‘
3
(c) โœ”
๐‘Ž+๐‘+๐‘
2
(d)
๐‘Ž๐‘๐‘
2
17. In any triangle ๐‘จ๐‘ฉ๐‘ช, โˆš
๐’”(๐’”โˆ’๐’„)
(๐’”โˆ’๐’‚)(๐’”โˆ’๐’ƒ)
=
(a) sin
๐›พ
2
(b) cos
๐›พ
2
(c) tan
๐›พ
2
(d) โœ” cot
๐›พ
2
18. In any triangle ๐‘จ๐‘ฉ๐‘ช, โˆš
(๐’”โˆ’๐’‚)(๐’”โˆ’๐’ƒ)
๐’”(๐’”โˆ’๐’„)
=
(a) sin
๐›พ
2
(b) cos
๐›พ
2
(c) โœ” tan
๐›พ
2
(d) cot
๐›พ
2
19. To solve an oblique triangles when measure of three sides are given , we can use:
(a) โœ”Heroโ€™s formula (b) Law of cosine (c) Law of sine (d) Law of tangents
20. The smallest angle of โˆ†๐‘จ๐‘ฉ๐‘ช, when ๐’‚ = ๐Ÿ‘๐Ÿ•. ๐Ÿ‘๐Ÿ’ , ๐’ƒ = ๐Ÿ‘. ๐Ÿ๐Ÿ’ , ๐’„ = ๐Ÿ‘๐Ÿ“. ๐ŸŽ๐Ÿ” is
(a) ๐›ผ (b) โœ”๐›ฝ (c) ๐›พ (d) cannot be determined
21. In any triangle ๐‘จ๐‘ฉ๐‘ช Area if triangle is :
(a) ๐‘๐‘ sin๐›ผ (b)
1
2
๐‘๐‘Ž ๐‘ ๐‘–๐‘›๐›ผ (c)
1
2
๐‘Ž๐‘ ๐‘ ๐‘–๐‘›๐›ฝ (d) โœ”
1
2
๐‘Ž๐‘๐‘ ๐‘–๐‘›๐›พ
22. The circle passing through the thee vertices of a triangle is called:
(a) โœ”Circum circle (b) in-circle (c) ex-centre (d) escribed circle
23. The point of intersection of the right bisectors of the sides of the triangle is :
(a) โœ”Circum centre (b) In-centre (c) Escribed center (d) Diameter
24. In any triangle ๐‘จ๐‘ฉ๐‘ช, with usual notations,
๐’‚
๐Ÿ๐’”๐’Š๐’๐œถ
=
(a) ๐‘Ÿ (b) ๐‘Ÿ1 (c) โœ” ๐‘… (d) โˆ†
25. In any triangle ๐‘จ๐‘ฉ๐‘ช, with usual notations,
๐’‚
๐’”๐’Š๐’๐œท
=
(a) 2๐‘Ÿ (b)2 ๐‘Ÿ1 (c) โœ”2๐‘… (d) 2โˆ†
26. In any triangle ๐‘จ๐‘ฉ๐‘ช, with usual notations, ๐’”๐’Š๐’ ๐œธ =
(a) ๐‘… (b) โœ”
๐‘
2๐‘…
(c)
2๐‘…
๐‘
(d)
๐‘…
2
27. In any triangle ๐‘จ๐‘ฉ๐‘ช, with usual notations, ๐’‚๐’ƒ๐’„ =
(a) ๐‘… (b) ๐‘…๐‘  (c) โœ”4๐‘…โˆ† (d)
โˆ†
๐‘ 
28. In any triangle ๐‘จ๐‘ฉ๐‘ช, with usual notations,
โˆ†
๐’”โˆ’๐’‚
=
(a) ๐‘Ÿ (b) ๐‘… (c) โœ” ๐‘Ÿ1 (d) ๐‘Ÿ2
29. In any triangle ๐‘จ๐‘ฉ๐‘ช, with usual notations,
โˆ†
๐’”โˆ’๐’ƒ
=
(a) ๐‘Ÿ (b) ๐‘… (c) ๐‘Ÿ1 (d) โœ” ๐‘Ÿ2
30. In any triangle ๐‘จ๐‘ฉ๐‘ช, with usual notations,
โˆ†
๐’”โˆ’๐’„
=
(a) โœ”๐‘Ÿ3 (b) ๐‘… (c) ๐‘Ÿ1 (d) ๐‘Ÿ2
31. In any triangle ๐‘จ๐‘ฉ๐‘ช, with usual notation , ๐’“: ๐‘น: ๐’“๐Ÿ =
(a) 3:2:1 (b) 1:2:2 (c) โœ” 1:2:3 (d) 1:1:1
32. In any triangle ๐‘จ๐‘ฉ๐‘ช, with usual notation , ๐’“: ๐‘น: ๐’“๐Ÿ: ๐’“๐Ÿ: ๐’“๐Ÿ‘ =
(a) 3:3:3:2:1 (b) 1:2:2:3:3 (c) โœ” 1:2:3:3:3 (d) 1:1:1:1:1
33. In a triangle ๐‘จ๐‘ฉ๐‘ช, if ๐œท = ๐Ÿ”๐ŸŽยฐ , ๐œธ = ๐Ÿ๐Ÿ“ยฐ then ๐œถ =
(a) 90ยฐ (b) 180ยฐ (c) 150ยฐ (d) โœ”105ยฐ
UNIT # 13 Inverse trigonometric functions
Each question has four possible answer.Tick the correct answer.
1. If ๐’š = ๐‘บ๐’Š๐’๐’™, then Domain is :
(a) โœ”โˆ’
๐œ‹
2
โ‰ค ๐‘ฅ โ‰ค
๐œ‹
2
(b) 0 โ‰ค ๐‘ฅ โ‰ค ๐œ‹ (c) [0, ๐œ‹], ๐‘ฅ โ‰ 
๐œ‹
2
(d) [โˆ’
๐œ‹
2
,
๐œ‹
2
] , ๐‘ฅ โ‰  0
2. If ๐’š = ๐‘ช๐’๐’”๐’™, then Domain is :
(a) โˆ’
๐œ‹
2
โ‰ค ๐‘ฅ โ‰ค
๐œ‹
2
(b) โœ” 0 โ‰ค ๐‘ฅ โ‰ค ๐œ‹ (c) [0, ๐œ‹], ๐‘ฅ โ‰ 
๐œ‹
2
(d) [โˆ’
๐œ‹
2
,
๐œ‹
2
] , ๐‘ฅ โ‰  0
3. If ๐’š = ๐‘บ๐’†๐’„๐’™, then Domain is :
(a) โˆ’
๐œ‹
2
โ‰ค ๐‘ฅ โ‰ค
๐œ‹
2
(b) 0 โ‰ค ๐‘ฅ โ‰ค ๐œ‹ (c) โœ” [0, ๐œ‹], ๐‘ฅ โ‰ 
๐œ‹
2
(d) [โˆ’
๐œ‹
2
,
๐œ‹
2
] , ๐‘ฅ โ‰  0
20 | P a g e
4. If ๐’š = ๐‘ช๐’๐’”๐’†๐’„๐’™, then Domain is :
(a) โˆ’
๐œ‹
2
โ‰ค ๐‘ฅ โ‰ค
๐œ‹
2
(b) 0 โ‰ค ๐‘ฅ โ‰ค ๐œ‹ (c) [0, ๐œ‹], ๐‘ฅ โ‰ 
๐œ‹
2
(d) โœ” [โˆ’
๐œ‹
2
,
๐œ‹
2
] , ๐‘ฅ โ‰  0
5. If ๐’š = ๐‘ป๐’‚๐’๐’™, then Domain is :
(a) โˆ’
๐œ‹
2
โ‰ค ๐‘ฅ โ‰ค
๐œ‹
2
(b) โœ”0 โ‰ค ๐‘ฅ โ‰ค ๐œ‹ (c) [0, ๐œ‹], ๐‘ฅ โ‰ 
๐œ‹
2
(d) [โˆ’
๐œ‹
2
,
๐œ‹
2
] , ๐‘ฅ โ‰  0
6. If ๐’š = ๐‘ช๐’๐’•๐’™, then Domain is :
(a) โˆ’
๐œ‹
2
โ‰ค ๐‘ฅ โ‰ค
๐œ‹
2
(b) 0 โ‰ค ๐‘ฅ โ‰ค ๐œ‹ (c) โœ” [0, ๐œ‹], ๐‘ฅ โ‰ 
๐œ‹
2
(d) [โˆ’
๐œ‹
2
,
๐œ‹
2
] , ๐‘ฅ โ‰  0
7. If ๐’š = ๐‘บ๐’Š๐’๐’™, then range is :
(a) โœ”โˆ’1 โ‰ค ๐‘ฅ โ‰ค 1 (b) (โˆ’โˆž, +โˆž)๐‘œ๐‘Ÿ ๐‘… (c) ๐‘ฆ โ‰ค โˆ’1 ๐‘œ๐‘Ÿ ๐‘ฆ โ‰ฅ 1 (d) ๐‘ฆ < โˆ’1๐‘œ๐‘Ÿ ๐‘ฆ > 1
8. If ๐’š = ๐‘ช๐’๐’”๐’™, then range is :
(a) โœ”โˆ’1 โ‰ค ๐‘ฅ โ‰ค 1 (b) (โˆ’โˆž, +โˆž)๐‘œ๐‘Ÿ ๐‘… (c) ๐‘ฆ โ‰ค โˆ’1 ๐‘œ๐‘Ÿ ๐‘ฆ โ‰ฅ 1 (d) ๐‘ฆ < โˆ’1๐‘œ๐‘Ÿ ๐‘ฆ > 1
9. If ๐’š = ๐‘ป๐’‚๐’๐’™, then range is :
(a) โˆ’1 โ‰ค ๐‘ฅ โ‰ค 1 (b) โœ” (โˆ’โˆž, +โˆž)๐‘œ๐‘Ÿ ๐‘… (c) ๐‘ฆ โ‰ค โˆ’1 ๐‘œ๐‘Ÿ ๐‘ฆ โ‰ฅ 1 (d) ๐‘ฆ < โˆ’1๐‘œ๐‘Ÿ ๐‘ฆ > 1
10. If ๐’š = ๐‘ช๐’๐’•๐’™, then range is :
(a) โˆ’1 โ‰ค ๐‘ฅ โ‰ค 1 (b) โœ” (โˆ’โˆž, +โˆž)๐‘œ๐‘Ÿ ๐‘… (c) ๐‘ฆ โ‰ค โˆ’1 ๐‘œ๐‘Ÿ ๐‘ฆ โ‰ฅ 1 (d) ๐‘ฆ < โˆ’1๐‘œ๐‘Ÿ ๐‘ฆ > 1
11. If ๐’š = ๐‘ช๐’๐’”๐’†๐’„๐’™, then range is :
(a) โˆ’1 โ‰ค ๐‘ฅ โ‰ค 1 (b) (โˆ’โˆž, +โˆž)๐‘œ๐‘Ÿ ๐‘… (c) โœ”๐‘ฆ โ‰ค โˆ’1 ๐‘œ๐‘Ÿ ๐‘ฆ โ‰ฅ 1 (d) ๐‘ฆ < โˆ’1๐‘œ๐‘Ÿ ๐‘ฆ > 1
12. If ๐’š = ๐‘บ๐’†๐’„๐’™, then range is :
(a) โˆ’1 โ‰ค ๐‘ฅ โ‰ค 1 (b) (โˆ’โˆž, +โˆž)๐‘œ๐‘Ÿ ๐‘… (c) โœ”๐‘ฆ โ‰ค โˆ’1 ๐‘œ๐‘Ÿ ๐‘ฆ โ‰ฅ 1 (d) ๐‘ฆ < โˆ’1๐‘œ๐‘Ÿ ๐‘ฆ > 1
13. If ๐’š = ๐‘บ๐’Š๐’โˆ’๐Ÿ
๐’™, then domain is:
(a) โœ” โˆ’ 1 โ‰ค ๐‘ฅ โ‰ค 1 (b) (โˆ’โˆž, +โˆž)๐‘œ๐‘Ÿ ๐‘… (c) ๐‘ฅ โ‰ฅ โˆ’1 ๐‘œ๐‘Ÿ ๐‘ฅ โ‰ค 1 (d) ๐‘ฅ โ‰ค โˆ’1 ๐‘œ๐‘Ÿ ๐‘ฅ โ‰ฅ 1
14. If ๐’š = ๐‘ช๐’๐’”โˆ’๐Ÿ
๐’™, then domain is:
(a) โœ” โˆ’ 1 โ‰ค ๐‘ฅ โ‰ค 1 (b) (โˆ’โˆž, +โˆž)๐‘œ๐‘Ÿ ๐‘… (c) ๐‘ฅ โ‰ฅ โˆ’1 ๐‘œ๐‘Ÿ ๐‘ฅ โ‰ค 1 (d) ๐‘ฅ โ‰ค โˆ’1 ๐‘œ๐‘Ÿ ๐‘ฅ โ‰ฅ 1
15. If ๐’š = ๐‘ป๐’‚๐’โˆ’๐Ÿ
๐’™, then domain is:
(a) โˆ’1 โ‰ค ๐‘ฅ โ‰ค 1 (b) โœ” (โˆ’โˆž, +โˆž)๐‘œ๐‘Ÿ ๐‘… (c) ๐‘ฅ โ‰ฅ โˆ’1 ๐‘œ๐‘Ÿ ๐‘ฅ โ‰ค 1 (d) ๐‘ฅ โ‰ค โˆ’1 ๐‘œ๐‘Ÿ ๐‘ฅ โ‰ฅ 1
16. If ๐’š = ๐‘ช๐’๐’•โˆ’๐Ÿ
๐’™, then domain is:
(a) โˆ’1 โ‰ค ๐‘ฅ โ‰ค 1 (b) โœ” (โˆ’โˆž, +โˆž)๐‘œ๐‘Ÿ ๐‘… (c) ๐‘ฅ โ‰ฅ โˆ’1 ๐‘œ๐‘Ÿ ๐‘ฅ โ‰ค 1 (d) ๐‘ฅ โ‰ค โˆ’1 ๐‘œ๐‘Ÿ ๐‘ฅ โ‰ฅ 1
17. If ๐’š = ๐‘บ๐’†๐’„โˆ’๐Ÿ
๐’™, then domain is:
(a) โˆ’1 โ‰ค ๐‘ฅ โ‰ค 1 (b) (โˆ’โˆž, +โˆž)๐‘œ๐‘Ÿ ๐‘… (c) โœ” ๐‘ฅ โ‰ฅ โˆ’1 ๐‘œ๐‘Ÿ ๐‘ฅ โ‰ค 1 (d) ๐‘ฅ โ‰ค โˆ’1 ๐‘œ๐‘Ÿ ๐‘ฅ โ‰ฅ 1
18. If ๐’š = ๐‘ช๐’๐’”๐’†๐’„โˆ’๐Ÿ
๐’™, then domain is:
(a) โˆ’1 โ‰ค ๐‘ฅ โ‰ค 1 (b) (โˆ’โˆž, +โˆž)๐‘œ๐‘Ÿ ๐‘… (c) โœ” ๐‘ฅ โ‰ฅ โˆ’1 ๐‘œ๐‘Ÿ ๐‘ฅ โ‰ค 1 (d) ๐‘ฅ โ‰ค โˆ’1 ๐‘œ๐‘Ÿ ๐‘ฅ โ‰ฅ 1
19. If ๐’š = ๐‘บ๐’Š๐’โˆ’๐Ÿ
๐’™, then range is:
(a) โœ” โˆ’
๐œ‹
2
โ‰ค ๐‘ฅ โ‰ค
๐œ‹
2
(b) 0 โ‰ค ๐‘ฅ โ‰ค ๐œ‹ (c) โ€“
๐œ‹
2
< ๐‘ฅ <
๐œ‹
2
(d) 0 < ๐‘ฅ < ๐œ‹
20. If ๐’š = ๐‘ช๐’๐’”โˆ’๐Ÿ
๐’™, then range is:
(a) โˆ’
๐œ‹
2
โ‰ค ๐‘ฅ โ‰ค
๐œ‹
2
(b) โœ” 0 โ‰ค ๐‘ฅ โ‰ค ๐œ‹ (c) โ€“
๐œ‹
2
< ๐‘ฅ <
๐œ‹
2
(d) 0 < ๐‘ฅ < ๐œ‹
21. If ๐’š = ๐‘ป๐’‚๐’โˆ’๐Ÿ
๐’™, then range is:
(a) โˆ’
๐œ‹
2
โ‰ค ๐‘ฅ โ‰ค
๐œ‹
2
(b) 0 โ‰ค ๐‘ฅ โ‰ค ๐œ‹ (c) โœ” โ€“
๐œ‹
2
< ๐‘ฅ <
๐œ‹
2
(d) 0 < ๐‘ฅ < ๐œ‹
22. If ๐’š = ๐‘ช๐’๐’•โˆ’๐Ÿ
๐’™, then range is:
(a) โˆ’
๐œ‹
2
โ‰ค ๐‘ฅ โ‰ค
๐œ‹
2
(b) 0 โ‰ค ๐‘ฅ โ‰ค ๐œ‹ (c) โ€“
๐œ‹
2
< ๐‘ฅ <
๐œ‹
2
(d) โœ” 0 < ๐‘ฅ < ๐œ‹
23. If ๐’š = ๐’”๐’†๐’„โˆ’๐Ÿ
๐’™, then range is:
(a) โœ” โˆ’
๐œ‹
2
โ‰ค ๐‘ฅ โ‰ค
๐œ‹
2
(b) 0 โ‰ค ๐‘ฅ โ‰ค ๐œ‹ (c) โ€“
๐œ‹
2
< ๐‘ฅ <
๐œ‹
2
(d) 0 < ๐‘ฅ < ๐œ‹
24. If ๐’š = ๐‘ช๐’๐’”๐’†๐’„โˆ’๐Ÿ
๐’™, then range is:
(a) โˆ’
๐œ‹
2
โ‰ค ๐‘ฅ โ‰ค
๐œ‹
2
(b) โœ” 0 โ‰ค ๐‘ฅ โ‰ค ๐œ‹ (c) โ€“
๐œ‹
2
< ๐‘ฅ <
๐œ‹
2
(d) 0 < ๐‘ฅ < ๐œ‹
25. Inverse of a function exist only if it is:
(a) Trigonometric function (b) โœ”(1 โˆ’ 1) function (c) onto function (d) an into function
26. ๐‘บ๐’Š๐’โˆ’๐Ÿ
๐’™ =
(a) โœ”
๐œ‹
2
โˆ’ cosโˆ’1
๐‘ฅ (b)
๐œ‹
2
โˆ’ sinโˆ’1
๐‘ฅ (c)
๐œ‹
2
+ cosโˆ’1
๐‘ฅ (d)
๐œ‹
2
โˆ’ ๐‘๐‘œ๐‘ ๐‘’๐‘โˆ’1
๐‘ฅ
27. ๐‘ช๐’๐’”โˆ’๐Ÿ
๐’™ =
(a)
๐œ‹
2
โˆ’ cosโˆ’1
๐‘ฅ (b) โœ”
๐œ‹
2
โˆ’ sinโˆ’1
๐‘ฅ (c)
๐œ‹
2
+ cosโˆ’1
๐‘ฅ (d)
๐œ‹
2
โˆ’ ๐‘๐‘œ๐‘ ๐‘’๐‘โˆ’1
๐‘ฅ
28. ๐‘ช๐’๐’”โˆ’๐Ÿ
๐’™ =
(a) โœ”
๐œ‹
2
โˆ’ secโˆ’1
๐‘ฅ (b)
๐œ‹
2
โˆ’ sinโˆ’1
๐‘ฅ (c)
๐œ‹
2
+ secโˆ’1
๐‘ฅ (d)
๐œ‹
2
โˆ’ ๐‘๐‘œ๐‘ ๐‘’๐‘โˆ’1
๐‘ฅ
29. ๐‘บ๐’†๐’„โˆ’๐Ÿ
๐’™ =
(a)
๐œ‹
2
โˆ’ secโˆ’1
๐‘ฅ (b)
๐œ‹
2
โˆ’ sinโˆ’1
๐‘ฅ (c)
๐œ‹
2
+ secโˆ’1
๐‘ฅ (d) โœ”
๐œ‹
2
โˆ’ ๐‘๐‘œ๐‘ ๐‘’๐‘โˆ’1
๐‘ฅ
30. ๐‘ป๐’‚๐’โˆ’๐Ÿ
๐’™ =
(a)
๐œ‹
2
โˆ’ secโˆ’1
๐‘ฅ (b)
๐œ‹
2
โˆ’ sinโˆ’1
๐‘ฅ (c) โœ”
๐œ‹
2
โˆ’cotโˆ’1
๐‘ฅ (d)
๐œ‹
2
โˆ’ ๐‘๐‘œ๐‘ ๐‘’๐‘โˆ’1
๐‘ฅ
31. ๐‘ช๐’๐’•โˆ’๐Ÿ
๐’™ =
(a)
๐œ‹
2
โˆ’ secโˆ’1
๐‘ฅ (b)โœ”
๐œ‹
2
โˆ’ tanโˆ’1
๐‘ฅ (c)
๐œ‹
2
+ secโˆ’1
๐‘ฅ (d)
๐œ‹
2
โˆ’ ๐‘๐‘œ๐‘ ๐‘’๐‘โˆ’1
๐‘ฅ
21 | P a g e
32. ๐‘บ๐’Š๐’ (๐‘ช๐’๐’”โˆ’๐Ÿ โˆš๐Ÿ‘
๐Ÿ
) =
(a)
๐œ‹
6
(b) โœ”
1
2
(c) โˆ’
1
2
(d)
โˆš3
2
33. ๐‘ป๐’‚๐’โˆ’๐Ÿ
(โˆš๐Ÿ‘) =
(a)
๐œ‹
6
(b) โ€“
๐œ‹
6
(c) โ€“
๐œ‹
3
(d) โœ”
๐œ‹
3
34. ๐‘บ๐’Š๐’ (๐‘บ๐’Š๐’โˆ’๐Ÿ ๐Ÿ
๐Ÿ
) =
(a) โœ”
1
2
(b)
2
3
(c) 2 (d)
1
3
35. ๐‘บ๐’Š๐’โˆ’๐Ÿ
๐‘จ + ๐‘บ๐’Š๐’โˆ’๐Ÿ
๐‘ฉ =
(a) โœ”๐‘†๐‘–๐‘›โˆ’1
(๐ดโˆš1 โˆ’ ๐ต2
+ ๐ตโˆš1 โˆ’ ๐ด2
) (b) ๐‘†๐‘–๐‘›โˆ’1
(๐ดโˆš1 โˆ’ ๐ด2 โˆ’ ๐ตโˆš1 โˆ’ ๐ต2)
(c) ๐‘†๐‘–๐‘›โˆ’1
(๐ตโˆš1 โˆ’ ๐ด2 + ๐ดโˆš1 โˆ’ ๐ต2) (d) ๐‘†๐‘–๐‘›โˆ’1
(๐ด๐ตโˆš(1 โˆ’ ๐ด2)(1 โˆ’ ๐ต2))
36. ๐‘บ๐’Š๐’โˆ’๐Ÿ
๐‘จ โˆ’ ๐‘บ๐’Š๐’โˆ’๐Ÿ
๐‘ฉ =
(a) ๐‘†๐‘–๐‘›โˆ’1
(๐ดโˆš1 โˆ’ ๐ต2 + ๐ตโˆš1 โˆ’ ๐ด2) (b) โœ” ๐‘†๐‘–๐‘›โˆ’1
(๐ดโˆš1 โˆ’ ๐ด2 โˆ’ ๐ตโˆš1 โˆ’ ๐ต2)
(c) ๐‘†๐‘–๐‘›โˆ’1
(๐ตโˆš1 โˆ’ ๐ด2 + ๐ดโˆš1 โˆ’ ๐ต2) (d) ๐‘†๐‘–๐‘›โˆ’1
(๐ด๐ตโˆš(1 โˆ’ ๐ด2)(1 โˆ’ ๐ต2))
37. ๐‘ช๐’๐’”โˆ’๐Ÿ
๐‘จ + ๐‘ช๐’๐’”โˆ’๐Ÿ
๐‘ฉ =
(a) ๐ถ๐‘œ๐‘ โˆ’1
(๐ด๐ต โˆ’ โˆš(1 โˆ’ ๐ด2)(1 โˆ’ ๐ต2)) (b) ๐ถ๐‘œ๐‘ โˆ’1
(๐ด๐ต + โˆš(1 โˆ’ ๐ด2)(1 โˆ’ ๐ต2))
(c) ๐ถ๐‘œ๐‘ โˆ’1
(๐ด๐ต โˆ’ โˆš(1 + ๐ด2)(1 + ๐ต2)) (d) ๐ถ๐‘œ๐‘ โˆ’1
(๐ด๐ต + โˆš(1 + ๐ด2)(1 + ๐ต2))
38. ๐‘ช๐’๐’”โˆ’๐Ÿ
๐‘จ + ๐‘ช๐’๐’”โˆ’๐Ÿ
๐‘ฉ =
(a) โœ” ๐ถ๐‘œ๐‘ โˆ’1
(๐ด๐ต โˆ’ โˆš(1 โˆ’ ๐ด2)(1 โˆ’ ๐ต2)) (b) โœ” ๐ถ๐‘œ๐‘ โˆ’1
(๐ด๐ต + โˆš(1 โˆ’ ๐ด2)(1 โˆ’ ๐ต2))
(c) ๐ถ๐‘œ๐‘ โˆ’1
(๐ด๐ต โˆ’ โˆš(1 + ๐ด2)(1 + ๐ต2)) (d) ๐ถ๐‘œ๐‘ โˆ’1
(๐ด๐ต + โˆš(1 + ๐ด2)(1 + ๐ต2))
39. ๐‘ป๐’‚๐’โˆ’๐Ÿ
๐‘จ + ๐‘ป๐’‚๐’โˆ’๐Ÿ
๐‘ฉ =
(a) โœ”๐‘‡๐‘Ž๐‘›โˆ’1
(
๐ดโˆ’๐ต
1+๐ด๐ต
) (b) ๐‘‡๐‘Ž๐‘›โˆ’1
(
๐ด+๐ต
1+๐ด๐ต
) (c) ๐‘‡๐‘Ž๐‘›โˆ’1
(
๐ดโˆ’๐ต
1โˆ’๐ด๐ต
) (d) ๐‘‡๐‘Ž๐‘›โˆ’1
(
๐ด+๐ต
1+๐ด๐ต
)
40. ๐‘ป๐’‚๐’โˆ’๐Ÿ
๐‘จ + ๐‘ป๐’‚๐’โˆ’๐Ÿ
๐‘ฉ =
(a) ๐‘‡๐‘Ž๐‘›โˆ’1
(
๐ดโˆ’๐ต
1+๐ด๐ต
) (b) โœ” ๐‘‡๐‘Ž๐‘›โˆ’1
(
๐ด+๐ต
1+๐ด๐ต
) (c) ๐‘‡๐‘Ž๐‘›โˆ’1
(
๐ดโˆ’๐ต
1โˆ’๐ด๐ต
) (d) ๐‘‡๐‘Ž๐‘›โˆ’1
(
๐ด+๐ต
1+๐ด๐ต
)
41. ๐‘บ๐’Š๐’โˆ’๐Ÿ(โˆ’๐’™) =
(a) โœ”โ€“ ๐‘†๐‘–๐‘›โˆ’1
๐‘ฅ (b) ๐‘†๐‘–๐‘›โˆ’1
๐‘ฅ (c) ๐œ‹ โˆ’ ๐‘†๐‘–๐‘›โˆ’1
๐‘ฅ (d) ๐œ‹ โˆ’ ๐‘†๐‘–๐‘›๐‘ฅ
42. ๐‘ช๐’๐’”โˆ’๐Ÿ(โˆ’๐’™) =
(a) โ€“ ๐ถ๐‘œ๐‘ โˆ’1
๐‘ฅ (b) ๐ถ๐‘œ๐‘ โˆ’1
๐‘ฅ (c) โœ” ๐œ‹ โˆ’ ๐ถ๐‘œ๐‘ โˆ’1
๐‘ฅ (d) ๐œ‹ โˆ’ ๐ถ๐‘œ๐‘ ๐‘ฅ
43. ๐‘ป๐’‚๐’โˆ’๐Ÿ(โˆ’๐’™) =
(a) โœ”โ€“ ๐‘‡๐‘Ž๐‘›โˆ’1
๐‘ฅ (b) ๐‘‡๐‘Ž๐‘›โˆ’1
๐‘ฅ (c) ๐œ‹ โˆ’ ๐‘‡๐‘Ž๐‘›โˆ’1
๐‘ฅ (d) ๐œ‹ โˆ’ ๐‘‡๐‘Ž๐‘›๐‘ฅ
44. ๐‘ช๐’๐’”๐’†๐’„โˆ’๐Ÿ(โˆ’๐’™) =
(a) โœ”โ€“ ๐ถ๐‘œ๐‘ ๐‘’๐‘โˆ’1
๐‘ฅ (b) ๐ถ๐‘œ๐‘ ๐‘’๐‘โˆ’1
๐‘ฅ (c) ๐œ‹ โˆ’ ๐ถ๐‘œ๐‘ ๐‘’๐‘โˆ’1
๐‘ฅ (d) ๐œ‹ โˆ’ ๐ถ๐‘œ๐‘ ๐‘’๐‘๐‘ฅ
45. ๐‘บ๐’†๐’„โˆ’๐Ÿ(โˆ’๐’™) =
(a) โ€“ ๐‘†๐‘’๐‘โˆ’1
๐‘ฅ (b) ๐‘†๐‘’๐‘โˆ’1
๐‘ฅ (c) โœ” ๐œ‹ โˆ’ ๐‘†๐‘’๐‘โˆ’1
๐‘ฅ (d) ๐œ‹ โˆ’ ๐‘†๐‘’๐‘๐‘ฅ
46. ๐‘ช๐’๐’•โˆ’๐Ÿ(โˆ’๐’™) =
(a) โˆ’๐ถ๐‘œ๐‘กโˆ’1
๐‘ฅ (b) ๐ถ๐‘œ๐‘กโˆ’1
๐‘ฅ (c) โœ” ๐œ‹ โˆ’ ๐ถ๐‘œ๐‘กโˆ’1
๐‘ฅ (d) ๐œ‹ โˆ’ ๐ถ๐‘œ๐‘ก๐‘ฅ
UNIT # 14 Solution of Trigonometric
Equations
Each question has four possible answer. Tick the correct answer.
1. An equation containing at least one trigonometric function is called:
(a) Trigonometric function (b) โœ”Trigonometric equation (c) Trigonometric value (d) None
2. If ๐’”๐’Š๐’๐’™ =
๐Ÿ
๐Ÿ
, then solution in the interval [๐ŸŽ, ๐Ÿ๐…] is:
(a) โœ”{
๐œ‹
6
,
5๐œ‹
6
} (b) {
๐œ‹
6
,
7๐œ‹
6
} (c) {
๐œ‹
3
,
4๐œ‹
6
} (d) {
๐œ‹
3
,
2๐œ‹
6
}
3. If ๐’„๐’๐’”๐’™ =
๐Ÿ
๐Ÿ
, then the reference angle is:
(a) โœ”
๐œ‹
3
(b) โ€“
๐œ‹
3
(c)
๐œ‹
6
(d) โ€“
๐œ‹
6
4. If ๐’”๐’Š๐’๐’™ =
๐Ÿ
๐Ÿ
, then the reference angle is:
(a)
๐œ‹
3
(b) โ€“
๐œ‹
3
(c) โœ”
๐œ‹
6
(d) โ€“
๐œ‹
6
22 | P a g e
5. General solution of ๐’•๐’‚๐’๐’™ = ๐Ÿ is:
(a) โœ”{
๐œ‹
4
+ ๐‘›๐œ‹,
5๐œ‹
4
+ ๐‘›๐œ‹} (b) {
๐œ‹
4
+ 2๐‘›๐œ‹,
5๐œ‹
4
+ 2๐‘›๐œ‹} (c) {
๐œ‹
4
+ ๐‘›๐œ‹,
3๐œ‹
4
+ ๐‘›๐œ‹} (d) {
๐œ‹
4
+ 2๐‘›๐œ‹,
3๐œ‹
4
+ 2๐‘›๐œ‹}
6. If ๐’•๐’‚๐’๐Ÿ๐’™ = โˆ’๐Ÿ, then solution in the interval [๐ŸŽ, ๐…]is:
(a) โœ”
๐œ‹
8
(b)
๐œ‹
4
(c)
3๐œ‹
8
(d)
3๐œ‹
4
7. If ๐’”๐’Š๐’๐’™ + ๐’„๐’๐’”๐’™ = ๐ŸŽ then value of ๐’™ โˆˆ [๐ŸŽ, ๐Ÿ๐…]
(a) {
๐œ‹
4
,
3๐œ‹
4
} (b) {
๐œ‹
4
,
7๐œ‹
4
} (c) โœ” {
3๐œ‹
4
,
7๐œ‹
4
} (d) {
๐œ‹
4
,
โˆ’๐œ‹
4
}
8. General solution of ๐Ÿ’๐’”๐’Š๐’๐’™ โˆ’ ๐Ÿ– = ๐ŸŽ is:
(a) {๐œ‹ + 2๐‘›๐œ‹} (b) {๐œ‹ + ๐‘›๐œ‹} (c) {โˆ’๐œ‹ + ๐‘›๐œ‹} (d) โœ”not possible
9. General solution of ๐Ÿ + ๐’„๐’๐’”๐’™ = ๐ŸŽ is:
(a) โœ”{๐œ‹ + 2๐‘›๐œ‹} (b) {๐œ‹ + ๐‘›๐œ‹} (c) {โˆ’๐œ‹ + ๐‘›๐œ‹} (d) not possible
10. For the general solution , we first find the solution in the interval whose length is equal to
its:
(a) Range (b) domain (c) co-domain (d) โœ” period
11. All trigonometric functions are โ€ฆโ€ฆโ€ฆโ€ฆโ€ฆโ€ฆ.. functions.
(a) โœ”Periodic (b) continues (c) injective (d) bijective
12. General solution of every trigonometric equation consists of :
(a) One solution only (b) two solutions (c) โœ” infinitely many solutions (d) no real solution
13. Solution of the equation ๐Ÿ๐’”๐’Š๐’๐’™ + โˆš๐Ÿ‘ = ๐ŸŽ in the 4th
quadrant is:
(a)
๐œ‹
2
(bโœ”)
โˆ’๐œ‹
3
(c)
โˆ’๐œ‹
6
(d)
11๐œ‹
6
14. If ๐’”๐’Š๐’๐’™ = ๐’„๐’๐’”๐’™, then general solution is:
(a) {
๐œ‹
4
+ ๐‘›๐œ‹, ๐‘› โˆˆ ๐‘} (b) {
๐œ‹
4
+ 2๐‘›๐œ‹, ๐‘› โˆˆ ๐‘}
(c) โœ” {
๐œ‹
4
+ ๐‘›๐œ‹,
5๐œ‹
4
+ ๐‘›๐œ‹} (d){
๐œ‹
4
+ ๐‘›๐œ‹,
5๐œ‹
4
+ ๐‘›๐œ‹}
15. In which quadrant is the solution of the equation ๐’”๐’Š๐’๐’™ + ๐Ÿ = ๐ŸŽ
(a) 1st
and 2nd
(b) 2nd
and 3rd
(c) โœ” 3rd
and 4th
(d) Only 1st
16. If ๐’”๐’Š๐’๐’™ = ๐ŸŽ then ๐’™ =
(a) โœ”๐‘›๐œ‹ , ๐‘› โˆˆ ๐‘ (b)
๐‘›๐œ‹
2
, ๐‘› โˆˆ ๐‘ (c) 0 (d)
๐œ‹
2
17. If ๐’•๐’‚๐’๐’™ = ๐ŸŽ then ๐’™ =
(a) โœ”๐‘›๐œ‹ , ๐‘› โˆˆ ๐‘ (b)
๐‘›๐œ‹
2
, ๐‘› โˆˆ ๐‘ (c) 0 (d)
๐œ‹
2
18. The solution of the ๐’„๐’๐’•๐’™ =
๐Ÿ
โˆš๐Ÿ‘
in [๐ŸŽ, ๐…] is
(a)
๐œ‹
6
(b)
๐œ‹
4
(c)
2๐œ‹
3
(d) โœ”
๐œ‹
3
19. One solution of ๐’”๐’†๐’„๐’™ = โˆ’๐Ÿ is :
(a)
๐œ‹
6
(b)
๐œ‹
4
(c) โœ”
2๐œ‹
3
(d)
๐œ‹
3
20. ๐‘บ๐’Š๐’๐Ÿ๐’™ =
โˆš๐Ÿ‘
๐Ÿ
has two values of ๐’™ in the interval:
(a) [0,
๐œ‹
2
] (b) [0,2๐œ‹] (c) [โ€“ ๐œ‹,
๐œ‹
2
] (d) [โˆ’
๐œ‹
2
, 0]
๏ƒŸ----------THE END--------๏ƒ 
WITH BEST WISHES BY:-
MUHAMMAD SALMAN
SHERAZI M.Sc(Math) , B.ed
03337727666/03067856232

More Related Content

What's hot

5.4 mathematical induction
5.4 mathematical induction5.4 mathematical induction
5.4 mathematical induction
math260
ย 
Complex Numbers
Complex NumbersComplex Numbers
Complex Numbers
itutor
ย 
Infinite series
Infinite seriesInfinite series
Infinite series
jaflint718
ย 

What's hot (20)

CONVERGENCE.ppt
CONVERGENCE.pptCONVERGENCE.ppt
CONVERGENCE.ppt
ย 
5.4 mathematical induction
5.4 mathematical induction5.4 mathematical induction
5.4 mathematical induction
ย 
Maths
MathsMaths
Maths
ย 
Complex number
Complex numberComplex number
Complex number
ย 
Complex number
Complex numberComplex number
Complex number
ย 
21 monotone sequences x
21 monotone sequences x21 monotone sequences x
21 monotone sequences x
ย 
Linear algebra-Basis & Dimension
Linear algebra-Basis & DimensionLinear algebra-Basis & Dimension
Linear algebra-Basis & Dimension
ย 
Complex Numbers
Complex NumbersComplex Numbers
Complex Numbers
ย 
Complex analysis
Complex analysisComplex analysis
Complex analysis
ย 
Differential equations
Differential equationsDifferential equations
Differential equations
ย 
1 directed angle
1 directed angle1 directed angle
1 directed angle
ย 
ORTHOGONAL, ORTHONORMAL VECTOR, GRAM SCHMIDT PROCESS, ORTHOGONALLY DIAGONALI...
ORTHOGONAL, ORTHONORMAL  VECTOR, GRAM SCHMIDT PROCESS, ORTHOGONALLY DIAGONALI...ORTHOGONAL, ORTHONORMAL  VECTOR, GRAM SCHMIDT PROCESS, ORTHOGONALLY DIAGONALI...
ORTHOGONAL, ORTHONORMAL VECTOR, GRAM SCHMIDT PROCESS, ORTHOGONALLY DIAGONALI...
ย 
Vcla - Inner Products
Vcla - Inner ProductsVcla - Inner Products
Vcla - Inner Products
ย 
Group Actions
Group ActionsGroup Actions
Group Actions
ย 
1631 the binomial theorem
1631 the binomial theorem1631 the binomial theorem
1631 the binomial theorem
ย 
02 linear algebra
02 linear algebra02 linear algebra
02 linear algebra
ย 
Ordinary differential equations
Ordinary differential equationsOrdinary differential equations
Ordinary differential equations
ย 
Series solutions at ordinary point and regular singular point
Series solutions at ordinary point and regular singular pointSeries solutions at ordinary point and regular singular point
Series solutions at ordinary point and regular singular point
ย 
Infinite series
Infinite seriesInfinite series
Infinite series
ย 
Group Theory
Group TheoryGroup Theory
Group Theory
ย 

Similar to FSC part1 Math MCQS -

9th class maths MCQs by Ustani G.docx
9th class maths MCQs by Ustani G.docx9th class maths MCQs by Ustani G.docx
9th class maths MCQs by Ustani G.docx
Waseem798409
ย 

Similar to FSC part1 Math MCQS - (20)

9th class maths MCQs by Ustani G.docx
9th class maths MCQs by Ustani G.docx9th class maths MCQs by Ustani G.docx
9th class maths MCQs by Ustani G.docx
ย 
IIT JAM Math 2022 Question Paper | Sourav Sir's Classes
IIT JAM Math 2022 Question Paper | Sourav Sir's ClassesIIT JAM Math 2022 Question Paper | Sourav Sir's Classes
IIT JAM Math 2022 Question Paper | Sourav Sir's Classes
ย 
Maths mind paper 2013
Maths mind paper 2013Maths mind paper 2013
Maths mind paper 2013
ย 
Mte (1)
Mte (1)Mte (1)
Mte (1)
ย 
IIT JAM MATH 2018 Question Paper | Sourav Sir's Classes
IIT JAM MATH 2018 Question Paper | Sourav Sir's ClassesIIT JAM MATH 2018 Question Paper | Sourav Sir's Classes
IIT JAM MATH 2018 Question Paper | Sourav Sir's Classes
ย 
Mcqs -Matrices and determinants
Mcqs -Matrices and determinantsMcqs -Matrices and determinants
Mcqs -Matrices and determinants
ย 
IIT JAM Mathematical Statistics - MS 2022 | Sourav Sir's Classes
IIT JAM Mathematical Statistics - MS 2022 | Sourav Sir's ClassesIIT JAM Mathematical Statistics - MS 2022 | Sourav Sir's Classes
IIT JAM Mathematical Statistics - MS 2022 | Sourav Sir's Classes
ย 
Advanced algebra
Advanced algebraAdvanced algebra
Advanced algebra
ย 
ObjectiveQuestionsonEngineeringMathematicsForGATE2022.pdf
ObjectiveQuestionsonEngineeringMathematicsForGATE2022.pdfObjectiveQuestionsonEngineeringMathematicsForGATE2022.pdf
ObjectiveQuestionsonEngineeringMathematicsForGATE2022.pdf
ย 
IIT JAM MATH 2019 Question Paper | Sourav Sir's Classes
IIT JAM MATH 2019 Question Paper | Sourav Sir's ClassesIIT JAM MATH 2019 Question Paper | Sourav Sir's Classes
IIT JAM MATH 2019 Question Paper | Sourav Sir's Classes
ย 
10 std objective test-1
10  std objective test-110  std objective test-1
10 std objective test-1
ย 
Mcq exemplar class 12
Mcq exemplar class 12Mcq exemplar class 12
Mcq exemplar class 12
ย 
Mcq exemplar class 12
Mcq exemplar class 12Mcq exemplar class 12
Mcq exemplar class 12
ย 
MATRICES-MATHED204.pptx
MATRICES-MATHED204.pptxMATRICES-MATHED204.pptx
MATRICES-MATHED204.pptx
ย 
3 capitulo-iii-matriz-asociada-sem-13-t-l-c
3 capitulo-iii-matriz-asociada-sem-13-t-l-c3 capitulo-iii-matriz-asociada-sem-13-t-l-c
3 capitulo-iii-matriz-asociada-sem-13-t-l-c
ย 
Banco de preguntas para el ap
Banco de preguntas para el apBanco de preguntas para el ap
Banco de preguntas para el ap
ย 
NUMERICAL METHODS MULTIPLE CHOICE QUESTIONS
NUMERICAL METHODS MULTIPLE CHOICE QUESTIONSNUMERICAL METHODS MULTIPLE CHOICE QUESTIONS
NUMERICAL METHODS MULTIPLE CHOICE QUESTIONS
ย 
Engg math's practice question by satya prakash 1
Engg math's practice question by satya prakash 1Engg math's practice question by satya prakash 1
Engg math's practice question by satya prakash 1
ย 
Class 9th Mcq for Preparing Exams , Maths
Class 9th Mcq for Preparing Exams , MathsClass 9th Mcq for Preparing Exams , Maths
Class 9th Mcq for Preparing Exams , Maths
ย 
Maths (2)
Maths (2)Maths (2)
Maths (2)
ย 

More from Muhammad Fahad Khan

Ancient history of south asia
Ancient history of south asiaAncient history of south asia
Ancient history of south asia
Muhammad Fahad Khan
ย 

More from Muhammad Fahad Khan (8)

Different kind of advertisements
Different kind of advertisements  Different kind of advertisements
Different kind of advertisements
ย 
assertive and aggressive commuication communication skills assignment
assertive and aggressive commuication  communication skills assignmentassertive and aggressive commuication  communication skills assignment
assertive and aggressive commuication communication skills assignment
ย 
New product development process with example
New product development process with example  New product development process with example
New product development process with example
ย 
Define and Explain Price adjustment strategies
Define and Explain Price adjustment strategies Define and Explain Price adjustment strategies
Define and Explain Price adjustment strategies
ย 
Amazing Facts about Pakistan is Best
Amazing Facts about Pakistan is BestAmazing Facts about Pakistan is Best
Amazing Facts about Pakistan is Best
ย 
Urdu Lines and Thoughts by Fahd khan Fahad ( MUhammad Fahad Bashir )
Urdu Lines and Thoughts by Fahd khan Fahad ( MUhammad Fahad Bashir )Urdu Lines and Thoughts by Fahd khan Fahad ( MUhammad Fahad Bashir )
Urdu Lines and Thoughts by Fahd khan Fahad ( MUhammad Fahad Bashir )
ย 
Building Right Relationship with customers and Changing Landscape of Marketing
Building Right Relationship with customers and Changing Landscape of MarketingBuilding Right Relationship with customers and Changing Landscape of Marketing
Building Right Relationship with customers and Changing Landscape of Marketing
ย 
Ancient history of south asia
Ancient history of south asiaAncient history of south asia
Ancient history of south asia
ย 

Recently uploaded

Spellings Wk 4 and Wk 5 for Grade 4 at CAPS
Spellings Wk 4 and Wk 5 for Grade 4 at CAPSSpellings Wk 4 and Wk 5 for Grade 4 at CAPS
Spellings Wk 4 and Wk 5 for Grade 4 at CAPS
AnaAcapella
ย 
24 ฤแป€ THAM KHแบขO KรŒ THI TUYแป‚N SINH Vร€O LแปšP 10 Mร”N TIแบพNG ANH Sแปž GIรO DแปคC HแบขI Dฦฏ...
24 ฤแป€ THAM KHแบขO KรŒ THI TUYแป‚N SINH Vร€O LแปšP 10 Mร”N TIแบพNG ANH Sแปž GIรO DแปคC HแบขI Dฦฏ...24 ฤแป€ THAM KHแบขO KรŒ THI TUYแป‚N SINH Vร€O LแปšP 10 Mร”N TIแบพNG ANH Sแปž GIรO DแปคC HแบขI Dฦฏ...
24 ฤแป€ THAM KHแบขO KรŒ THI TUYแป‚N SINH Vร€O LแปšP 10 Mร”N TIแบพNG ANH Sแปž GIรO DแปคC HแบขI Dฦฏ...
Nguyen Thanh Tu Collection
ย 
Tแป”NG HแปขP Hฦ N 100 ฤแป€ THI THแปฌ TแปT NGHIแป†P THPT TOรN 2024 - Tแปช CรC TRฦฏแปœNG, TRฦฏแปœNG...
Tแป”NG HแปขP Hฦ N 100 ฤแป€ THI THแปฌ TแปT NGHIแป†P THPT TOรN 2024 - Tแปช CรC TRฦฏแปœNG, TRฦฏแปœNG...Tแป”NG HแปขP Hฦ N 100 ฤแป€ THI THแปฌ TแปT NGHIแป†P THPT TOรN 2024 - Tแปช CรC TRฦฏแปœNG, TRฦฏแปœNG...
Tแป”NG HแปขP Hฦ N 100 ฤแป€ THI THแปฌ TแปT NGHIแป†P THPT TOรN 2024 - Tแปช CรC TRฦฏแปœNG, TRฦฏแปœNG...
Nguyen Thanh Tu Collection
ย 
ๆœƒ่€ƒ่‹ฑๆ–‡ๆœƒ่€ƒ่‹ฑๆ–‡ๆœƒ่€ƒ่‹ฑๆ–‡ๆœƒ่€ƒ่‹ฑๆ–‡ๆœƒ่€ƒ่‹ฑๆ–‡ๆœƒ่€ƒ่‹ฑๆ–‡ๆœƒ่€ƒ่‹ฑๆ–‡ๆœƒ่€ƒ่‹ฑๆ–‡ๆœƒ่€ƒ่‹ฑๆ–‡ๆœƒ่€ƒ่‹ฑๆ–‡ๆœƒ่€ƒ่‹ฑๆ–‡
ๆœƒ่€ƒ่‹ฑๆ–‡ๆœƒ่€ƒ่‹ฑๆ–‡ๆœƒ่€ƒ่‹ฑๆ–‡ๆœƒ่€ƒ่‹ฑๆ–‡ๆœƒ่€ƒ่‹ฑๆ–‡ๆœƒ่€ƒ่‹ฑๆ–‡ๆœƒ่€ƒ่‹ฑๆ–‡ๆœƒ่€ƒ่‹ฑๆ–‡ๆœƒ่€ƒ่‹ฑๆ–‡ๆœƒ่€ƒ่‹ฑๆ–‡ๆœƒ่€ƒ่‹ฑๆ–‡ๆœƒ่€ƒ่‹ฑๆ–‡ๆœƒ่€ƒ่‹ฑๆ–‡ๆœƒ่€ƒ่‹ฑๆ–‡ๆœƒ่€ƒ่‹ฑๆ–‡ๆœƒ่€ƒ่‹ฑๆ–‡ๆœƒ่€ƒ่‹ฑๆ–‡ๆœƒ่€ƒ่‹ฑๆ–‡ๆœƒ่€ƒ่‹ฑๆ–‡ๆœƒ่€ƒ่‹ฑๆ–‡ๆœƒ่€ƒ่‹ฑๆ–‡ๆœƒ่€ƒ่‹ฑๆ–‡
ๆœƒ่€ƒ่‹ฑๆ–‡ๆœƒ่€ƒ่‹ฑๆ–‡ๆœƒ่€ƒ่‹ฑๆ–‡ๆœƒ่€ƒ่‹ฑๆ–‡ๆœƒ่€ƒ่‹ฑๆ–‡ๆœƒ่€ƒ่‹ฑๆ–‡ๆœƒ่€ƒ่‹ฑๆ–‡ๆœƒ่€ƒ่‹ฑๆ–‡ๆœƒ่€ƒ่‹ฑๆ–‡ๆœƒ่€ƒ่‹ฑๆ–‡ๆœƒ่€ƒ่‹ฑๆ–‡
ไธญ ๅคฎ็คพ
ย 
ๆœƒ่€ƒ่‹ฑ่ฝๆœƒ่€ƒ่‹ฑ่ฝๆœƒ่€ƒ่‹ฑ่ฝๆœƒ่€ƒ่‹ฑ่ฝๆœƒ่€ƒ่‹ฑ่ฝๆœƒ่€ƒ่‹ฑ่ฝๆœƒ่€ƒ่‹ฑ่ฝๆœƒ่€ƒ่‹ฑ่ฝๆœƒ่€ƒ่‹ฑ่ฝๆœƒ่€ƒ่‹ฑ่ฝ
ๆœƒ่€ƒ่‹ฑ่ฝๆœƒ่€ƒ่‹ฑ่ฝๆœƒ่€ƒ่‹ฑ่ฝๆœƒ่€ƒ่‹ฑ่ฝๆœƒ่€ƒ่‹ฑ่ฝๆœƒ่€ƒ่‹ฑ่ฝๆœƒ่€ƒ่‹ฑ่ฝๆœƒ่€ƒ่‹ฑ่ฝๆœƒ่€ƒ่‹ฑ่ฝๆœƒ่€ƒ่‹ฑ่ฝๆœƒ่€ƒ่‹ฑ่ฝๆœƒ่€ƒ่‹ฑ่ฝๆœƒ่€ƒ่‹ฑ่ฝๆœƒ่€ƒ่‹ฑ่ฝๆœƒ่€ƒ่‹ฑ่ฝๆœƒ่€ƒ่‹ฑ่ฝๆœƒ่€ƒ่‹ฑ่ฝๆœƒ่€ƒ่‹ฑ่ฝๆœƒ่€ƒ่‹ฑ่ฝๆœƒ่€ƒ่‹ฑ่ฝ
ๆœƒ่€ƒ่‹ฑ่ฝๆœƒ่€ƒ่‹ฑ่ฝๆœƒ่€ƒ่‹ฑ่ฝๆœƒ่€ƒ่‹ฑ่ฝๆœƒ่€ƒ่‹ฑ่ฝๆœƒ่€ƒ่‹ฑ่ฝๆœƒ่€ƒ่‹ฑ่ฝๆœƒ่€ƒ่‹ฑ่ฝๆœƒ่€ƒ่‹ฑ่ฝๆœƒ่€ƒ่‹ฑ่ฝ
ไธญ ๅคฎ็คพ
ย 
Transparency, Recognition and the role of eSealing - Ildiko Mazar and Koen No...
Transparency, Recognition and the role of eSealing - Ildiko Mazar and Koen No...Transparency, Recognition and the role of eSealing - Ildiko Mazar and Koen No...
Transparency, Recognition and the role of eSealing - Ildiko Mazar and Koen No...
EADTU
ย 

Recently uploaded (20)

Spellings Wk 4 and Wk 5 for Grade 4 at CAPS
Spellings Wk 4 and Wk 5 for Grade 4 at CAPSSpellings Wk 4 and Wk 5 for Grade 4 at CAPS
Spellings Wk 4 and Wk 5 for Grade 4 at CAPS
ย 
Spring gala 2024 photo slideshow - Celebrating School-Community Partnerships
Spring gala 2024 photo slideshow - Celebrating School-Community PartnershipsSpring gala 2024 photo slideshow - Celebrating School-Community Partnerships
Spring gala 2024 photo slideshow - Celebrating School-Community Partnerships
ย 
Andreas Schleicher presents at the launch of What does child empowerment mean...
Andreas Schleicher presents at the launch of What does child empowerment mean...Andreas Schleicher presents at the launch of What does child empowerment mean...
Andreas Schleicher presents at the launch of What does child empowerment mean...
ย 
The Story of Village Palampur Class 9 Free Study Material PDF
The Story of Village Palampur Class 9 Free Study Material PDFThe Story of Village Palampur Class 9 Free Study Material PDF
The Story of Village Palampur Class 9 Free Study Material PDF
ย 
24 ฤแป€ THAM KHแบขO KรŒ THI TUYแป‚N SINH Vร€O LแปšP 10 Mร”N TIแบพNG ANH Sแปž GIรO DแปคC HแบขI Dฦฏ...
24 ฤแป€ THAM KHแบขO KรŒ THI TUYแป‚N SINH Vร€O LแปšP 10 Mร”N TIแบพNG ANH Sแปž GIรO DแปคC HแบขI Dฦฏ...24 ฤแป€ THAM KHแบขO KรŒ THI TUYแป‚N SINH Vร€O LแปšP 10 Mร”N TIแบพNG ANH Sแปž GIรO DแปคC HแบขI Dฦฏ...
24 ฤแป€ THAM KHแบขO KรŒ THI TUYแป‚N SINH Vร€O LแปšP 10 Mร”N TIแบพNG ANH Sแปž GIรO DแปคC HแบขI Dฦฏ...
ย 
DEMONSTRATION LESSON IN ENGLISH 4 MATATAG CURRICULUM
DEMONSTRATION LESSON IN ENGLISH 4 MATATAG CURRICULUMDEMONSTRATION LESSON IN ENGLISH 4 MATATAG CURRICULUM
DEMONSTRATION LESSON IN ENGLISH 4 MATATAG CURRICULUM
ย 
Tแป”NG HแปขP Hฦ N 100 ฤแป€ THI THแปฌ TแปT NGHIแป†P THPT TOรN 2024 - Tแปช CรC TRฦฏแปœNG, TRฦฏแปœNG...
Tแป”NG HแปขP Hฦ N 100 ฤแป€ THI THแปฌ TแปT NGHIแป†P THPT TOรN 2024 - Tแปช CรC TRฦฏแปœNG, TRฦฏแปœNG...Tแป”NG HแปขP Hฦ N 100 ฤแป€ THI THแปฌ TแปT NGHIแป†P THPT TOรN 2024 - Tแปช CรC TRฦฏแปœNG, TRฦฏแปœNG...
Tแป”NG HแปขP Hฦ N 100 ฤแป€ THI THแปฌ TแปT NGHIแป†P THPT TOรN 2024 - Tแปช CรC TRฦฏแปœNG, TRฦฏแปœNG...
ย 
8 Tips for Effective Working Capital Management
8 Tips for Effective Working Capital Management8 Tips for Effective Working Capital Management
8 Tips for Effective Working Capital Management
ย 
diagnosting testing bsc 2nd sem.pptx....
diagnosting testing bsc 2nd sem.pptx....diagnosting testing bsc 2nd sem.pptx....
diagnosting testing bsc 2nd sem.pptx....
ย 
OSCM Unit 2_Operations Processes & Systems
OSCM Unit 2_Operations Processes & SystemsOSCM Unit 2_Operations Processes & Systems
OSCM Unit 2_Operations Processes & Systems
ย 
UChicago CMSC 23320 - The Best Commit Messages of 2024
UChicago CMSC 23320 - The Best Commit Messages of 2024UChicago CMSC 23320 - The Best Commit Messages of 2024
UChicago CMSC 23320 - The Best Commit Messages of 2024
ย 
Graduate Outcomes Presentation Slides - English (v3).pptx
Graduate Outcomes Presentation Slides - English (v3).pptxGraduate Outcomes Presentation Slides - English (v3).pptx
Graduate Outcomes Presentation Slides - English (v3).pptx
ย 
Mattingly "AI and Prompt Design: LLMs with NER"
Mattingly "AI and Prompt Design: LLMs with NER"Mattingly "AI and Prompt Design: LLMs with NER"
Mattingly "AI and Prompt Design: LLMs with NER"
ย 
ๆœƒ่€ƒ่‹ฑๆ–‡ๆœƒ่€ƒ่‹ฑๆ–‡ๆœƒ่€ƒ่‹ฑๆ–‡ๆœƒ่€ƒ่‹ฑๆ–‡ๆœƒ่€ƒ่‹ฑๆ–‡ๆœƒ่€ƒ่‹ฑๆ–‡ๆœƒ่€ƒ่‹ฑๆ–‡ๆœƒ่€ƒ่‹ฑๆ–‡ๆœƒ่€ƒ่‹ฑๆ–‡ๆœƒ่€ƒ่‹ฑๆ–‡ๆœƒ่€ƒ่‹ฑๆ–‡
ๆœƒ่€ƒ่‹ฑๆ–‡ๆœƒ่€ƒ่‹ฑๆ–‡ๆœƒ่€ƒ่‹ฑๆ–‡ๆœƒ่€ƒ่‹ฑๆ–‡ๆœƒ่€ƒ่‹ฑๆ–‡ๆœƒ่€ƒ่‹ฑๆ–‡ๆœƒ่€ƒ่‹ฑๆ–‡ๆœƒ่€ƒ่‹ฑๆ–‡ๆœƒ่€ƒ่‹ฑๆ–‡ๆœƒ่€ƒ่‹ฑๆ–‡ๆœƒ่€ƒ่‹ฑๆ–‡ๆœƒ่€ƒ่‹ฑๆ–‡ๆœƒ่€ƒ่‹ฑๆ–‡ๆœƒ่€ƒ่‹ฑๆ–‡ๆœƒ่€ƒ่‹ฑๆ–‡ๆœƒ่€ƒ่‹ฑๆ–‡ๆœƒ่€ƒ่‹ฑๆ–‡ๆœƒ่€ƒ่‹ฑๆ–‡ๆœƒ่€ƒ่‹ฑๆ–‡ๆœƒ่€ƒ่‹ฑๆ–‡ๆœƒ่€ƒ่‹ฑๆ–‡ๆœƒ่€ƒ่‹ฑๆ–‡
ๆœƒ่€ƒ่‹ฑๆ–‡ๆœƒ่€ƒ่‹ฑๆ–‡ๆœƒ่€ƒ่‹ฑๆ–‡ๆœƒ่€ƒ่‹ฑๆ–‡ๆœƒ่€ƒ่‹ฑๆ–‡ๆœƒ่€ƒ่‹ฑๆ–‡ๆœƒ่€ƒ่‹ฑๆ–‡ๆœƒ่€ƒ่‹ฑๆ–‡ๆœƒ่€ƒ่‹ฑๆ–‡ๆœƒ่€ƒ่‹ฑๆ–‡ๆœƒ่€ƒ่‹ฑๆ–‡
ย 
ๆœƒ่€ƒ่‹ฑ่ฝๆœƒ่€ƒ่‹ฑ่ฝๆœƒ่€ƒ่‹ฑ่ฝๆœƒ่€ƒ่‹ฑ่ฝๆœƒ่€ƒ่‹ฑ่ฝๆœƒ่€ƒ่‹ฑ่ฝๆœƒ่€ƒ่‹ฑ่ฝๆœƒ่€ƒ่‹ฑ่ฝๆœƒ่€ƒ่‹ฑ่ฝๆœƒ่€ƒ่‹ฑ่ฝ
ๆœƒ่€ƒ่‹ฑ่ฝๆœƒ่€ƒ่‹ฑ่ฝๆœƒ่€ƒ่‹ฑ่ฝๆœƒ่€ƒ่‹ฑ่ฝๆœƒ่€ƒ่‹ฑ่ฝๆœƒ่€ƒ่‹ฑ่ฝๆœƒ่€ƒ่‹ฑ่ฝๆœƒ่€ƒ่‹ฑ่ฝๆœƒ่€ƒ่‹ฑ่ฝๆœƒ่€ƒ่‹ฑ่ฝๆœƒ่€ƒ่‹ฑ่ฝๆœƒ่€ƒ่‹ฑ่ฝๆœƒ่€ƒ่‹ฑ่ฝๆœƒ่€ƒ่‹ฑ่ฝๆœƒ่€ƒ่‹ฑ่ฝๆœƒ่€ƒ่‹ฑ่ฝๆœƒ่€ƒ่‹ฑ่ฝๆœƒ่€ƒ่‹ฑ่ฝๆœƒ่€ƒ่‹ฑ่ฝๆœƒ่€ƒ่‹ฑ่ฝ
ๆœƒ่€ƒ่‹ฑ่ฝๆœƒ่€ƒ่‹ฑ่ฝๆœƒ่€ƒ่‹ฑ่ฝๆœƒ่€ƒ่‹ฑ่ฝๆœƒ่€ƒ่‹ฑ่ฝๆœƒ่€ƒ่‹ฑ่ฝๆœƒ่€ƒ่‹ฑ่ฝๆœƒ่€ƒ่‹ฑ่ฝๆœƒ่€ƒ่‹ฑ่ฝๆœƒ่€ƒ่‹ฑ่ฝ
ย 
Transparency, Recognition and the role of eSealing - Ildiko Mazar and Koen No...
Transparency, Recognition and the role of eSealing - Ildiko Mazar and Koen No...Transparency, Recognition and the role of eSealing - Ildiko Mazar and Koen No...
Transparency, Recognition and the role of eSealing - Ildiko Mazar and Koen No...
ย 
Sternal Fractures & Dislocations - EMGuidewire Radiology Reading Room
Sternal Fractures & Dislocations - EMGuidewire Radiology Reading RoomSternal Fractures & Dislocations - EMGuidewire Radiology Reading Room
Sternal Fractures & Dislocations - EMGuidewire Radiology Reading Room
ย 
Mattingly "AI & Prompt Design: Named Entity Recognition"
Mattingly "AI & Prompt Design: Named Entity Recognition"Mattingly "AI & Prompt Design: Named Entity Recognition"
Mattingly "AI & Prompt Design: Named Entity Recognition"
ย 
AIM of Education-Teachers Training-2024.ppt
AIM of Education-Teachers Training-2024.pptAIM of Education-Teachers Training-2024.ppt
AIM of Education-Teachers Training-2024.ppt
ย 
An overview of the various scriptures in Hinduism
An overview of the various scriptures in HinduismAn overview of the various scriptures in Hinduism
An overview of the various scriptures in Hinduism
ย 

FSC part1 Math MCQS -

  • 1. MATHEMATICS 11 MULTIPLE CHOICE QUESTIONS WITH ANSWERS MUHAMMAD SALMAN SHERAZI M.Sc(Math) , B.ed 03337727666/03067856232
  • 2. 2 | P a g e Contents UNIT # Title Page # 1 Number System 3 2 Sets, Functions and Groups 4 3 Matrices and Determinants 6 4 Quadratic Equations 8 5 Partial Fractions 9 6 Sequence and Series 10 7 Permutation, Combinations and Probability 11 8 Mathematical Induction and Binomial Theorem 13 9 Fundamental of Trigonometry 13 10 Trigonometric Identities 15 11 Trigonometric Functions and their Graphs 17 12 Application of Trigonometry 18 13 Inverse Trigonometric Functions 19 14 Solution of Trigonometric Equations 21
  • 3. 3 | P a g e UNIT # 01 Number Systems Each question has four possible answer. Tick the correct answer. 1. For any complex number ๐’›, it is always true that |๐’›| is equal to: (a) |๐‘ง| (b) | โˆ’ ๐‘ง| (c) | โˆ’ ๐‘ง| (d) โœ” all of these 2. The numbers which can be written in the form of ๐’‘ ๐’’ , ๐’‘, ๐’’ โˆˆ ๐’ , ๐’’ โ‰  ๐ŸŽ are : (a) โœ”Rational number (b) Irrational number (c) Complex number (d) Whole number 3. A decimal which has a finite numbers of digits in its decimal part is called______ decimal. (a) โœ”Terminating (b) Non-Terminating (c) Recurring (d) Non recurring 4. ๐Ÿ“. ๐Ÿ‘๐Ÿ‘๐Ÿ‘ โ€ฆ. Is (a) Rational (b) Irrational (c) an integer (d) a prime number 5. ๐… is (a) Rational (b) โœ” Irrational (c) Natural number (d) None 6. ๐Ÿ๐Ÿ ๐Ÿ• is (a) โœ”Rational (b) Irrational (c) an integer (d) a whole number 7. Multiplicative inverse of โ€ฒ๐ŸŽโ€ฒ is (a) 0 (b) any real number (c) โœ” not defined (d) 1 8. If ๐’‚ is any non-zero real number, then multiplicative inverse is (a) โ€“ ๐‘Ž (b) โœ” 1 ๐‘Ž (c) โˆ’ 1 ๐‘Ž (d) not defined 9. For all ๐’‚ โˆˆ ๐‘น , ๐’‚ = ๐’‚ is โ€ฆ. Property. (a) โœ”Reflexive (b) Symmetric (c) Transitive (d) Trichotomy 10. For all ๐’‚, ๐’ƒ โˆˆ ๐‘น , ๐’‚ = ๐’ƒ โŸน ๐’ƒ = ๐’‚ is calledโ€ฆ.. property. (a) Reflexive (b) โœ” Symmetric (c) Transitive (d) Trichotomy 11. Golden rule of fraction is that for ๐’Œ โ‰  ๐ŸŽ, ๐’‚ ๐’ƒ = (a) โœ” ๐‘˜๐‘Ž ๐‘˜๐‘ (b) ๐‘Ž๐‘ ๐‘™ (c) ๐‘˜๐‘Ž ๐‘ (d) ๐‘˜๐‘ ๐‘ 12. The set {๐Ÿ, โˆ’๐Ÿ} possesses closure property ๐’˜. ๐’“. ๐’• (a) โ€ฒ + โ€ฒ (b) โœ” โ€ฒ ร— โ€ฒ (c) โ€ฒ รท โ€ฒ (d) โ€ฒ โˆ’ โ€ฒ 13. If ๐’‚ < ๐‘ then (a) ๐‘Ž < ๐‘ (b) 1 ๐‘Ž < 1 ๐‘ (c) โœ” 1 ๐‘Ž > 1 ๐‘ (d) ๐‘Ž โˆ’ ๐‘ > 0 14. The additive identity in set of complex number is (a) โœ”(0,0) (b) (0,1) (c) (1,0) (d) (1,1) 15. The multiplicative identity of complex number is (a) (0,0) (b) (0,1) (c) โœ” (1,0) (d) (1,1) 16. The modulus of ๐’› = ๐’‚ + ๐’Š๐’ƒ is (a) โˆš๐‘Ž + ๐‘ (b) โœ”โˆš๐‘Ž2 + ๐‘2 (c) ๐‘Ž โˆ’ ๐‘ (d) โˆš๐‘Ž2 โˆ’ ๐‘2 17. ๐’Š๐Ÿ๐Ÿ‘ equals: (a) โœ”๐‘– (b) โ€“ ๐‘– (c) 1 (d) -1 18. The multiplicative inverse of (๐Ÿ’, โˆ’๐Ÿ•) is: (a) (โˆ’ 4 65 , โˆ’ 7 65 ) (b) (โˆ’ 4 65 , 7 65 ) (c) ( 4 65 , โˆ’ 7 65 ) (d) โœ”( 4 65 , 7 65 ) 19. (๐ŸŽ, ๐Ÿ‘)(๐ŸŽ, ๐Ÿ“) = (a) 15 (b) โœ”-15 (c) โˆ’8๐‘– (d) 8๐‘– 20. (โˆ’๐Ÿ)โˆ’ ๐Ÿ๐Ÿ ๐Ÿ = (a) ๐‘– (b) โœ” โ€“ ๐‘– (c) 1 (d) -1 21. โˆš๐Ÿ‘ is __________ (a) Rational (b) โœ” Irrational (c) Integer (d) Prime 22. Product โˆšโˆ’๐Ÿ ร— โˆšโˆ’๐Ÿ is equal to _____ (a) -2 (b) โœ”2 (c) 0 (d) 4 23. The imaginary part of the complex number (๐’ƒ , ๐’‚) is _________ (a) โœ”๐‘Ž (b) ๐‘ (c) ๐‘–๐‘Ž (d) None of these 24. If ๐’› = โˆ’๐Ÿ โˆ’ ๐’Š then ๐’› =______ (a) (โˆ’1, โˆ’1) (b) โœ”(โˆ’1,1) (c) (1, โˆ’1) (d) (1,1) 25. The property ๐Ÿ•. ๐Ÿ– + (โˆ’๐Ÿ•. ๐Ÿ–) = ๐ŸŽ is______ (a) Commutative (b)โ€˜.โ€™ inverse (c) โœ” โ€˜ + โ€™ inverse (d) Associative
  • 4. 4 | P a g e 26. If ๐’™ = ๐ŸŽ then multiplicative inverse of ๐’™ is (a) 0 (b) 1 (c) 1 ๐‘ฅ (d) โœ” None of these 27. (โˆ’๐’Š)๐Ÿ๐Ÿ“ =_______ (a) 1 (b) -1 (c) โœ” ๐‘– (d) โ€“ ๐‘– 28. If ๐’›1 and ๐’›2 are complex numbers then |๐’›1+๐’›2| is _______ (a) <|๐‘ง1+๐‘ง2| (b) โœ” โ‰ค|๐‘ง1 |+|๐‘ง2| (c) โ‰ฅ |๐‘ง1+๐‘ง2| (d) None of these 29. A recurring decimal represents_____ (a) Real number (b) Natural number (c) โœ”Rational number (d) None of these 30. Every recurring decimal is (a) a rational number (b) an irrational number (c) a prime number (d) a whole number 31. Imaginary part of (โˆ’๐Ÿ + ๐Ÿ‘๐’Š๐Ÿ‘ ) is (a) -2 (b) 9 (c) 26 (d) โœ” -3 32. The product of two conjugate complex numbers is (a) โœ”A real number (b) an imaginary number (c) may be an irrational number (d) not defined 33. (๐ŸŽ, ๐Ÿ)๐Ÿ‘ is equal to: (a) โœ”1 (b) -1 (c) ๐‘– (d) โ€“ ๐‘– 34. If ๐’ is an even integer, then (๐’Š)๐’ is equal to: (a) โœ”๐‘– (b) โ€“ ๐‘– (c) ยฑ1 (d) 1 35. Factors of ๐Ÿ‘(๐’™๐Ÿ + ๐’š๐Ÿ ) are: (a) 3(๐‘ฅ + ๐‘ฆ)(๐‘ฅ โˆ’ ๐‘ฆ) (b) โœ” 3(๐‘ฅ + ๐‘–๐‘ฆ)(๐‘ฅ โˆ’ ๐‘–๐‘ฆ) (c) โˆš3(๐‘ฅ + ๐‘–๐‘ฆ)(๐‘ฅ โˆ’ ๐‘–๐‘ฆ) (d) None of these 36. Real part of ๐Ÿ+๐’Š ๐’Š is: (a) 1 (b) โœ”- 2 (c) -1 (d) 1 2 UNIT # 02 Sets, Functions and Groups Each question has four possible answer. Tick the correct answer. 1. If ๐’™ โˆˆ ๐‘ณ โˆช ๐‘ด then (a) ๐‘ฅ โˆ‰ ๐ฟ ๐‘œ๐‘Ÿ ๐‘ฅ โˆ‰ ๐‘€ (b) ๐‘ฅ โˆ‰ ๐ฟ ๐‘œ๐‘Ÿ ๐‘ฅ โˆˆ ๐‘€ (c) ๐‘ฅ โˆˆ ๐ฟ ๐‘œ๐‘Ÿ ๐‘ฅ โˆ‰ ๐‘€ (d) โœ”๐‘ฅ โˆˆ ๐ฟ ๐‘œ๐‘Ÿ ๐‘ฅ โˆˆ ๐‘€ 2. Total number of subsets that can be formed from the set {๐’™, ๐’š, ๐’›} is (a) 1 (b) โœ” 8 (c) 5 (d)2 3. If ๐’™ โˆˆ ๐‘ฉโ€ฒ = ๐‘ผ โˆ’ ๐‘ฉ then (a) ๐‘ฅ โˆˆ ๐ต ๐‘Ž๐‘›๐‘‘ ๐‘ฅ โˆˆ ๐‘ˆ (b) โœ”๐‘ฅ โˆ‰ ๐ต ๐‘Ž๐‘›๐‘‘ ๐‘ฅ โˆˆ ๐‘ˆ (c) ๐‘ฅ โˆ‰ ๐ต ๐‘Ž๐‘›๐‘‘ ๐‘ฅ โˆ‰ ๐‘ˆ (d)๐‘ฅ โˆˆ ๐ต ๐‘Ž๐‘›๐‘‘ ๐‘ฅ โˆ‰ ๐‘ˆ 4. ๐‘ณ โˆช ๐‘ด = ๐‘ณ โˆฉ ๐‘ดthen ๐‘ณ is equal to (a) โœ”๐‘€ (b) ๐ฟ (c) ๐œ‘ (d) ๐‘€โ€ฒ 5. A set is a collection of objects which are (a) Well defined (b) โœ”Well defined and distinct (c) identical (d) not defined 6. The set of odd numbers between 1 and 9 are (a) {1,3,5,7} (b) {3,5,7,9} (c) {1,3,5,7,9} (d) โœ”{3,5,7} 7. Every recurring non terminating decimal represents (a) โœ”๐‘„ (b) ๐‘„โ€ฒ (c) ๐‘… (d) None 8. A diagram which represents a set is called______ (a) โœ”Vennโ€™s (b) Argand (c) Plane (d) None 9. ๐‘น โˆ’ {๐ŸŽ} is a group ๐’˜. ๐’“. ๐’• the binary operation (a) + (b) โœ” ร— (c) รท (d)โˆ’ 10. In a proposition if ๐’‘ โ†’ ๐’’ ๐’•๐’‰๐’†๐’ ๐’’ โ†’ ๐’‘ is called (a) Inverse of ๐‘ โ†’ ๐‘ž (b) โœ” converse of ๐‘ โ†’ ๐‘ž (c) contrapositive of ๐‘ โ†’ ๐‘ž(d) None 11. If ๐‘จ and ๐‘ฉ are disjoint sets , then ๐‘จ โˆฉ ๐‘ฉ = (a) 0 (b) 1 (c) 2 (d) โœ” ๐œ‘ 12. ๐’‘ โ†’ ๐’’ is called converse of (a) ~๐‘๐‘ž (b) โœ” ๐‘ โ†’ ๐‘ž (c) ๐‘ž โ†’ ๐‘ž (d) ~๐‘ž โ†’ ๐‘ 13. If ๐‘จ โŠ† ๐‘ฉ then ๐’(๐‘จ โˆฉ ๐‘ฉ) = (a) โœ”๐‘›(๐ด) (b) ๐‘›(๐ต) (c) ๐‘›(๐ด) + ๐‘›(๐ต) (d) ๐‘›(๐ด). ๐‘›(๐ต) 14. Inverse of any element of a group is: (a) Not unique (b) โœ”unique (c) has many inverses (d) none of these 15. Every function is: (a) โœ”Relation (b) inverse function (c) one to one (d) none of these 16. The number of all subsets of a set having three elements is: (a) 4 (b) 6 (c) โœ” 8 (d) 10
  • 5. 5 | P a g e 17. The graph of linear function is : (a) Circle ( b) โœ”straight line (c) parabola (d) triangle 18. Identity element in (๐‘ช, . ) is: (a) (0,0) (b) (0,1) (c) โœ” (1,0) (d) (1,1) 19. Squaring a number is a: (a) โœ”Unary operation (b) binary operation (c) relation (d) Function 20. Which of the following is true: (a) ๐‘ โŠ‚ ๐‘ (b) ๐‘ โŠ‚ ๐‘„ (c) ๐‘„ โŠ‚ ๐‘… (d) โœ”all of these 21. Set of integers is a group ๐’˜. ๐’“. ๐’• (a) โœ”Addition (b) multiplication (c) subtraction (d) division 22. If ๐‘จ = ๐‹ then ๐‘ท(๐‘จ) = (a) Empty set (b) {0} (c) โœ” {๐œ‘} (d) none of these 23. If ๐‘จ and ๐‘ฉ are disjoint sets then : (a) โœ”๐ด โˆฉ ๐ต = ๐œ‘ (b) ๐ด โˆฉ ๐ต โ‰  ๐œ‘ (c) ๐ด โŠ‚ ๐ต (d) ๐ด โˆ’ ๐ต = ๐œ‘ 24. The set of non-zero real numbers ๐’˜. ๐’“. ๐’• multiplication is: (a) Groupied (b) Semi-group (c) Monoied (d) โœ” Group 25. Which of the following is not a binary operation : (a) + (b) ร— (c) โœ” โˆš (d) โ€“ 26. If ๐’š = โˆš๐’™ , ๐’™ โ‰ฅ ๐ŸŽ is a function, then its inverse is: (a) A line (b) a parabola (c) a point (d) โœ” not a function 27. An onto function is also called: (a) Injective (b) โœ” Surjective (c) Bijective (d) Inverse 28. A (๐Ÿ โˆ’ ๐Ÿ) function is also called: (a) โœ”Injective (b) Surjective (c) Bijective (d) Inverse 29. For the propositions ๐’‘ and ๐’’, (๐’‘ โˆง ๐’’) โ†’ ๐’‘ is: (a) โœ”Tautology (b) Absurdity (c) contingency (d) None of these 30. For the propositions ๐’‘ and ๐’’ , ๐’‘ โ†’ (๐’‘ โˆจ ๐’’) is: (a) โœ”Tautology (b) Absurdity (c) Contingency (d) None of these 31. If set ๐‘จ has 2 elements and ๐‘ฉ has 4 elements , then number of elements in ๐‘จ ร— ๐‘ฉ is : (a) 6 (b) โœ” 8 (c) 16 (d) None of these 32. Inverse of a line is : (a) โœ”A line (b) a parabola (c) a point (d) not defined 33. The function ๐’‡ = {(๐’™, ๐’š), ๐’š = ๐’™} is : (a) โœ”Identity function (b) Null function (c) not a function (d) similar function 34. If ๐’š = โˆš๐’™, ๐’™ โ‰ฅ ๐ŸŽ is a function, then its inverse is : (a) A line (b) a parabola (c) a point (d) โœ” not defined 35. Truth set of a tautology is (a) โœ”Universal set (b) ๐œ‘ (c) True (d) False 36. A compound proposition which is always true is called: (a) โœ”Tautology (b) contradiction (c) absurdity (d) contingency 37. A compound proposition which is always neither true nor false is called: (a) Tautology (b) contradiction (c) absurdity (d) โœ” contingency 38. A compound proposition which is always wrong is called: (a) Tautology (b) โœ” contradiction (c) absurdity (d) contingency 39. If ๐‘บ = { } , then order of set ๐‘บ is: (a) โœ”0 (b) 1 (c) Infinite set (d) not defined 40. The symbol which is used to denote negation of a proposition is (a) โœ”~ (b) โ†’ (c) โˆง (d) โˆจ 41. Which of the following is true: (a) ๐œ‘ โˆ’ ๐ด = ๐œ‘ (b) ๐ด โˆช ๐ด = ๐ด (c) ๐ด โˆฉ ๐ด = ๐ด (d) โœ”All of these 42. Which of the following is true: (a) ๐ด โˆช ๐œ‘ = ๐ด (b) ๐ด โˆฉ ๐œ‘ = ๐œ‘ (c) ๐ด โˆ’ ๐œ‘ = ๐ด (d) โœ”All of these 43. If ๐‘ฉ โŠ† ๐‘จ, then ๐’(๐‘ฉ โˆ’ ๐‘จ) is equal to: (a) ๐‘›(๐ด) (b) ๐‘›(๐ต) (c) ๐‘›(๐ด โˆฉ ๐ต) (d) โœ” 0
  • 6. 6 | P a g e UNIT # 03 Matrices and Determinants Each question has four possible answer. Tick the correct answer. 1. A rectangular array of numbers enclosed by a square brackets is called: (a) โœ”Matrix (b) Row (c) Column (d) Determinant 2. The horizontal lines of numbers in a matrix are called: (a) Columns (b) โœ” Rows (c) Column matrix (d) Row matrix 3. The vertical lines of numbers in a matrix are called: (a) โœ”Columns (b) Rows (c) Column matrix (d) Row matrix 4. If a matrix A has ๐’Ž rows and ๐’ columns , then order of A is : (a) โœ”๐‘š ร— ๐‘› (b) ๐‘› ร— ๐‘š (c) ๐‘š + ๐‘› (d) ๐‘š๐‘› 5. The element ๐’‚๐’Š๐’‹ of any matrix A is present in: (a) โœ”๐‘–๐‘กโ„Ž row and ๐‘—๐‘กโ„Ž column (b)๐‘–๐‘กโ„Ž column and ๐‘—๐‘กโ„Ž row (c) (๐‘– + ๐‘—)๐‘กโ„Ž row and column (d) (๐‘– + ๐‘—)๐‘กโ„Ž row and column 6. A matrix A is called real if all ๐’‚๐’Š๐’‹ are (a) โœ”Real numbers (b) complex numbers (c) 0 (d) 1 7. If a matrix A has only one row then it is called: (a) โœ”Row vector (b) Column vector (c) square matrix (d) Rectangle matrix 8. If a matrix A has only one column then it is called: (a) Row vector (b) โœ” Column vector (c) square matrix (d) Rectangle matrix 9. If a matrix A has same number of rows and columns then A is called: (a) Row vector (b) Column vector (c) โœ”square matrix (d) Rectangular matrix 10. If a matrix A has different number of rows and columns then A is called: (a) Row vector (b) Column vector (c) square matrix (d) โœ” Rectangular matrix 11. For the square matrix ๐‘จ = [๐’‚๐’Š๐’‹]๐’ร—๐’ then ๐’‚๐Ÿ๐Ÿ, ๐’‚๐Ÿ๐Ÿ, ๐’‚๐Ÿ‘๐Ÿ‘โ€ฆ๐’‚๐’๐’ are: (a) โœ”Main diagonal (b) primary diagonal (c) proceding diagonal (d) secondary diagonal 12. For a square matrix ๐‘จ = [๐’‚๐’Š๐’‹]if all ๐’‚๐’Š๐’‹ = ๐ŸŽ, ๐’Š โ‰  ๐’‹ and all ๐’‚๐’Š๐’‹ = ๐’Œ(๐’๐’๐’ โˆ’ ๐’›๐’†๐’“๐’) for ๐’Š = ๐’‹ then A is called: (a) Diagonal matrix (b) โœ” Scalar Matrix (c) Scalar Matrix (d) Null matrix 13. For a square matrix ๐‘จ = [๐’‚๐’Š๐’‹]if all ๐’‚๐’Š๐’‹ = ๐ŸŽ, ๐’Š โ‰  ๐’‹ and at least ๐’‚๐’Š๐’‹ โ‰  ๐ŸŽ, ๐’Š = ๐’‹ then A is called: (a) โœ”Diagonal matrix (b) Scalar Matrix (c) Scalar Matrix (d) Null matrix 14. The matrix [๐Ÿ•] is : (a) Square matrix (b) row matrix (c) column matrix (d) โœ” all of these 15. If a matrix of order ๐’Ž ร— ๐’ then the matrix of order ๐’ ร— ๐’Ž is : (a) โœ”Transpose of A (b) Inverse of A (c) Main diagonal of A (d) Adjoint of A 16. For any two matrices A and B ,(๐‘จ + ๐‘ฉ)๐’• = (a) ๐ต๐‘ก ๐ด๐‘ก (b) ๐ด๐‘ก ๐ต๐‘ก (c) โœ”๐ด๐‘ก + ๐ต๐‘ก (d) ๐ต๐‘ก + ๐ด๐‘ก 17. Let A be a matrix and ๐’ is an integer then ๐‘จ + ๐‘จ + ๐‘จ + โ‹ฏ to ๐’ terms (a) โœ”๐‘›๐ด (b) ๐‘› + ๐ด (c) ๐‘› โˆ’ ๐ด (d) ๐ด๐‘› 18. If order of ๐‘จ is ๐’Ž ร— ๐’ and order of B is ๐’ ร— ๐’’ then order of AB is (a) ๐‘š ร— ๐‘› (b) ๐‘› ร— ๐‘š (d) ๐‘› ร— ๐‘ž (d) โœ”๐‘š ร— ๐‘ž 19. If ๐’‚๐’‹๐’… ๐‘จ = [ โˆ’๐Ÿ โˆ’๐Ÿ ๐Ÿ‘ ๐Ÿ’ ] then matrix ๐‘จ is (a) [ โˆ’1 โˆ’2 4 3 ] (b) [ 4 2 3 โˆ’1 ] (c) [ โˆ’4 3 โˆ’2 โˆ’1 ] (d) โœ” [ 4 2 โˆ’3 โˆ’1 ] 20. If ๐‘จ is non-singular matrix then ๐‘จโˆ’๐Ÿ = (a) โœ” 1 |๐ด| ๐‘Ž๐‘‘๐‘—๐ด (b) โˆ’ 1 |๐ด| ๐‘Ž๐‘‘๐‘—๐ด (c) |๐ด| ๐‘Ž๐‘‘๐‘—๐ด (d) 1 |๐ด|๐‘Ž๐‘‘๐‘—๐ด 21. If ๐‘จ๐‘ฟ = ๐‘ฉ then ๐‘ฟ is equal to: (a) ๐ด๐ต (b) โœ” ๐ดโˆ’1 ๐ต (c) ๐ตโˆ’1 ๐ด (d) ๐ต๐ด 22. Inverse of a matrix exists if it is: (a) Singular (b) Null (c) Rectangular (d) โœ” Non-singular 23. Which of the property does not hold matrix multiplication? (a) Associative (b) โœ” Commutative (c) Closure (d) Inverse 24. For any matrix A , it is always true that (a) ๐ด = ๐ด๐‘ก (b) โ€“ ๐ด = ๐ดฬ… (c) โœ”|๐ด| = |๐ด๐‘ก | (d) ๐ดโˆ’1 = 1 ๐ด 25. If all entries of a square matrix of order ๐Ÿ‘ is multiplied by ๐’Œ, then value of |๐’Œ๐‘จ| is equal to: (a) ๐‘˜|๐ด| (b) ๐‘˜2 |๐ด| (c) โœ”๐‘˜3 |๐ด| (d) |๐ด|
  • 7. 7 | P a g e 26. For a non-singular matrix it is true that : (a) (๐ดโˆ’1)โˆ’1 = ๐ด (b) (๐ด๐‘ก)๐‘ก = ๐ด (c) ๐ดฬฟ = ๐ด (d) โœ”all of these 27. For any non-singular matrices A and B it is true that: (a) (๐ด๐ต)โˆ’1 = ๐ตโˆ’1 ๐ดโˆ’1 (b) (๐ด๐ต)๐‘ก = ๐ต๐‘ก ๐ด๐‘ก (c) ๐ด๐ต โ‰  ๐ต๐ด (d) โœ” all of these 28. A square matrix ๐‘จ = [๐’‚๐’Š๐’‹] for which ๐’‚๐’Š๐’‹ = ๐ŸŽ, ๐’Š > ๐‘— then A is called: (a) โœ”Upper triangular (b) Lower triangular (c) Symmetric (d) Hermitian 29. A square matrix ๐‘จ = [๐’‚๐’Š๐’‹] for which ๐’‚๐’Š๐’‹ = ๐ŸŽ, ๐’Š < ๐‘— then A is called: (a) Upper triangular (b) โœ”Lower triangular (c) Symmetric (d) Hermitian 30. Any matrix A is called singular if: (a) โœ”|๐ด| = 0 (b) |๐ด| โ‰  0 (c) ๐ด๐‘ก = ๐ด (d) ๐ด๐ดโˆ’1 = ๐ผ 31. Which of the following Sets is a field. (a) R (b) Q (c) C (d) โœ”all of these 32. Which of the following Sets is not a field. (a) R (b) Q (c) C (d) โœ”Z 33. A square matrix A is symmetric if: (a) โœ”๐ด๐‘ก = ๐ด (b) ๐ด๐‘ก = โˆ’๐ด (c) (๐ด) ๐‘ก = ๐ด (d) (๐ด) ๐‘ก = โˆ’๐ด 34. A square matrix A is skew symmetric if: (a) ๐ด๐‘ก = ๐ด (b) โœ” ๐ด๐‘ก = โˆ’๐ด (c) (๐ด) ๐‘ก = ๐ด (d) (๐ด) ๐‘ก = โˆ’๐ด 35. A square matrix A is Hermitian if: (a) ๐ด๐‘ก = ๐ด (b) ๐ด๐‘ก = โˆ’๐ด (c) โœ”(๐ด) ๐‘ก = ๐ด (d) (๐ด) ๐‘ก = โˆ’๐ด 36. A square matrix A is skew- Hermitian if: (a) ๐ด๐‘ก = ๐ด (b) ๐ด๐‘ก = โˆ’๐ด (c) (๐ด) ๐‘ก = ๐ด (d) โœ”(๐ด) ๐‘ก = โˆ’๐ด 37. The main diagonal elements of a skew symmetric matrix must be: (a) 1 (b) โœ” 0 (c) any non-zero number (d) any complex number 38. The main diagonal elements of a skew hermitian matrix must be: (a) 1 (b) โœ” 0 (c) any non-zero number (d) any complex number 39. In echelon form of matrix, the first non zero entry is called: (a) โœ”Leading entry (b) first entry (c) preceding entry (d) Diagonal entry 40. The additive inverse of a matrix exist only if it is: (a) Singular (b) non singular (c) null matrix (d) โœ” any matrix of order ๐‘š ร— ๐‘› 41. The multiplicative inverse of a matrix exist only if it is: (a) Singular (b) โœ” non singular (c) null matrix (d) any matrix of order ๐‘š ร— ๐‘› 42. The number of non zero rows in echelon form of a matrix is called: (a) Order of matrix (b) Rank of matrix (c) leading (d) leading row 43. If A is any square matrix then ๐‘จ + ๐‘จ๐’• is a (a) โœ”Symmetric (b) skew symmetric (c) hermitian (d) skew hermitian 44. If A is any square matrix then ๐‘จ โˆ’ ๐‘จ๐’• is a (a) Symmetric (b) โœ”skew symmetric (c) hermitian (d) skew hermitian 45. If A is any square matrix then ๐‘จ + (๐‘จ) ๐’• is a (a) Symmetric (b) skew symmetric (c) โœ” hermitian (d) skew hermitian 46. If A is any square matrix then ๐‘จ + (๐‘จ) ๐’• is a (a) Symmetric (b) skew symmetric (c) hermitian (d) โœ” skew hermitian 47. If A is symmetric (Skew symmetric), then ๐‘จ๐Ÿ must be (a) Singular (b) non singular (c) โœ”symmetric (d) non trivial solution 48. In a homogeneous system of linear equations , the solution (0,0,0) is: (a) โœ”Trivial solution (b) non trivial solution (c) exact solution (d) anti symmetric 49. If ๐‘จ๐‘ฟ = ๐‘ถ then ๐‘ฟ = (a) ๐ผ (b) โœ” ๐‘‚ (c) ๐ดโˆ’1 (d) Not possible 50. If the system of linear equations have no solution at all, then it is called a/an (a) Consistent system (b)โœ” Inconsistent system (c) Trivial System (d) Non Trivial System 51. The value of ๐€ for which the system ๐’™ + ๐Ÿ๐’š = ๐Ÿ’; ๐Ÿ๐’™ + ๐€๐’š = โˆ’๐Ÿ‘ does not possess the unique solution (a) โœ”4 (b) -4 (c) ยฑ4 (d) any real number 52. If the system ๐’™ + ๐Ÿ๐’š = ๐ŸŽ; ๐Ÿ๐’™ + ๐€๐’š = ๐ŸŽ has non-trivial solution, then ๐€ is: (a) โœ”4 (b) -4 (c) ยฑ4 (d) any real number 53. The inverse of unit matrix is: (a) โœ”Unit (b) Singular (c) Skew Symmetric (d) rectangular
  • 8. 8 | P a g e 54. Transpose of a row matrix is: (a) Diagonal matrix (b) zero matrix (c) โœ” column matrix (d) scalar matrix 55. If | ๐’™ ๐Ÿ’ ๐Ÿ“ ๐Ÿ๐ŸŽ | = ๐ŸŽ โ‡’ ๐’™ equals (a) โœ”2 (b) 4 (c) 6 (d) 8 UNIT # 04 Quadratic Equations Each question has four possible answer. Tick the correct answer. 1. The equation ๐’‚๐’™๐Ÿ + ๐’ƒ๐’™ + ๐’„ = ๐ŸŽ will be quadratic if: (a) ๐‘Ž = 0, ๐‘ โ‰  0 (b) โœ” ๐‘Ž โ‰  0 (c) ๐‘Ž = ๐‘ = 0 (d) ๐‘ = any real number 2. Solution set of the equation ๐’™๐Ÿ โˆ’ ๐Ÿ’๐’™ + ๐Ÿ’ = ๐ŸŽ is: (a) {2, โˆ’2} (b) โœ” {2} (c) {โˆ’2} (d) {4, โˆ’4} 3. The quadratic formula for solving the equation ๐’‚๐’™๐Ÿ + ๐’ƒ๐’™ + ๐’„ = ๐ŸŽ; ๐’‚ โ‰  ๐ŸŽ is (a) โœ”๐‘ฅ = โˆ’๐‘ยฑโˆš๐‘2 โˆ’4๐‘Ž๐‘ 2๐‘Ž (b) ๐‘ฅ = โˆ’๐‘ยฑโˆš๐‘Ž2โˆ’4๐‘Ž๐‘ 2๐‘ (c) ๐‘ฅ = โˆ’๐‘Žยฑโˆš๐‘2โˆ’4๐‘Ž๐‘ 2 (d) None of these 4. To convert ๐’‚๐’™๐Ÿ๐’ + ๐’ƒ๐’™๐’ + ๐’„ = ๐ŸŽ(๐’‚ โ‰  ๐ŸŽ) into quadratic form , the correct substitution is: (a) โœ”๐‘ฆ = ๐‘ฅ๐‘› (b) ๐‘ฅ = ๐‘ฆ๐‘› (c) ๐‘ฆ = ๐‘ฅโˆ’๐‘› (d) ๐‘ฆ = 1 ๐‘ฅ 5. The equation in which variable occurs in exponent , called: (a) Exponential function (b) Quadratic equation (c) Reciprocal equation (d) โœ”Exponential equation 6. To convert ๐Ÿ’๐Ÿ+๐’™ + ๐Ÿ’๐Ÿโˆ’๐’™ = ๐Ÿ๐ŸŽ into quadratic , the substitution is: (a) ๐‘ฆ = ๐‘ฅ1โˆ’๐‘ฅ (b) ๐‘ฆ = 41+๐‘ฅ (c) ๐‘ฆ = 4๐‘ฅ (d) ๐‘ฆ = 4โˆ’๐‘ฅ 7. The equations involving redical expressions of the variable are called: (a) Reciprocal equations(b) โœ” Redical equations (c) Quadratic functions (d) exponential equations 8. The cube roots of unity are : (a) โœ”1, โˆ’1+โˆš3๐‘– 2 , โˆ’1+โˆš3๐‘– 2 (b) 1, 1+โˆš3๐‘– 2 , 1+โˆš3๐‘– 2 (c) โˆ’1, โˆ’1+โˆš3๐‘– 2 , โˆ’1+โˆš3๐‘– 2 (d) โˆ’1, 1+โˆš3๐‘– 2 , 1+โˆš3๐‘– 2 9. Sum of all cube roots of 64 is : (a) โœ”0 (b) 1 (c) 64 (d) -64 10. Product of cube roots of -1 is: (a) 0 (b) -1 (c) โœ”1 (d) None 11. ๐Ÿ๐Ÿ”๐Ž๐Ÿ– + ๐Ÿ๐Ÿ”๐’˜๐Ÿ’ = (a) 0 (b) โœ” -16 (c) 16 (d) -1 12. The sum of all four fourth roots of unity is: (a) Unity (b) โœ”0 (c) -1 (d) None 13. The product of all four fourth roots of unity is: (a) Unity (b) 0 (c) โœ”-1 (d) None 14. The sum of all four fourth roots of 16 is: (a) 16 (b) -16 (c) โœ” 0 (d) 1 15. The complex cube roots of unity areโ€ฆโ€ฆโ€ฆโ€ฆโ€ฆโ€ฆ. each other. (a) Additive inverse (b) Equal to (c) โœ”Conjugate (d) None of these 16. The complex cube roots of unity areโ€ฆโ€ฆโ€ฆโ€ฆโ€ฆโ€ฆ. each other. (a) Multiplicative inverse (b) reciprocal (c) square (d) โœ”None of these 17. The complex fourth roots of unity are โ€ฆโ€ฆ. of each other. (a) โœ”Additive inverse (b) equal to (c) square of (d) None of these 18. If sum of all cube roots of unity is equal to ๐’™๐Ÿ + ๐Ÿ , then ๐’™ is equal to: (a) โˆ’1 (b) 0 (c) โœ”ยฑ๐‘– (d) 1 19. If product of all cube roots of unity is equal to ๐’‘๐Ÿ + ๐Ÿ , then ๐’‘ is equal to: (a) โˆ’1 (b) โœ” 0 (c) ยฑ๐‘– (d) 1 20. The expression ๐’™๐Ÿ + ๐Ÿ ๐’™ โˆ’ ๐Ÿ‘ is polynomial of degree: (a) 2 (b) 3 (c) 1 (d) โœ” not a polynomial 21. If ๐’‡(๐’™) is divided by ๐’™ โˆ’ ๐’‚ , then dividend = (Divisor)(โ€ฆโ€ฆ.)+ Remainder. (a) Divisor (b) Dividend (c) โœ” Quotient (d) ๐‘“(๐‘Ž) 22. If ๐’‡(๐’™) is divided by ๐’™ โˆ’ ๐’‚ by remainder theorem then remainder is: (a) โœ” ๐‘“(๐‘Ž) (b) ๐‘“(โˆ’๐‘Ž) (c) ๐‘“(๐‘Ž) + ๐‘… (d) ๐‘ฅ โˆ’ ๐‘Ž = ๐‘… 23. The polynomial (๐’™ โˆ’ ๐’‚) is a factor of ๐’‡(๐’™) if and only if (a) โœ” ๐‘“(๐‘Ž) = 0 (b) ๐‘“(๐‘Ž) = ๐‘… (c) Quotient = ๐‘… (d) ๐‘ฅ = โˆ’๐‘Ž 24. ๐’™ โˆ’ ๐Ÿ is a factor of ๐’™๐Ÿ โˆ’ ๐’Œ๐’™ + ๐Ÿ’, if ๐’Œ is: (a) 2 (b) โœ” 4 (c) 8 (d) -4 25. If ๐’™ = โˆ’๐Ÿ is the root of ๐’Œ๐’™๐Ÿ’ โˆ’ ๐Ÿ๐Ÿ‘๐’™๐Ÿ + ๐Ÿ‘๐Ÿ” = ๐ŸŽ, then ๐’Œ =
  • 9. 9 | P a g e (a) 2 (b) -2 (c) 1 (d) โœ” -1 26. ๐’™ + ๐’‚ is a factor of ๐’™๐’ + ๐’‚๐’ when ๐’ is (a) Any integer (b) any positive integer (c) โœ” any odd integer (d) any real number 27. ๐’™ โˆ’ ๐’‚ is a factor of ๐’™๐’ โˆ’ ๐’‚๐’ when ๐’ is (a) โœ” Any integer (b) any positive integer (c) any odd integer (d) any real number 28. Sum of roots of ๐’‚๐’™๐Ÿ โˆ’ ๐’ƒ๐’™ โˆ’ ๐’„ = ๐ŸŽ is (๐’‚ โ‰  ๐ŸŽ) (a) โœ” ๐‘ ๐‘Ž (b) โ€“ ๐‘ ๐‘Ž (c) ๐‘ ๐‘Ž (d) โ€“ ๐‘ ๐‘Ž 29. Sum of roots of ๐’‚๐’™๐Ÿ โˆ’ ๐’ƒ๐’™ โˆ’ ๐’„ = ๐ŸŽ is (๐’‚ โ‰  ๐ŸŽ) (a) ๐‘ ๐‘Ž (b) โ€“ ๐‘ ๐‘Ž (c) ๐‘ ๐‘Ž (d) โœ” โ€“ ๐‘ ๐‘Ž 30. If 2 and -5 are roots of a quadratic equation , then equation is: (a) ๐‘ฅ2 โˆ’ 3๐‘ฅ โˆ’ 10 = 0 (b) ๐‘ฅ2 โˆ’ 3๐‘ฅ + 10 = 0 (c) โœ” ๐‘ฅ2 + 3๐‘ฅ โˆ’ 10 = 0 (d) ๐‘ฅ2 + 3๐‘ฅ + 10 = 0 31. If ๐œถ and ๐œท are the roots of ๐Ÿ‘๐’™๐Ÿ โˆ’ ๐Ÿ๐’™ + ๐Ÿ’ = ๐ŸŽ, then the value of ๐œถ + ๐œท is: (a) โœ” 2 3 (b) โˆ’ 2 3 (c) 4 3 (d) โˆ’ 4 3 32. If roots of ๐’‚๐’™๐Ÿ + ๐’ƒ๐’™ + ๐’„ = ๐ŸŽ, (๐’‚ โ‰  ๐ŸŽ) are real , then (a) Discโ‰ฅ 0 (b) Disc< 0 (c) Discโ‰  0 (d) Discโ‰ค 0 33. If roots of ๐’‚๐’™๐Ÿ + ๐’ƒ๐’™ + ๐’„ = ๐ŸŽ, (๐’‚ โ‰  ๐ŸŽ) are complex , then (a) โœ”Discโ‰ฅ 0 (b) โœ” Disc< 0 (c) Discโ‰  0 (d) Discโ‰ค 0 34. If roots of ๐’‚๐’™๐Ÿ + ๐’ƒ๐’™ + ๐’„ = ๐ŸŽ, (๐’‚ โ‰  ๐ŸŽ) are equal , then (a) โœ”Disc= 0 (b) Disc< 0 (c) Discโ‰  0 (d) None of these 35. Graph of quadratic equation is: (a) Straight line (b) Circle (c) square (d) โœ”Parabola 36. ๐Ž๐Ÿ๐Ÿ– + ๐Ž๐Ÿ๐Ÿ— + ๐Ÿ = (a) โœ”0 (b) 1 (c) -1 (d) ๐œ” 37. Synthetic division is a process of: (a) Addition (b) multiplication (c) subtraction (d) โœ” division 38. Degree of quadratic equation is: (a) 0 (b)1 (c) โœ”2 (d) 3 39. Basic techniques for solving quadratic equations are (a) 1 (b) 2 (c) โœ”3 (d) 4 40. ๐Ÿ๐Ÿ”๐Ž๐Ÿ’ + ๐Ÿ๐Ÿ”๐Ž๐Ÿ– = (a) 0 (b) โœ” -16 (c) 16 (d) -1 UNIT # 05 Partial Fractions Each question has four possible answer. Tick the correct answer. 1. An open sentence formed by using sign of โ€œ = โ€ is called a/an (a) โœ”Equation (b) Formula (c) Rational fraction (d) Theorem 2. If an equation is true for all values of the variable, then it is called: (a) a conditional equation (b) โœ” an identity (c) proper rational fraction (d) All of these 3. (๐’™ + ๐Ÿ‘)(๐’™ + ๐Ÿ’) = ๐’™๐Ÿ + ๐Ÿ•๐’™ + ๐Ÿ๐Ÿ is a/an: (a) Conditional equation (b) โœ”an identity (c) proper rational fraction (d) a formula 4. The quotient of two polynomials ๐‘ท(๐’™) ๐‘ธ(๐’™) , ๐‘ธ(๐’™) โ‰  ๐ŸŽ is called : (a) โœ”Rational fraction (b) Irrational fraction (c) Partial fraction (d) Proper fraction 5. A fraction ๐‘ท(๐’™) ๐‘ธ(๐’™) , ๐‘ธ(๐’™) โ‰  ๐ŸŽ is called proper fraction if : (a) โœ”Degree of ๐‘ƒ(๐‘ฅ) < Degree of ๐‘„(๐‘ฅ) (b) Degree of ๐‘ƒ(๐‘ฅ) = Degree of ๐‘„(๐‘ฅ) (c) Degree of ๐‘ƒ(๐‘ฅ) > Degree of ๐‘„(๐‘ฅ) (d) Degree of ๐‘ƒ(๐‘ฅ) โ‰ฅ Degree of ๐‘„(๐‘ฅ) 6. A fraction ๐‘ท(๐’™) ๐‘ธ(๐’™) , ๐‘ธ(๐’™) โ‰  ๐ŸŽ is called proper fraction if : (a) Degree of ๐‘ƒ(๐‘ฅ) < Degree of ๐‘„(๐‘ฅ) (b) Degree of ๐‘ƒ(๐‘ฅ) = Degree of ๐‘„(๐‘ฅ) (c) Degree of ๐‘ƒ(๐‘ฅ) > Degree of ๐‘„(๐‘ฅ) (d) โœ” Degree of ๐‘ƒ(๐‘ฅ) โ‰ฅ Degree of ๐‘„(๐‘ฅ) 7. A mixed form of fraction is : (a) An integer+ improper fraction (b) a polynomial+improper fraction (c) โœ”a polynomial+proper fraction (d) a polynomial+rational fraction 8. When a rational fraction is separated into partial fractions, then result is always : (a) A conditional equations (b) โœ”an identity (c) a partial fraction (d) an improper fraction
  • 10. 10 | P a g e 9. The number of Partial fraction of ๐’™๐Ÿ‘ ๐’™(๐’™+๐Ÿ)(๐’™๐Ÿโˆ’๐Ÿ) are: (a) 2 (b) 3 (c) โœ”4 (d) None of these 10. The number of Partial fraction of ๐’™๐Ÿ“ ๐’™(๐’™+๐Ÿ)(๐’™๐Ÿโˆ’๐Ÿ’) are: (a) 2 (b) 3 (c) 4 (d) โœ” 6 11. Partial fractions of ๐Ÿ ๐’™๐Ÿโˆ’๐Ÿ are: (a) 1 2(๐‘ฅโˆ’1) + 1 2(๐‘ฅ+1) (b) โœ” 1 2(๐‘ฅโˆ’1) โˆ’ 1 2(๐‘ฅ+1) (c) โˆ’ 1 2(๐‘ฅโˆ’1) + 1 2(๐‘ฅ+1) (d) โˆ’ 1 2(๐‘ฅโˆ’1) โˆ’ 1 2(๐‘ฅ+1) 12. Conditional equation ๐Ÿ๐’™ + ๐Ÿ‘ = ๐ŸŽ holds when ๐’™ is equal to: (a) โœ”โˆ’ 3 2 (b) 3 2 (c) 1 3 (d) 1 13. Which is a reducible factor: (a) ๐‘ฅ3 โˆ’ 6๐‘ฅ2 + 8๐‘ฅ (b) ๐‘ฅ2 + 16๐‘ฅ (c) ๐‘ฅ2 + 5๐‘ฅ โˆ’ 6 (d) โœ”all of these 14. A quadratic factor which cannot written as a product of linear factors with real coefficients is called: (a) โœ”An irreducible factor (b) reducible factor (c) an irrational factor (d) an improper factor 15. ๐Ÿ—๐’™๐Ÿ ๐’™๐Ÿ‘โˆ’๐Ÿ is an (a) Improper fraction (b) โœ” Proper fraction (c) Polynomial (d) equation UNIT # 06 Sequence and Series Each question has four possible answer. Tick the correct answer. 1. An arrangement of numbers according to some definite rule is called: (a) โœ”Sequence (b) Combination (c) Series (d) Permutation 2. A sequence is also known as: (a) Real sequence (b) โœ”Progression (c) Arrangement (d) Complex sequence 3. A sequence is function whose domain is (a) ๐‘ (b) ๐‘ (c) ๐‘„ (d) โœ” ๐‘… 4. As sequence whose range is ๐‘น ๐’Š. ๐’†., set of real numbers is called: (a) โœ”Real sequence (b) Imaginary sequence (c) Natural sequence (d) Complex sequence 5. If ๐’‚๐’ = {๐’ + (โˆ’๐Ÿ)๐’}, then ๐’‚๐Ÿ๐ŸŽ = (a) 10 (b) โœ” 11 (c) 12 (d) 13 6. The last term of an infinite sequence is called : (a) ๐‘›๐‘กโ„Ž term (b) ๐‘Ž๐‘› (c) last term (d) โœ” does not exist 7. The next term of the sequence ๐Ÿ, ๐Ÿ, ๐Ÿ๐Ÿ, ๐Ÿ’๐ŸŽ, โ€ฆ is (a) โœ”112 (b) 120 (c) 124 (d) None of these 8. A sequence {๐’‚๐’} in which ๐’‚๐’ โˆ’ ๐’‚๐’โˆ’๐Ÿ is the same number for all ๐’ โˆˆ ๐‘ต,๐’ > 1 is called: (a) โœ”A.P (b) G.P (c) H.P (d) None of these 9. ๐’๐’•๐’‰ term of an A.P is ๐Ÿ‘๐’ โˆ’ ๐Ÿ then 10th term is : (a) 9 (b) โœ” 29 (c) 12 (d) cannot determined 10. ๐’๐’•๐’‰ term of the series ( ๐Ÿ ๐Ÿ‘ ) ๐Ÿ + ( ๐Ÿ“ ๐Ÿ ) ๐Ÿ + ( ๐Ÿ• ๐Ÿ‘ ) ๐Ÿ + โ‹ฏ (a) โœ” ( 2๐‘›โˆ’1 3 ) 2 (b) ( 2๐‘›โˆ’1 3 ) 2 (c) ( 2๐‘› 3 ) 2 (d) cannot determined 11. If ๐’‚๐’โˆ’๐Ÿ, ๐’‚๐’, ๐’‚๐’+๐Ÿ are in A.P, then ๐’‚๐’ is (a) โœ”A.M (b) G.M (c) H.M (d) Mid point 12. Arithmetic mean between ๐’„ and ๐’… is: (a) โœ” ๐‘+๐‘‘ 2 (b) ๐‘+๐‘‘ 2๐‘๐‘‘ (c) 2๐‘๐‘‘ ๐‘+๐‘‘ (d) 2 ๐‘+๐‘‘ 13. The arithmetic mean between โˆš๐Ÿ and ๐Ÿ‘โˆš๐Ÿ is: (a) 4โˆš2 (b) โœ” 4 โˆš2 (c) โˆš2 (d) none of these 14. The sum of terms of a sequence is called: (a) Partial sum (b) โœ” Series (c) Finite sum (d) none of these 15. Forth partial sum of the sequence {๐’๐Ÿ } is called: (a) 16 (b) โœ” 1+4+9+16 (c) 8 (d) 1+2+3+4 16. Sum of ๐’ โˆ’term of an Arithmetic series ๐‘บ๐’ is equal to: (a) โœ” ๐‘› 2 [2๐‘Ž + (๐‘› โˆ’ 1)๐‘‘] (b) ๐‘› 2 [๐‘Ž + (๐‘› โˆ’ 1)๐‘‘] (c) ๐‘› 2 [2๐‘Ž + (๐‘› + 1)๐‘‘] (d) ๐‘› 2 [2๐‘Ž + ๐‘™] 17. For any ๐‘ฎ. ๐‘ท., the common ratio ๐’“ is equal to: (a) ๐‘Ž๐‘› ๐‘Ž๐‘›+1 (b) ๐‘Ž๐‘›โˆ’1 ๐‘Ž๐‘› (c) โœ” ๐‘Ž๐‘› ๐‘Ž๐‘›โˆ’1 (d) ๐‘Ž๐‘›+1 โˆ’ ๐‘Ž๐‘›, ๐‘› โˆˆ ๐‘, ๐‘› > 1
  • 11. 11 | P a g e 18. No term of a ๐‘ฎ. ๐‘ท., is: (a) โœ”0 (b) 1 (c) negative (d) imaginary number 19. The general term of a ๐‘ฎ. ๐‘ท., is : (a) โœ”๐‘Ž๐‘› = ๐‘Ž๐‘Ÿ๐‘›โˆ’1 (b) ๐‘Ž๐‘› = ๐‘Ž๐‘Ÿ๐‘› (c) ๐‘Ž๐‘› = ๐‘Ž๐‘Ÿ๐‘›+1 (d) None of these 20. The sum of infinite geometric series is valid if (a) |๐‘Ÿ| > 1 (b) |๐‘Ÿ| = 1 (c) |๐‘Ÿ| โ‰ฅ 1 (d) โœ” |๐‘Ÿ| < 1 21. For the series ๐Ÿ + ๐Ÿ“ + ๐Ÿ๐Ÿ“ + ๐Ÿ๐Ÿ๐Ÿ“ + โ‹ฏ + โˆž , the sum is (a) -4 (b) 4 (c) 1โˆ’5๐‘› โˆ’4 (d) โœ” not defined 22. An infinite geometric series is convergent if (a) |๐‘Ÿ| > 1 (b) |๐‘Ÿ| = 1 (c) |๐‘Ÿ| โ‰ฅ 1 (d) โœ” |๐‘Ÿ| < 1 23. An infinite geometric series is divergent if (a) |๐‘Ÿ| < 1 (b) |๐‘Ÿ| โ‰  1 (c) ๐‘Ÿ = 0 (dโœ”) |๐‘Ÿ| > 1 24. If sum of series is defined then it is called: (a) โœ”Convergent series (b) Divergent series (c) finite series (d) Geometric series 25. If sum of series is not defined then it is called: (a) Convergent series (b) โœ” Divergent series (c) finite series (d) Geometric series 26. The interval in which series ๐Ÿ + ๐Ÿ๐’™ + ๐Ÿ’๐’™๐Ÿ + ๐Ÿ–๐’™๐Ÿ‘ + โ‹ฏ is convergent if : (a) โˆ’2 < ๐‘ฅ < 2 (b) โœ” โˆ’ 1 2 < ๐‘ฅ < 1 2 (c) |2๐‘ฅ| > 1 (d) |๐‘ฅ| < 1 27. If the reciprocal of the terms a sequence form an ๐‘จ. ๐‘ท., then it is called: (a) โœ”๐ป. ๐‘ƒ (b) ๐บ. ๐‘ƒ (c) ๐ด. ๐‘ƒ (d) sequence 28. The ๐’๐’•๐’‰ term of ๐Ÿ ๐Ÿ , ๐Ÿ ๐Ÿ“ , ๐Ÿ ๐Ÿ– , โ€ฆ is (a) โœ” 1 3๐‘›โˆ’1 (b) 3๐‘› โˆ’ 1 (c) 2๐‘› + 1 (d) 1 3๐‘›+1 29. Harmonic mean between ๐Ÿ and ๐Ÿ– is: (a) โœ”5 (b) 16 5 (c) ยฑ4 (d) 5 16 30. If ๐‘จ, ๐‘ฎ and ๐‘ฏ are Arithmetic , Geometric and Harmonic means between two positive numbers then (a) ๐บ2 = ๐ด๐ป (b) โœ”๐ด, ๐บ, ๐ป ๐‘Ž๐‘Ÿ๐‘’ ๐‘–๐‘› ๐บ. ๐‘ƒ (c) ๐ด > ๐บ > ๐ป (d) all of these 31. If ๐‘จ, ๐‘ฎ and ๐‘ฏ are Arithmetic , Geometric and Harmonic means between two negative numbers then (a) ๐บ2 = ๐ด๐ป (b) ๐ด, ๐บ, ๐ป ๐‘Ž๐‘Ÿ๐‘’ ๐‘–๐‘› ๐บ. ๐‘ƒ (c) ๐ด < ๐บ < ๐ป (d) โœ”all of these 32. If ๐’‚ and ๐’ƒ are two positive number then (a) ๐ด < ๐บ < ๐ป (b) โœ” ๐ด > ๐บ > ๐ป (c) ๐ด = ๐บ = ๐ป (d) ๐ด โ‰ฅ ๐บ โ‰ฅ ๐ป 33. If ๐’‚ and ๐’ƒ are two negative number then (a) โœ”๐ด < ๐บ < ๐ป (b) ๐ด > ๐บ > ๐ป (c) ๐ด = ๐บ = ๐ป (d) ๐ด โ‰ฅ ๐บ โ‰ฅ ๐ป 34. If ๐’‚๐’+๐Ÿ+๐’ƒ๐’+๐Ÿ ๐’‚๐’+๐’ƒ๐’ is A.M between ๐’‚ and ๐’ƒ then ๐’ is equal to: (a) โœ”0 (b) -1 (c) 1 (d) 1 2 35. If ๐’‚๐’+๐’ƒ๐’ ๐’‚๐’โˆ’๐Ÿ+๐’ƒ๐’โˆ’๐Ÿ is G.M between ๐’‚ and ๐’ƒ then ๐’ is equal to: (a) 0 (b) -1 (c) 1 (d) โœ” 1 2 36. If ๐’‚๐’+๐Ÿ+๐’ƒ๐’+๐Ÿ ๐’‚๐’+๐’ƒ๐’ is H.M between ๐’‚ and ๐’ƒ then ๐’ is equal to: (a) 0 (b) โœ” -1 (c) 1 (d) 1 2 37. If ๐‘บ๐’ = (๐’ + ๐Ÿ)๐Ÿ , then ๐‘บ๐Ÿ๐’ is equal to: (a) 2๐‘› + 1 (b) โœ” 4๐‘›2 + 4๐‘› + 1 (c) (2๐‘› โˆ’ 1)2 (d) cannot be determined 38. โˆ‘ ๐’๐Ÿ‘ = (a) ๐‘›(๐‘›+1) 2 (b) ๐‘›(๐‘›+1)(๐‘›+2) 6 (c) โœ” ๐‘›2(๐‘›+1)2 4 (d) ๐‘›(๐‘›+1)2 2 UNIT # 07 Permutation, Combination and Probability Each question has four possible answer. Tick the correct answer. 1. ๐Ÿ๐ŸŽ๐‘ท๐Ÿ‘= (a) 6890 (b) 6810 (c) โœ”6840 (d) 6880 2. If ๐’๐‘ท๐Ÿ=30 then ๐’ = (a) 4 (b) 5 (c) โœ”6 (d) 10
  • 12. 12 | P a g e 3. The number of diagonals in 10-sided figure is (a) 10 (b) 10๐ถ2 (c) โœ”10๐ถ2 โˆ’ 10 (d) 45 4. ๐’๐‘ช๐ŸŽ= (a) 0 (b) โœ” 1 (c) ๐‘› (d) ๐‘›! 5. How many arrangement of the word โ€œMATHEMATICSโ€ can be made (a) 11! (b) ( 11 3,2,1,1,1,1,1 ) (c) โœ” ( 11 2,2,2,1,1,1,1,1 ) (d) None 6. If ๐’ is negative then ๐’! is (a) 1 (b) 0 (c) unique (d) โœ”Not defined 7. ๐’ โˆ’ ๐Ÿ๐‘ช๐’“ + ๐’ โˆ’ ๐Ÿ๐‘ช๐’“โˆ’๐Ÿ equals (a) โœ”๐‘›๐ถ๐‘Ÿ (b) ๐‘›๐ถ๐‘Ÿโˆ’1 (c) ๐‘› โˆ’ 1๐ถ๐‘Ÿ (d) ๐‘› + 1๐ถ๐‘Ÿ 8. How many signals can be given by 5 flags of different colors , using 3 at a time (a) 120 (b) โœ”60 (c) 24 (d) 15 9. ๐’๐‘ท๐’= (a) 1 (b) ๐‘› (c) โœ” ๐‘›! (d) None 10. ๐Ÿ–! ๐Ÿ•! = (a) โœ”8 (b) 7 (c) 56 (d) 8 7 11. If an event ๐‘จ can occur in ๐’‘ ways and ๐‘ฉ can occur ๐’’ ways , then number of ways that both events occur is: (a) ๐‘ + ๐‘ž (b) โœ” ๐‘. ๐‘ž (c) (๐‘๐‘ž)! (d) (๐‘ + ๐‘ž)! 12. If ( ๐’ ๐Ÿ๐Ÿ ) = (๐’ ๐Ÿ– ) then the value of ๐’, (a) 15 (b) 16 (c) 18 (d) โœ”20 13. Probability of non-occurrence of an event E is equal to : (a) โœ”1 โˆ’ ๐‘ƒ(๐ธ) (b) ๐‘ƒ(๐ธ) + ๐‘›(๐‘†) ๐‘›(๐ธ) (c) ๐‘›(๐‘†) ๐‘›(๐ธ) (d) 1 + ๐‘ƒ(๐ธ) 14. For independent events ๐‘ท(๐‘จ โˆฉ ๐‘ฉ) = (a) ๐‘ƒ(๐ด) + ๐‘ƒ(๐ต) (b) ๐‘ƒ(๐ด) โˆ’ ๐‘ƒ(๐ต) (c) โœ” ๐‘ƒ(๐ด). ๐‘ƒ(๐ต) (d) ๐‘ƒ(๐ด) ๐‘ƒ(๐ต) 15. A card is drawn from a deck of 52 playing cards. The probability of card that it is an ace card is: (a) 2 13 (b) 4 13 (c) โœ” 1 13 (d) 17 13 16. Four persons wants to sit in a circular sofa, the total ways are: (a) 24 (b) โœ”6 (c) 4 (d) None of these 17. Two teams ๐‘จ and ๐‘ฉ are playing a match, the probability that team ๐‘จ does not lose is: (a) 1 3 (b) 2 3 (c) 1 (d) 0 18. Let ๐‘บ = {๐Ÿ, ๐Ÿ, ๐Ÿ‘, โ€ฆ , ๐Ÿ๐ŸŽ} the probability that a number is divided by 4 is : (a) 2 5 (b) โœ” 1 5 (c) 1 10 (d) 1 2 19. A die is rolled , the probability of getting 3 or 5 is: (a) 2 3 (b) โœ” 15 36 (c) 15 36 (d) 1 36 20. A coin is tossed 5 times , then ๐’(๐‘บ) is equal to: (a) โœ”32 (b) 25 (c) 10 (d) 20 21. The number of ways for sitting 4 persons in a train on a straight sofa is: (a) โœ”24 (b) 6 (c) 4 (d) None of these 22. Sample space for tossing a coin is: (a) {๐ป} (b) {๐‘‡} (c) {๐ป, ๐ป} (d) โœ” {๐ป, ๐‘‡} 23. If ๐‘ท(๐‘ฌ) = ๐Ÿ• ๐Ÿ๐Ÿ , ๐’(๐‘บ) = ๐Ÿ–๐Ÿ’๐ŸŽ๐ŸŽ , ๐’(๐‘ฌ) = (a) 108 (b) โœ”4900 (c) 144 (d) 14400 24. In a permutation ๐’๐‘ท๐’“ ๐’๐’“ ๐‘ท(๐’, ๐’“) , it is always true that (a) โœ”๐‘› โ‰ฅ ๐‘Ÿ (b) ๐‘› < ๐‘Ÿ (c) ๐‘› โ‰ค ๐‘Ÿ (d) ๐‘› < 0, ๐‘Ÿ < 0 25. If an event always occurs , then it is called: (a) Null Event (b) Possible Event (c) โœ”Certain event (d) Independent Event 26. If ๐‘ฌ is a certain event , then (a) ๐‘ƒ(๐ธ) = 0 (b) โœ”๐‘ƒ(๐ธ) = 1 (c) 0 < ๐‘ƒ(๐ธ) < 1 (d) ๐‘ƒ(๐ธ) > 1 27. If ๐‘ฌ is an impossible event ,then (a) โœ”๐‘ƒ(๐ธ) = 0 (b) ๐‘ƒ(๐ธ) = 1 (c) ๐‘ƒ(๐ธ) โ‰  0 (d) 0 < ๐‘ƒ(๐ธ) < 1
  • 13. 13 | P a g e 28. Non occurrence of an event E is denoted by: (a) โˆผ ๐ธ (b) โœ” ๐ธ (c) ๐ธ๐‘ (d) All of these UNIT # 08 Mathematical Induction and Binomial Theorem Each question has four possible answer. Tick the correct answer. 1. The statement ๐Ÿ’๐’ + ๐Ÿ‘๐’ + ๐Ÿ’ is true when : (a) ๐‘› = 0 (b) ๐‘› = 1 (c) โœ” ๐‘› โ‰ฅ 2 (d) ๐‘› is any +iv integer 2. The number of terms in the expansion of (๐’‚ + ๐’ƒ)๐’ are: (a) ๐‘› (b) โœ” ๐‘› + 1 (c) 2๐‘› (d) 2๐‘›โˆ’1 3. Middle term/s in the expansion of (๐’‚ โˆ’ ๐Ÿ‘๐’™)๐Ÿ๐Ÿ’ is/are : (a) ๐‘‡7 (b) โœ” ๐‘‡8 (c) ๐‘‡6&๐‘‡7 (d) ๐‘‡7&๐‘‡8 4. The coefficient of the last term in the expansion of (๐Ÿ โˆ’ ๐’™)๐Ÿ• is : (a) 1 (b) โœ” โˆ’1 (c) 7 (d) โˆ’7 5. (๐Ÿ๐’ ๐ŸŽ ) + (๐Ÿ๐’ ๐Ÿ ) + (๐Ÿ๐’ ๐Ÿ ) + โ‹ฏ + (๐Ÿ๐’ ๐Ÿ๐’ ) is equal to: (a) 2๐‘› (b) โœ”22๐‘› (c) 22๐‘›โˆ’1 (d) 22๐‘›+1 6. ๐Ÿ + ๐’™ + ๐’™๐Ÿ + ๐’™๐Ÿ‘ + โ‹ฏ (a) (1 + ๐‘ฅ)โˆ’1 (b) โœ”(1 โˆ’ ๐‘ฅ)โˆ’1 (c) (1 + ๐‘ฅ)โˆ’2 (d) (1 โˆ’ ๐‘ฅ)โˆ’2 7. The middle term in the expansion of (๐’‚ + ๐’ƒ)๐’ is ( ๐’ ๐Ÿ + ๐Ÿ) ; then ๐’ is (a) Odd (b) โœ” even (c) prime (d) none of these 8. The number of terms in the expansion of (๐’‚ + ๐’ƒ)๐Ÿ๐ŸŽ is: (a) 18 (b) 20 (c) โœ” 21 (d) 19 9. The expansion (๐Ÿ โˆ’ ๐Ÿ’๐’™)โˆ’๐Ÿ is valid if: (a) โœ”|๐‘ฅ| < 1 4 (b) |๐‘ฅ| > 1 4 (c) โˆ’1 < ๐‘ฅ < 1 (d) |๐‘ฅ| < โˆ’1 10. The statement ๐Ÿ‘๐’ < ๐‘›! is true, when (a) ๐‘› = 2 (b) ๐‘› = 4 (c) ๐‘› = 6 (d) โœ”๐‘› > 6 11. General term in the expansion of (๐’‚ + ๐’ƒ)๐’ is: (a) (๐‘›+1 ๐‘Ÿ )๐‘Ž๐‘›โˆ’๐‘Ÿ ๐‘ฅ^๐‘Ÿ (b)โœ”( ๐‘› ๐‘Ÿโˆ’1 )๐‘Ž๐‘›โˆ’๐‘Ÿ ๐‘ฅ๐‘Ÿ (c) ( ๐‘› ๐‘Ÿ+1 )๐‘Ž๐‘›โˆ’๐‘Ÿ ๐‘ฅ๐‘Ÿ (d) (๐‘› ๐‘Ÿ )๐‘Ž๐‘›โˆ’๐‘Ÿ ๐‘ฅ๐‘Ÿ 12. The method of induction was given by Francesco who lived from: (a) โœ”1494-1575 (b) 1500-1575 (c) 1498-1575 (d) 1494-1570 UNIT # 09 Fundamentals of Trigonometry Each question has four possible answer. Tick the correct answer. 1. Two rays with a common starting point form: (a) Triangle (b) โœ” Angle (c) Radian (d) Minute 2. The common starting point of two rays is called: (a) Origin (b) Initial Point (c) โœ” Vertex (d)All of these 3. If the rotation of the angle is counter clock wise, then angle is: (a) Negative (b) โœ” Positive (c) Non-Negative (d) None of these 4. If the initial ray ๐‘ถ๐‘จ โƒ—โƒ—โƒ—โƒ—โƒ—โƒ— rates in anti-clockwise direction in such a way that it coincides with itself, the angle then formed is: (a) 180ยฐ (b) 270ยฐ (c) 300ยฐ (d) โœ” 360ยฐ 5. One rotation in anti-clock wise direction is equal to: (a) 180ยฐ (b) 270ยฐ (c) โœ”360ยฐ (d) 90ยฐ 6. Straight line angle is equal to (a) 1 2 ๐‘Ÿ๐‘œ๐‘ก๐‘Ž๐‘ก๐‘–๐‘œ๐‘› (b) ๐œ‹ radian (c) 180ยฐ (d) โœ” All of these 7. One right angle is equal to (a) ๐œ‹ 2 ๐‘Ÿ๐‘Ž๐‘‘๐‘–๐‘Ž๐‘› (b) 90ยฐ (c) 1 4 ๐‘Ÿ๐‘œ๐‘ก๐‘Ž๐‘ก๐‘–๐‘œ๐‘› (d) โœ” All of these 8. ๐Ÿยฐ is equal to (a) 30 minutes (b) โœ” 60 minutes (c) 1 60 minutes (d) 1 2 minutes 9. ๐Ÿยฐ is equal to (a) 360โ€ฒโ€ฒ (b) โœ” 3600โ€ฒโ€ฒ (c) ( 1 360 )โ€ฒ (d) 60โ€ฒโ€ฒ
  • 14. 14 | P a g e 10. 60th part of ๐Ÿยฐ is equal to (a) One second (b) โœ” One minute (c) 1 Radian (d) ๐œ‹ radian 11. 60th part of ๐Ÿโ€ฒ is equal to (a) 1โ€™ (b) โœ” 1โ€™โ€™ (c) 60โ€™โ€™ (d) 3600โ€™โ€™ 12. 3600th part of ๐Ÿยฐ is equal to (a) 1โ€™ (b) โœ” 1โ€™โ€™ (c) 60โ€™โ€™ (d) 3600โ€™โ€™ 13. Sexagesimal system is also called (a) German System (b) โœ” English System (c) C.G.S System (d) SI System 14. ๐Ÿ๐Ÿ”ยฐ๐Ÿ‘๐ŸŽโ€ฒ equal to (a) โœ”16.5ยฐ (b) 32ยฐ 2 (c) 16.05ยฐ (d) 16.2ยฐ 15. Conversion of ๐Ÿ๐Ÿ. ๐Ÿ๐Ÿ“๐Ÿ”ยฐ to ๐‘ซยฐ๐‘ดโ€ฒ ๐‘บโ€ฒ โ€ฒ form is: (a) 21ยฐ25โ€™6โ€™โ€™ (b) 21ยฐ40โ€ฒ27โ€ฒโ€ฒ (c) โœ” 21ยฐ15โ€ฒ22โ€ฒโ€ฒ (d) 21ยฐ30โ€ฒ2โ€ฒโ€ฒ 16. The angle subtended at the center of a circle by an arc whose length is equal to the radius of the circle is called: (a) 1 Degree (b) 1โ€™ (c) โœ” 1 Radian (d) 1โ€™โ€™ 17. The system of angular measurement in which the angle is measured in radian is called: (a) Sexagesimal System (b) โœ” Circular System (c) English System (d) Gradient System 18. Relation between the length of arc of a circle and the circular measure of it central angle is: (a) ๐‘™ = ๐‘Ÿ ๐œƒ (b) ๐œƒ = ๐‘™๐‘Ÿ (c) โœ” ๐œƒ = ๐‘™ ๐‘Ÿ (d) ๐‘™ = 1 2 ๐‘Ÿ2 ๐œƒ 19. With usual notation , if ๐’ = ๐Ÿ”๐’„๐’Ž, ๐’“ = ๐Ÿ๐’„๐’Ž, then unit of ๐œฝ is: (a) ๐‘๐‘š (b) ๐‘๐‘š2 (c) โœ” No unit (d) ๐‘๐‘š3 20. ๐Ÿยฐ is equal to: (a) ( ๐œ‹ 180 ) ยฐ (b) 180 ๐œ‹ ๐‘Ÿ๐‘Ž๐‘‘ (c) ( 180 ๐œ‹ ) ๐‘Ÿ๐‘Ž๐‘‘ (d) โœ” ๐œ‹ 180 ๐‘Ÿ๐‘Ž๐‘‘ 21. ๐Ÿยฐ is equal to: (a) 0.175 ๐‘Ÿ๐‘Ž๐‘‘ (b) โœ”0.0175๐‘Ÿ๐‘Ž๐‘‘ (c) 1.75 ๐‘Ÿ๐‘Ž๐‘‘ (d) 0.00175๐‘Ÿ๐‘Ž๐‘‘ 22. 1 radian is equal to (a) ( ๐œ‹ 180 ) ยฐ (b) 180 ๐œ‹ ๐‘Ÿ๐‘Ž๐‘‘ (c) โœ” ( 180 ๐œ‹ ) ยฐ (d) ๐œ‹ 180 ๐‘Ÿ๐‘Ž๐‘‘ 23. 1 radian is equal to: (a) โœ”57.296ยฐ (b) 5.7296ยฐ (c) 175.27ยฐ (d) 17.5276 24. 3 radian is: (a) โœ”171.888ยฐ (b) 120ยฐ (c) 300ยฐ (d) 270ยฐ 25. ๐Ÿ๐ŸŽ๐Ÿ“ยฐ = __________๐’“๐’‚๐’…๐’Š๐’‚๐’ (a) โœ” 7๐œ‹ 12 (b) 2๐œ‹ 3 (c) 5๐œ‹ 12 (d) 5๐œ‹ 6 26. 3โ€™โ€™= ___________ ๐’“๐’‚๐’…๐’Š๐’‚๐’ (a) 53๐œ‹ 270 (b) โœ” ๐œ‹ 216000 (c) 41๐œ‹ 720 (d) 27721๐œ‹ 32400 27. ๐… ๐Ÿ’ ๐’“๐’‚๐’…๐’Š๐’‚๐’ = ________๐’…๐’†๐’ˆ๐’“๐’†๐’† (a) โœ”45ยฐ (b) 30ยฐ (c) 60ยฐ (d) 75ยฐ 28. Circular measure of angle between the hands of a watch at ๐Ÿ’โ€ฒ ๐‘ถ ๐’„๐’๐’๐’„๐’Œ is (a) 45ยฐ (b) โœ” 120ยฐ (c) 3๐œ‹ 2 (d) 270ยฐ 29. If ๐’ = ๐Ÿ. ๐Ÿ“ ๐’„๐’Ž & ๐’“ = ๐Ÿ. ๐Ÿ“ ๐’„๐’Ž then ๐œฝ is equal to: (a) โœ” 3 5 (b) 5 3 (c) 3.75 (d) ๐‘๐‘œ๐‘›๐‘’ 30. If ๐œฝ = ๐Ÿ’๐Ÿ“ยฐ , ๐’“ = ๐Ÿ๐Ÿ–๐’Ž๐’Ž , then ๐’ = (a) โœ” 9 2 ๐œ‹ (b) 2 9 ๐œ‹ (c) 812mm (d) 810mm 31. Area of sector of circle of radius ๐’“ is: (a) โœ” 1 2 ๐‘Ÿ2 ๐œƒ (b) 1 2 ๐‘Ÿ๐œƒ2 (c) 1 2 (๐‘Ÿ๐œƒ)2 (d) 1 2๐‘Ÿ2๐œƒ 32. Angles with same initial and terminal sides are called: (a) Acute angles (b) Allied Angles (c) โœ”Coterminal angles(d) Quadrentel angles 33. If angle ๐œฝ is in degree, then the angle coterminal with ๐œฝ is: (a) ๐œƒ + 180ยฐ๐‘˜, ๐‘˜ โˆˆ ๐‘ (b) โœ” ๐œƒ + 360ยฐ๐‘˜, ๐‘˜ โˆˆ ๐‘ (c) ๐œƒ + 90ยฐ๐‘˜, ๐‘˜ โˆˆ ๐‘ (d) ๐œƒ + 60ยฐ๐‘˜, ๐‘˜ โˆˆ ๐‘ 34. If angle ๐œฝ is in degree, then the angle coterminal with ๐œฝ is: (a) โœ”๐œƒ + 2๐œ‹๐‘˜, ๐‘˜ โˆˆ ๐‘ (b) ๐œƒ + ๐œ‹๐‘˜, ๐‘˜ โˆˆ ๐‘ (c) ๐œƒ + ๐œ‹ 2 ๐‘˜, ๐‘˜ โˆˆ ๐‘ (d) ๐œƒ + ๐œ‹ 3 ๐‘˜, ๐‘˜ โˆˆ ๐‘ 35. An angle is in standard position , if its vertex is (a) โœ”At origin (b) at ๐‘ฅ โˆ’ ๐‘Ž๐‘ฅ๐‘–๐‘  (c) ๐‘Ž๐‘ก๐‘ฆ โˆ’ ๐‘Ž๐‘ฅ๐‘–๐‘  (d) in 1st Quad Only
  • 15. 15 | P a g e 36. If initial and the terminal side of an angle falls on ๐’™ โˆ’ ๐’‚๐’™๐’Š๐’” ๐’๐’“ ๐’š โˆ’ ๐’‚๐’™๐’Š๐’” then it is called: (a) Coterminal angle (b) โœ” Quadrantal angl (c) Allied angle (d) None of these 37. 0ยฐ, 90ยฐ, 180ยฐ, 270ยฐ and 360ยฐ are called (a) Coterminal angle (b) โœ” Quadrantal angl (c) Allied angle (d) None of these 38. ๐’”๐’Š๐’๐Ÿ ๐œฝ + ๐’„๐’๐’”๐Ÿ ๐œฝ is equal to: (a) 0 (b) -1 (c) 2 (d) โœ” 1 39. ๐Ÿ + ๐’•๐’‚๐’๐Ÿ ๐œฝ is equal to: (a) csc2 ๐œƒ (b) sin2 ๐œƒ (c) โœ”sec2 ๐œƒ (d) tan2 ๐œƒ 40. ๐œ๐ฌ๐œ๐Ÿ ๐œฝ โˆ’ ๐œ๐จ๐ญ๐Ÿ ๐œฝ is equal to: (a) 0 (b) โœ” 1 (c) -1 (d) 2 41. If ๐’”๐’Š๐’๐œฝ < 0 and ๐’„๐’๐’”๐œฝ > 0 then the terminal arm of angle lies in โ€ฆโ€ฆโ€ฆโ€ฆ.. Quad. (a) I (b) II (c) III (d) โœ” IV 42. If ๐’„๐’๐’•๐œฝ > 0 and ๐’„๐’๐’”๐’†๐’„๐œฝ > 0 then the terminal arm of angle lies in โ€ฆโ€ฆโ€ฆโ€ฆ.. Quad. (a) โœ”I (b) II (c) III (d) IV 43. If ๐’•๐’‚๐’๐œฝ < 0 and ๐’„๐’๐’”๐’†๐’„๐œฝ > 0 then the terminal arm of angle lies in โ€ฆโ€ฆโ€ฆโ€ฆ.. Quad. (a) I (b) โœ” II (c) III (d) IV 44. If ๐’”๐’†๐’„๐œฝ < 0 and ๐’”๐’Š๐’๐œฝ < 0 then the terminal arm of angle lies in โ€ฆโ€ฆโ€ฆโ€ฆ.. Quad. (a) I (b) II (c) โœ” III (d) IV 45. In right angle triangle, the measure of the side opposite to ๐Ÿ‘๐ŸŽยฐ is: (a) โœ”Half of Hypotenuse (b) Half of Base (c) Double of base (d) None of these 46. The point (๐ŸŽ, ๐Ÿ) lies on the terminal side of angle: (a) 0ยฐ (b) โœ” 90ยฐ (c) 180ยฐ (d) 270ยฐ 47. The point (โˆ’๐Ÿ, ๐ŸŽ) lies on the terminal side of angle: (a) 0ยฐ (b) 90ยฐ (c) โœ” 180ยฐ (d) 270ยฐ 48. The point (๐ŸŽ, โˆ’๐Ÿ) lies on the terminal side of angle: (a) 0ยฐ (b) 90ยฐ (c) 180ยฐ (d) โœ” 270ยฐ 49. ๐Ÿ๐‘บ๐’Š๐’๐Ÿ’๐Ÿ“ยฐ + ๐Ÿ ๐Ÿ ๐‘ช๐’๐’”๐’†๐’„๐Ÿ’๐Ÿ“ยฐ = (a) โˆš 2 3 (b) โœ” 3 โˆš2 (c) โˆ’1 (d) 1 50. Domain of ๐’”๐’Š๐’๐œฝ is: (a) โœ”๐‘… (b) ๐œƒ โˆˆ ๐‘… ๐‘๐‘ข๐‘ก ๐œƒ โ‰  ๐‘›๐œ‹, ๐‘› โˆˆ ๐‘ (c) ๐œƒ โˆˆ ๐‘… ๐‘๐‘ข๐‘ก ๐œƒ โ‰  (2๐‘›+1)๐œ‹ 2 , ๐‘› โˆˆ ๐‘ (d) None of these 51. Domain of ๐’„๐’๐’”๐œฝ is: (a) โœ”๐‘… (b) ๐œƒ โˆˆ ๐‘… ๐‘๐‘ข๐‘ก ๐œƒ โ‰  ๐‘›๐œ‹, ๐‘› โˆˆ ๐‘ (c) ๐œƒ โˆˆ ๐‘… ๐‘๐‘ข๐‘ก ๐œƒ โ‰  (2๐‘›+1)๐œ‹ 2 , ๐‘› โˆˆ ๐‘ (d) None of these 52. Domain of ๐’•๐’‚๐’๐œฝ is: (a) ๐‘… (b) ๐œƒ โˆˆ ๐‘… ๐‘๐‘ข๐‘ก ๐œƒ โ‰  ๐‘›๐œ‹, ๐‘› โˆˆ ๐‘ (c) โœ” ๐œƒ โˆˆ ๐‘… ๐‘๐‘ข๐‘ก ๐œƒ โ‰  (2๐‘›+1)๐œ‹ 2 , ๐‘› โˆˆ ๐‘ (d) None of these 53. Domain of ๐’”๐’†๐’„๐œฝ is: (a) ๐‘… (b) ๐œƒ โˆˆ ๐‘… ๐‘๐‘ข๐‘ก ๐œƒ โ‰  ๐‘›๐œ‹, ๐‘› โˆˆ ๐‘ (c) โœ” ๐œƒ โˆˆ ๐‘… ๐‘๐‘ข๐‘ก ๐œƒ โ‰  (2๐‘›+1)๐œ‹ 2 , ๐‘› โˆˆ ๐‘ (d) None of these 54. ๐’„๐’๐’”๐’†๐’„๐œฝ๐’”๐’†๐’„๐œฝ๐’”๐’Š๐’๐œฝ๐’„๐’๐’”๐œฝ = (a) โœ”1 (b) 0 (c) ๐‘ ๐‘–๐‘›๐œƒ (d) ๐‘๐‘œ๐‘ ๐œƒ 55. (๐’”๐’†๐’„๐œฝ + ๐’•๐’‚๐’๐œฝ)(๐’”๐’†๐’„๐œฝ โˆ’ ๐’•๐’‚๐’๐œฝ) = (a) โœ”1 (b) 0 ๐‘ ๐‘’๐‘๐œƒ (d) ๐‘ก๐‘Ž๐‘›๐œƒ 56. ๐Ÿโˆ’๐’”๐’Š๐’๐œฝ ๐’„๐’๐’”๐œฝ = (a) ๐‘๐‘œ๐‘  1โˆ’๐‘ ๐‘–๐‘›๐œƒ (b) โœ” ๐‘๐‘œ๐‘ ๐œƒ 1+๐‘ ๐‘–๐‘›๐œƒ (c) ๐‘ ๐‘–๐‘›๐œƒ 1โˆ’๐‘๐‘œ๐‘ ๐œƒ (d) ๐‘ ๐‘–๐‘›๐œƒ 1+๐‘๐‘œ๐‘ ๐œƒ UNIT # 10 Trigonometric Identities Each question has four possible answer. Tick the correct answer. 1. Distance between the points ๐‘จ(๐Ÿ‘, ๐Ÿ–) & ๐ต(5,6) is: (a) โœ”2โˆš2 (b) 3 (c) 4 (d) โˆš2 2. Fundamental law of trigonometry is , ๐’„๐’๐’”(๐œถ โˆ’ ๐œท) (a) โœ”๐‘๐‘œ๐‘ ๐›ผ๐‘๐‘œ๐‘ ๐›ฝ + ๐‘ ๐‘–๐‘›๐›ผ๐‘ ๐‘–๐‘›๐›ฝ (b) ๐‘๐‘œ๐‘ ๐›ผ๐‘๐‘œ๐‘ ๐›ฝ โˆ’ ๐‘ ๐‘–๐‘›๐›ผ๐‘ ๐‘–๐‘›๐›ฝ (c) ๐‘ ๐‘–๐‘›๐›ผ๐‘๐‘œ๐‘ ๐›ฝ + ๐‘๐‘œ๐‘ ๐›ผ๐‘ ๐‘–๐‘›๐›ฝ (d) ๐‘ ๐‘–๐‘›๐›ผ๐‘๐‘œ๐‘ ๐›ฝ โˆ’ ๐‘๐‘œ๐‘ ๐›ผ๐‘ ๐‘–๐‘›๐›ฝ 3. ๐’„๐’๐’”(๐œถ + ๐œท) is equal to: (a) ๐‘๐‘œ๐‘ ๐›ผ๐‘๐‘œ๐‘ ๐›ฝ + ๐‘ ๐‘–๐‘›๐›ผ๐‘ ๐‘–๐‘›๐›ฝ (b) โœ” ๐‘๐‘œ๐‘ ๐›ผ๐‘๐‘œ๐‘ ๐›ฝ โˆ’ ๐‘ ๐‘–๐‘›๐›ผ๐‘ ๐‘–๐‘›๐›ฝ (c) ๐‘ ๐‘–๐‘›๐›ผ๐‘๐‘œ๐‘ ๐›ฝ + ๐‘๐‘œ๐‘ ๐›ผ๐‘ ๐‘–๐‘›๐›ฝ (d) ๐‘ ๐‘–๐‘›๐›ผ๐‘๐‘œ๐‘ ๐›ฝ โˆ’ ๐‘๐‘œ๐‘ ๐›ผ๐‘ ๐‘–๐‘›๐›ฝ
  • 16. 16 | P a g e 4. ๐’”๐’Š๐’(๐œถ + ๐œท) is equal to: (a) ๐‘๐‘œ๐‘ ๐›ผ๐‘๐‘œ๐‘ ๐›ฝ + ๐‘ ๐‘–๐‘›๐›ผ๐‘ ๐‘–๐‘›๐›ฝ (b) ๐‘๐‘œ๐‘ ๐›ผ๐‘๐‘œ๐‘ ๐›ฝ โˆ’ ๐‘ ๐‘–๐‘›๐›ผ๐‘ ๐‘–๐‘›๐›ฝ (c) โœ” ๐‘ ๐‘–๐‘›๐›ผ๐‘๐‘œ๐‘ ๐›ฝ + ๐‘๐‘œ๐‘ ๐›ผ๐‘ ๐‘–๐‘›๐›ฝ (d) ๐‘ ๐‘–๐‘›๐›ผ๐‘๐‘œ๐‘ ๐›ฝ โˆ’ ๐‘๐‘œ๐‘ ๐›ผ๐‘ ๐‘–๐‘›๐›ฝ 5. ๐’”๐’Š๐’(๐œถ โˆ’ ๐œท) is equal to: (a) ๐‘๐‘œ๐‘ ๐›ผ๐‘๐‘œ๐‘ ๐›ฝ + ๐‘ ๐‘–๐‘›๐›ผ๐‘ ๐‘–๐‘›๐›ฝ (b) ๐‘๐‘œ๐‘ ๐›ผ๐‘๐‘œ๐‘ ๐›ฝ โˆ’ ๐‘ ๐‘–๐‘›๐›ผ๐‘ ๐‘–๐‘›๐›ฝ (c) ๐‘ ๐‘–๐‘›๐›ผ๐‘๐‘œ๐‘ ๐›ฝ + ๐‘๐‘œ๐‘ ๐›ผ๐‘ ๐‘–๐‘›๐›ฝ (d) โœ” ๐‘ ๐‘–๐‘›๐›ผ๐‘๐‘œ๐‘ ๐›ฝ โˆ’ ๐‘๐‘œ๐‘ ๐›ผ๐‘ ๐‘–๐‘›๐›ฝ 6. ๐’„๐’๐’” ( ๐… ๐Ÿ โˆ’ ๐œท) = (a) ๐‘๐‘œ๐‘ ๐›ฝ (b) โ€“ ๐‘๐‘œ๐‘ ๐›ฝ (c) โœ” ๐‘ ๐‘–๐‘›๐›ฝ (d) โ€“ ๐‘ ๐‘–๐‘›๐›ฝ 7. ๐’„๐’๐’” (๐œท + ๐… ๐Ÿ ) = (a) ๐‘๐‘œ๐‘ ๐›ฝ (b) โ€“ ๐‘๐‘œ๐‘ ๐›ฝ (c) ๐‘ ๐‘–๐‘›๐›ฝ (d) โœ” โ€“ ๐‘ ๐‘–๐‘›๐›ฝ 8. ๐’”๐’Š๐’ (๐œท โˆ’ ๐… ๐Ÿ ) = (a) ๐‘๐‘œ๐‘ ๐›ฝ (b) โœ” โ€“ ๐‘๐‘œ๐‘ ๐›ฝ (c) ๐‘ ๐‘–๐‘›๐›ฝ (d) โœ” โ€“ ๐‘ ๐‘–๐‘›๐›ฝ 9. ๐’„๐’๐’”(๐Ÿ๐… โˆ’ ๐œฝ) = (a) โœ”๐‘๐‘œ๐‘ ๐œƒ (b) โ€“ ๐‘๐‘œ๐‘ ๐œƒ (c) ๐‘ ๐‘–๐‘›๐œƒ (d) โ€“ ๐‘ ๐‘–๐‘›๐œƒ 10. ๐’”๐’Š๐’(๐Ÿ๐… โˆ’ ๐œฝ) = (a) ๐‘๐‘œ๐‘ ๐œƒ (b) โ€“ ๐‘๐‘œ๐‘ ๐œƒ (c) ๐‘ ๐‘–๐‘›๐œƒ (d) โœ” โ€“ ๐‘ ๐‘–๐‘›๐œƒ 11. ๐’•๐’‚๐’(๐œถ + ๐œท) = (a) ๐‘ก๐‘Ž๐‘›๐›ผโˆ’๐‘ก๐‘Ž๐‘›๐›ฝ 1+๐‘ก๐‘Ž๐‘›๐›ผ๐‘ก๐‘Ž๐‘›๐›ฝ (b) โœ” ๐‘ก๐‘Ž๐‘›๐›ผ+๐‘ก๐‘Ž๐‘›๐›ฝ 1โˆ’๐‘ก๐‘Ž๐‘›๐›ผ๐‘ก๐‘Ž๐‘›๐›ฝ (c) ๐‘ก๐‘Ž๐‘›๐›ผโˆ’๐‘ก๐‘Ž๐‘›๐›ฝ 1โˆ’๐‘ก๐‘Ž๐‘›๐›ผ๐‘ก๐‘Ž๐‘›๐›ฝ (d) ๐‘ก๐‘Ž๐‘›๐›ผ+๐‘ก๐‘Ž๐‘›๐›ฝ 1โˆ’๐‘ก๐‘Ž๐‘›๐›ผ๐‘ก๐‘Ž๐‘›๐›ฝ 12. ๐’•๐’‚๐’(๐œถ โˆ’ ๐œท) = (a) โœ” ๐‘ก๐‘Ž๐‘›๐›ผโˆ’๐‘ก๐‘Ž๐‘›๐›ฝ 1+๐‘ก๐‘Ž๐‘›๐›ผ๐‘ก๐‘Ž๐‘›๐›ฝ (b) ๐‘ก๐‘Ž๐‘›๐›ผ+๐‘ก๐‘Ž๐‘›๐›ฝ 1โˆ’๐‘ก๐‘Ž๐‘›๐›ผ๐‘ก๐‘Ž๐‘›๐›ฝ (c) ๐‘ก๐‘Ž๐‘›๐›ผโˆ’๐‘ก๐‘Ž๐‘›๐›ฝ 1โˆ’๐‘ก๐‘Ž๐‘›๐›ผ๐‘ก๐‘Ž๐‘›๐›ฝ (d) ๐‘ก๐‘Ž๐‘›๐›ผ+๐‘ก๐‘Ž๐‘›๐›ฝ 1โˆ’๐‘ก๐‘Ž๐‘›๐›ผ๐‘ก๐‘Ž๐‘›๐›ฝ 13. Angles associated with basic angles of measure ๐œฝ to a right angle or its multiple are called: (a) Coterminal angle (b) angle in standard position (c) โœ” Allied angle (d) obtuse angle 14. ๐’•๐’‚๐’ ( ๐… ๐Ÿ โˆ’ ๐œฝ) = (a) โœ”๐‘๐‘œ๐‘ก๐œƒ (b) ๐‘ก๐‘Ž๐‘›๐œƒ (c) โ€“ ๐‘๐‘œ๐‘ก๐œƒ (d) โ€“ ๐‘ก๐‘Ž๐‘›๐œƒ 15. ๐’•๐’‚๐’ ( ๐… ๐Ÿ + ๐œฝ) = (a) ๐‘๐‘œ๐‘ก๐œƒ (b) ๐‘ก๐‘Ž๐‘›๐œƒ (c) โœ”โ€“ ๐‘๐‘œ๐‘ก๐œƒ (d) โ€“ ๐‘ก๐‘Ž๐‘›๐œƒ 16. ๐’”๐’Š๐’ ( ๐Ÿ‘๐… ๐Ÿ + ๐œฝ) = (a) ๐‘ ๐‘–๐‘›๐œƒ (b) ๐‘๐‘œ๐‘ ๐œƒ (c) โˆ’๐‘ ๐‘–๐‘›๐œƒ (d) โœ” โ€“ ๐‘๐‘œ๐‘ ๐œƒ 17. ๐’„๐’๐’” ๐Ÿ‘๐Ÿ๐Ÿ“ยฐ is equal to: (a) 1 (b) 0 (c) โœ” 1 โˆš2 (d) โˆš3 2 18. ๐’•๐’‚๐’(โˆ’๐Ÿ๐Ÿ‘๐Ÿ“ยฐ) is equal to: (a) โœ” 1 (b) 0 (c) 1 โˆš3 (d) -1 19. ๐’”๐’†๐’„(โˆ’๐Ÿ‘๐ŸŽ๐ŸŽยฐ) = (a) 1 (b) โœ” 0 (c) 2 (d) -1 20. ๐’”๐’Š๐’(๐Ÿ๐Ÿ–๐ŸŽยฐ + ๐œถ)๐’”๐’Š๐’(๐Ÿ—๐ŸŽยฐ โˆ’ ๐œถ) = (a) โœ” ๐‘ ๐‘–๐‘›๐›ผ๐‘๐‘œ๐‘ ๐›ผ (b) โ€“ ๐‘ ๐‘–๐‘›๐›ผ๐‘๐‘œ๐‘ ๐›ผ (c) ๐‘๐‘œ๐‘ ๐›พ (d) โ€“ ๐‘๐‘œ๐‘ ๐›พ 21. If ๐œถ, ๐œท and ๐œธ are the angles of a triangle ABC then ๐’”๐’Š๐’(๐œถ + ๐œท) = (a) โœ” ๐‘ ๐‘–๐‘›๐›พ (b) โ€“ ๐‘ ๐‘–๐‘›๐›พ (c) ๐‘๐‘œ๐‘ ๐›พ (d) โ€“ ๐‘๐‘œ๐‘ ๐›พ 22. If ๐œถ, ๐œท and ๐œธ are the angles of a triangle ABC then ๐’„๐’๐’” ( ๐œถ+๐œท ๐Ÿ ) = (a) โœ” sin ๐›พ 2 (b) โ€“ sin ๐›พ 2 (c) cos ๐›พ 2 (d) โ€“ cos ๐›พ 2 23. If ๐œถ, ๐œท and ๐œธ are the angles of a triangle ABC then ๐’„๐’๐’”(๐œถ + ๐œท) = (a) ๐‘ ๐‘–๐‘›๐›พ (b) โ€“ ๐‘ ๐‘–๐‘›๐›พ (c) ๐‘๐‘œ๐‘ ๐›พ (d) โœ” โ€“ ๐‘๐‘œ๐‘ ๐›พ 24. ๐’„๐’๐’”๐Ÿ๐Ÿยฐ+๐’”๐’Š๐’๐Ÿ๐Ÿยฐ ๐’„๐’๐’”๐Ÿ๐Ÿยฐโˆ’๐’”๐’Š๐’๐Ÿ๐Ÿยฐ = (a) โœ” ๐‘ก๐‘Ž๐‘›56ยฐ (b) ๐‘ก๐‘Ž๐‘›34ยฐ (c) ๐‘๐‘œ๐‘ก56ยฐ (d) ๐‘๐‘œ๐‘ก34ยฐ 25. ๐’”๐’Š๐’๐Ÿ๐œถ is equal to: (a) cos2 ๐›ผ โˆ’ sin2 ๐›ผ (b) 1 + ๐‘๐‘œ๐‘ 2๐›ผ (c) โœ” 2๐‘ ๐‘–๐‘›๐›ผ๐‘๐‘œ๐‘ ๐›ผ (d) 2๐‘ ๐‘–๐‘›2๐›ผ๐‘๐‘œ๐‘ 2๐›ผ 26. ๐’„๐’๐’”๐Ÿ๐œถ = (a) cos2 ๐›ผ โˆ’ sin2 ๐›ผ (b) 1 โˆ’ 2 sin2 ๐›ผ (c) 2 cos2 ๐›ผ โˆ’ 1 (d) โœ” All of these 27. ๐’•๐’‚๐’๐Ÿ๐œถ = (a) 2๐‘ก๐‘Ž๐‘›๐›ผ 1+๐‘ก๐‘Ž๐‘›2 ๐›ผ (b) โœ” 2๐‘ก๐‘Ž๐‘›๐›ผ 1โˆ’tan2 ๐›ผ (c) 2 tan2 ๐›ผ 1โˆ’tan2 ๐›ผ (d) tan2 ๐›ผ 1โˆ’tan2 ๐›ผ 28. ๐’”๐’Š๐’๐œถ + ๐’”๐’Š๐’๐œท is equal to: (a) โœ” 2 sin ( ๐›ผ+๐›ฝ 2 ) cos ( ๐›ผโˆ’๐›ฝ 2 ) (b) 2 cos ( ๐›ผ+๐›ฝ 2 ) sin( ๐›ผโˆ’๐›ฝ 2 ) (c) โˆ’2 sin ( ๐›ผ+๐›ฝ 2 ) sin( ๐›ผโˆ’๐›ฝ 2 ) (d) 2 cos( ๐›ผ+๐›ฝ 2 ) cos ( ๐›ผโˆ’๐›ฝ 2 )
  • 17. 17 | P a g e 29. ๐’”๐’Š๐’๐œถ โˆ’ ๐’”๐’Š๐’๐œท is equal to: (b) 2 sin( ๐›ผ+๐›ฝ 2 ) cos ( ๐›ผโˆ’๐›ฝ 2 ) (b) โœ” 2 cos ( ๐›ผ+๐›ฝ 2 ) sin ( ๐›ผโˆ’๐›ฝ 2 ) (c) โˆ’2 sin ( ๐›ผ+๐›ฝ 2 ) sin( ๐›ผโˆ’๐›ฝ 2 ) (d) 2 cos( ๐›ผ+๐›ฝ 2 ) cos ( ๐›ผโˆ’๐›ฝ 2 ) 30. ๐’„๐’๐’”๐œถ + ๐’„๐’๐’”๐œท is equal to: (c) 2 sin( ๐›ผ+๐›ฝ 2 ) cos ( ๐›ผโˆ’๐›ฝ 2 ) (b) 2 cos ( ๐›ผ+๐›ฝ 2 ) sin( ๐›ผโˆ’๐›ฝ 2 ) (c) -2 sin( ๐›ผ+๐›ฝ 2 ) sin ( ๐›ผโˆ’๐›ฝ 2 ) (d) โœ” 2 cos ( ๐›ผ+๐›ฝ 2 )cos ( ๐›ผโˆ’๐›ฝ 2 ) 31. ๐’„๐’๐’”๐œถ โˆ’ ๐’„๐’๐’”๐œท is equal to: (d) 2 sin( ๐›ผ+๐›ฝ 2 ) cos ( ๐›ผโˆ’๐›ฝ 2 ) (b) 2 cos ( ๐›ผ+๐›ฝ 2 ) sin( ๐›ผโˆ’๐›ฝ 2 ) (c) โœ” โˆ’ 2 sin ( ๐›ผ+๐›ฝ 2 ) sin ( ๐›ผโˆ’๐›ฝ 2 ) (d) 2 cos( ๐›ผ+๐›ฝ 2 ) cos ( ๐›ผโˆ’๐›ฝ 2 ) 32. Which is the allied angle (a) โœ” 90ยฐ + ๐œƒ (b) 60ยฐ + ๐œƒ (c) 45ยฐ + ๐œƒ (d) 30ยฐ + ๐œƒ 33. ๐Ÿ๐’”๐’Š๐’๐Ÿ•๐œฝ๐’„๐’๐’”๐Ÿ‘๐œฝ = (a) โœ” ๐‘ ๐‘–๐‘›10๐œƒ + ๐‘ ๐‘–๐‘›4๐œƒ (b) ๐‘ ๐‘–๐‘›5๐œƒ โˆ’ ๐‘ ๐‘–๐‘›2๐œƒ (c) ๐‘๐‘œ๐‘ 10๐œƒ + ๐‘๐‘œ๐‘ 4๐œƒ (d) ๐‘๐‘œ๐‘ 5๐œƒ โˆ’ ๐‘๐‘œ๐‘ 2๐œƒ 34. ๐Ÿ๐’„๐’๐’”๐Ÿ“๐œฝ๐’”๐’Š๐’๐Ÿ‘๐œฝ = (b) โœ” ๐‘ ๐‘–๐‘›8๐œƒ โˆ’ ๐‘ ๐‘–๐‘›2๐œƒ (b) ๐‘ ๐‘–๐‘›8๐œƒ + ๐‘ ๐‘–๐‘›2๐œƒ (c) ๐‘๐‘œ๐‘ 8๐œƒ + ๐‘๐‘œ๐‘ 2๐œƒ (d) ๐‘๐‘œ๐‘ 8๐œƒ โˆ’ ๐‘๐‘œ๐‘ 2๐œƒ UNIT # 11 Trigonometric Functions and their Graphs Each question has four possible answer. Tick the correct answer. 1. Domain of ๐’š = ๐’”๐’Š๐’๐’™ is (a) โœ”โˆ’โˆž < ๐‘ฅ < โˆž (b) โˆ’1 โ‰ค ๐‘ฅ โ‰ค 1 (c) โˆ’โˆž < ๐‘ฅ < โˆž , ๐‘ฅ โ‰  ๐‘›๐œ‹ , ๐‘› โˆˆ ๐‘ (d) ๐‘ฅ โ‰ฅ 1, ๐‘ฅ โ‰ค โˆ’1 2. Domain of ๐’š = ๐’„๐’๐’”๐’™ is (a) โœ”โˆ’โˆž < ๐‘ฅ < โˆž (b) โˆ’1 โ‰ค ๐‘ฅ โ‰ค 1 (c) โˆ’โˆž < ๐‘ฅ < โˆž , ๐‘ฅ โ‰  ๐‘›๐œ‹ , ๐‘› โˆˆ ๐‘ (d) ๐‘ฅ โ‰ฅ 1, ๐‘ฅ โ‰ค โˆ’1 3. Domain of ๐’š = ๐’•๐’‚๐’๐’™ is (a) โˆ’โˆž < ๐‘ฅ < โˆž (b) โˆ’1 โ‰ค ๐‘ฅ โ‰ค 1 (c) โœ” โˆ’โˆž < ๐‘ฅ < โˆž , ๐‘ฅ โ‰  2๐‘›+1 2 ๐œ‹ , ๐‘› โˆˆ ๐‘ (d) ๐‘ฅ โ‰ฅ 1, ๐‘ฅ โ‰ค โˆ’1 4. Domain of ๐’š = ๐’”๐’†๐’„๐’™ is (a) โˆ’โˆž < ๐‘ฅ < โˆž (b) โˆ’1 โ‰ค ๐‘ฅ โ‰ค 1 (c) โœ” โˆ’โˆž < ๐‘ฅ < โˆž , ๐‘ฅ โ‰  2๐‘›+1 2 ๐œ‹ , ๐‘› โˆˆ ๐‘ (d) ๐‘ฅ โ‰ฅ 1, ๐‘ฅ โ‰ค โˆ’1 5. Domain of ๐’š = ๐’„๐’”๐’„๐’™ is (a) โˆ’โˆž < ๐‘ฅ < โˆž (b) โœ” โˆ’1 โ‰ค ๐‘ฅ โ‰ค 1 (c) โˆ’โˆž < ๐‘ฅ < โˆž , ๐‘ฅ โ‰  2๐‘›+1 2 ๐œ‹ , ๐‘› โˆˆ ๐‘ (d) ๐‘ฅ โ‰ฅ 1, ๐‘ฅ โ‰ค โˆ’1 6. Domain of ๐’š = ๐’„๐’๐’•๐’™ is (a) โˆ’โˆž < ๐‘ฅ < โˆž (b) โœ” โˆ’1 โ‰ค ๐‘ฅ โ‰ค 1 (c) โˆ’โˆž < ๐‘ฅ < โˆž , ๐‘ฅ โ‰  2๐‘›+1 2 ๐œ‹ , ๐‘› โˆˆ ๐‘ (d) ๐‘ฅ โ‰ฅ 1, ๐‘ฅ โ‰ค โˆ’1 7. Range of ๐’š = ๐’”๐’Š๐’๐’™ is (a) ๐‘… (b) โœ”โˆ’1 โ‰ค ๐‘ฆ โ‰ค 1 (c) (โˆ’โˆž, 1) โˆช (1, โˆž) (d) โˆ’1 < ๐‘ฆ < 1 8. Range of ๐’š = ๐’„๐’๐’”๐’™ is (a) ๐‘… (b) โœ” โˆ’1 โ‰ค ๐‘ฆ โ‰ค 1 (c) (โˆ’โˆž, 1) โˆช (1, โˆž) (d) โˆ’1 < ๐‘ฆ < 1 9. Range of ๐’š = ๐’•๐’‚๐’๐’™ is (a) โœ”๐‘… (b) โˆ’1 โ‰ค ๐‘ฆ โ‰ค 1 (c) ๐‘„ (d) ๐‘… โˆ’ {0} 10. Range of ๐’š = ๐’„๐’๐’•๐’™ is (a) โœ”๐‘… (b) ๐‘… โˆ’ [โˆ’1,1] (c) ๐‘… โˆ’ {0} (d) ๐‘ 11. Range of ๐’š = ๐’”๐’†๐’„๐’™ is (a) ๐‘… (b) โœ” ๐‘ฆ โ‰ฅ 1๐‘œ๐‘Ÿ ๐‘ฆ โ‰ค โˆ’1 (c) โˆ’1 โ‰ค ๐‘ฆ โ‰ค 1 (d) ๐‘… โˆ’ [โˆ’1,1] 12. Range of ๐’š = ๐’„๐’๐’”๐’†๐’„๐’™ is (a) ๐‘… (b) โœ” ๐‘ฆ โ‰ฅ 1๐‘œ๐‘Ÿ ๐‘ฆ โ‰ค โˆ’1 (c) โˆ’1 โ‰ค ๐‘ฆ โ‰ค 1 (d) ๐‘… โˆ’ [โˆ’1,1] 13. Smallest +๐’Š๐’—๐’† number which when added to the original circular measure of the angle gives the same value of the function is called: (a) Domain (b) Range (c) Co domain (d) โœ”Period 14. Period of ๐’”๐’Š๐’๐œฝ is (a) ๐œ‹ (b) โœ” 2๐œ‹ (c) โˆ’2๐œ‹ (d) ๐œ‹ 2 15. Period of ๐’„๐’๐’”๐’†๐’„๐œฝ is (a) ๐œ‹ (b) โœ”2๐œ‹ (c) โˆ’2๐œ‹ (d) ๐œ‹ 2
  • 18. 18 | P a g e 16. Period of ๐’„๐’๐’”๐œฝ is (a) ๐œ‹ (b) โœ”2๐œ‹ (c) โˆ’2๐œ‹ (d) ๐œ‹ 2 17. Period of ๐’•๐’‚๐’๐œฝ is (a) โœ”๐œ‹ (b) 2๐œ‹ (c) โˆ’2๐œ‹ (d) ๐œ‹ 2 18. Period of ๐’„๐’๐’•๐œฝ is (a) โœ”๐œ‹ (b) 2๐œ‹ (c) โˆ’2๐œ‹ (d) ๐œ‹ 2 19. Period of ๐’”๐’†๐’„๐œฝ is (a) ๐œ‹ (b) โœ”2๐œ‹ (c) โˆ’2๐œ‹ (d) ๐œ‹ 2 20. Period of ๐’•๐’‚๐’๐Ÿ’๐’™ is (a) ๐œ‹ (b) 2๐œ‹ (c) โˆ’2๐œ‹ (d) โœ” ๐œ‹ 4 21. Period of ๐’„๐’๐’•๐Ÿ‘๐’™ is (a) ๐œ‹ (b) โœ” ๐œ‹ 3 (c) โˆ’2๐œ‹ (d) ๐œ‹ 4 22. Period of ๐Ÿ‘๐’„๐’๐’” ๐’™ ๐Ÿ“ is (a) 2๐œ‹ (b) ๐œ‹ 2 (c) ๐œ‹ (d) โœ” 10๐œ‹ 23. The graph of trigonometric functions have: (a) Break segments (b) Sharp corners (c) Straight line segments (d) โœ” smooth curves 24. Curves of the trigonometric functions repeat after fixed intervals because trigonometric functions are (a) Simple (b) linear (c) quadratic (d) โœ” periodic 25. The graph of ๐’š = ๐’„๐’๐’”๐’™ lies between the horizontal line ๐’š = โˆ’๐Ÿ and (a) โœ” +1 (b) 0 (c) 2 (d) -2 UNIT # 12 Application of Trigonometry Each question has four possible answer. Tick the correct answer. 1. A โ€œTriangleโ€ has : (a) Two elements (b) 3 elements (c) 4 elements (d) โœ” 6 elments 2. When we look an object above the horizontal ray, the angle formed is called angle of: (a) โœ”Elevation (b) depression (c) incidence (d) reflects 3. When we look an object below the horizontal ray, the angle formed is called angle of: (a) Elevation (b) โœ” depression (c) incidence (d) reflects 4. A triangle which is not right is called: (a) โœ”Oblique triangle (b) Isosceles triangle (c) Scalene triangle (d) Right isosceles triangle 5. To solve an oblique triangle we use: (a) โœ”Law of Sine (b) Law of Cosine (c) Law of Tangents (d) All of these 6. In any triangle ๐‘จ๐‘ฉ๐‘ช, ๐’ƒ๐Ÿ+๐’„๐Ÿโˆ’๐’‚๐Ÿ ๐Ÿ๐’ƒ๐’„ = (a) โœ”๐‘๐‘œ๐‘ ๐›ผ (b) ๐‘ ๐‘–๐‘›๐›ผ (c) ๐‘๐‘œ๐‘ ๐›ฝ (d) ๐‘๐‘œ๐‘ ๐›พ 7. Which can be reduced to Pythagoras theorem, (a) Law of sine (b) โœ” law of cosine (c) law of tangents (d) Half angle formulas 8. In any triangle ๐‘จ๐‘ฉ๐‘ช, if ๐œท = ๐Ÿ—๐ŸŽยฐ , then ๐’ƒ๐Ÿ = ๐’‚๐Ÿ + ๐’„๐Ÿ โˆ’ ๐Ÿ๐’‚๐’„๐’„๐’๐’”๐œท becomes: (a) Law of sine (b) Law of tangents (c) โœ”Law of cosine (d) None of these 9. In any triangle ๐‘จ๐‘ฉ๐‘ช, law of tangent is : (a) ๐‘Žโˆ’๐‘ ๐‘Ž+๐‘ = tan(๐›ผโˆ’๐›ฝ) tan(๐›ผ+๐›ฝ) (b) ๐‘Ž+๐‘ ๐‘Žโˆ’๐‘ = tan(๐›ผ+๐›ฝ) tan(๐›ผโˆ’๐›ฝ) (c) โœ” ๐‘Žโˆ’๐‘ ๐‘Ž+๐‘ = tan ๐›ผโˆ’๐›ฝ 2 tan ๐›ผ+๐›ฝ 2 (d) ๐‘Žโˆ’๐‘ ๐‘Ž+๐‘ = tan ๐›ผ+๐›ฝ 2 tan ๐›ผโˆ’๐›ฝ 2 10. In any triangle ๐‘จ๐‘ฉ๐‘ช, โˆš (๐‘บโˆ’๐’‚)(๐’”โˆ’๐’ƒ) ๐’‚๐’ƒ = (a) sin ๐›ผ 2 (b) sin ๐›ฝ 2 (c) โœ” sin ๐›พ 2 (d) cos ๐›ผ 2 11. In any triangle ๐‘จ๐‘ฉ๐‘ช, โˆš (๐‘บโˆ’๐’ƒ)(๐’”โˆ’๐’„) ๐’ƒ๐’„ = (a) โœ” sin ๐›ผ 2 (b) sin ๐›ฝ 2 (c) sin ๐›พ 2 (d) cos ๐›ผ 2 12. In any triangle ๐‘จ๐‘ฉ๐‘ช, โˆš (๐‘บโˆ’๐’‚)(๐’”โˆ’๐’„) ๐’‚๐’„ = (a) sin ๐›ผ 2 (b) โœ” sin ๐›ฝ 2 (c) sin ๐›พ 2 (d) cos ๐›ผ 2 13. In any triangle ๐‘จ๐‘ฉ๐‘ช, ๐’„๐’๐’” ๐œถ ๐Ÿ = (a) โˆš ๐‘ (๐‘ โˆ’๐‘Ž) ๐‘Ž๐‘ (b) โˆš ๐‘ (๐‘ โˆ’๐‘) ๐‘Ž๐‘ (c) โœ” โˆš ๐‘ (๐‘ โˆ’๐‘Ž) ๐‘๐‘ (d) โˆš ๐‘ (๐‘ โˆ’๐‘) ๐‘Ž๐‘
  • 19. 19 | P a g e 14. In any triangle ๐‘จ๐‘ฉ๐‘ช, ๐’„๐’๐’” ๐œท ๐Ÿ = (a) โˆš ๐‘ (๐‘ โˆ’๐‘Ž) ๐‘Ž๐‘ (b) โœ”โˆš ๐‘ (๐‘ โˆ’๐‘) ๐‘Ž๐‘ (c) โˆš ๐‘ (๐‘ โˆ’๐‘Ž) ๐‘๐‘ (d) โˆš ๐‘ (๐‘ โˆ’๐‘) ๐‘Ž๐‘ 15. In any triangle ๐‘จ๐‘ฉ๐‘ช, ๐’„๐’๐’” ๐œธ ๐Ÿ = (a) โˆš ๐‘ (๐‘ โˆ’๐‘Ž) ๐‘Ž๐‘ (b) โˆš ๐‘ (๐‘ โˆ’๐‘) ๐‘Ž๐‘ (c) โˆš ๐‘ (๐‘ โˆ’๐‘Ž) ๐‘๐‘ (d) โœ” โˆš ๐‘ (๐‘ โˆ’๐‘) ๐‘Ž๐‘ 16. In any triangle ๐‘จ๐‘ฉ๐‘ช, with usual notations , ๐’” is equal to (a) ๐‘Ž + ๐‘ + ๐‘ (b) ๐‘Ž+๐‘+๐‘ 3 (c) โœ” ๐‘Ž+๐‘+๐‘ 2 (d) ๐‘Ž๐‘๐‘ 2 17. In any triangle ๐‘จ๐‘ฉ๐‘ช, โˆš ๐’”(๐’”โˆ’๐’„) (๐’”โˆ’๐’‚)(๐’”โˆ’๐’ƒ) = (a) sin ๐›พ 2 (b) cos ๐›พ 2 (c) tan ๐›พ 2 (d) โœ” cot ๐›พ 2 18. In any triangle ๐‘จ๐‘ฉ๐‘ช, โˆš (๐’”โˆ’๐’‚)(๐’”โˆ’๐’ƒ) ๐’”(๐’”โˆ’๐’„) = (a) sin ๐›พ 2 (b) cos ๐›พ 2 (c) โœ” tan ๐›พ 2 (d) cot ๐›พ 2 19. To solve an oblique triangles when measure of three sides are given , we can use: (a) โœ”Heroโ€™s formula (b) Law of cosine (c) Law of sine (d) Law of tangents 20. The smallest angle of โˆ†๐‘จ๐‘ฉ๐‘ช, when ๐’‚ = ๐Ÿ‘๐Ÿ•. ๐Ÿ‘๐Ÿ’ , ๐’ƒ = ๐Ÿ‘. ๐Ÿ๐Ÿ’ , ๐’„ = ๐Ÿ‘๐Ÿ“. ๐ŸŽ๐Ÿ” is (a) ๐›ผ (b) โœ”๐›ฝ (c) ๐›พ (d) cannot be determined 21. In any triangle ๐‘จ๐‘ฉ๐‘ช Area if triangle is : (a) ๐‘๐‘ sin๐›ผ (b) 1 2 ๐‘๐‘Ž ๐‘ ๐‘–๐‘›๐›ผ (c) 1 2 ๐‘Ž๐‘ ๐‘ ๐‘–๐‘›๐›ฝ (d) โœ” 1 2 ๐‘Ž๐‘๐‘ ๐‘–๐‘›๐›พ 22. The circle passing through the thee vertices of a triangle is called: (a) โœ”Circum circle (b) in-circle (c) ex-centre (d) escribed circle 23. The point of intersection of the right bisectors of the sides of the triangle is : (a) โœ”Circum centre (b) In-centre (c) Escribed center (d) Diameter 24. In any triangle ๐‘จ๐‘ฉ๐‘ช, with usual notations, ๐’‚ ๐Ÿ๐’”๐’Š๐’๐œถ = (a) ๐‘Ÿ (b) ๐‘Ÿ1 (c) โœ” ๐‘… (d) โˆ† 25. In any triangle ๐‘จ๐‘ฉ๐‘ช, with usual notations, ๐’‚ ๐’”๐’Š๐’๐œท = (a) 2๐‘Ÿ (b)2 ๐‘Ÿ1 (c) โœ”2๐‘… (d) 2โˆ† 26. In any triangle ๐‘จ๐‘ฉ๐‘ช, with usual notations, ๐’”๐’Š๐’ ๐œธ = (a) ๐‘… (b) โœ” ๐‘ 2๐‘… (c) 2๐‘… ๐‘ (d) ๐‘… 2 27. In any triangle ๐‘จ๐‘ฉ๐‘ช, with usual notations, ๐’‚๐’ƒ๐’„ = (a) ๐‘… (b) ๐‘…๐‘  (c) โœ”4๐‘…โˆ† (d) โˆ† ๐‘  28. In any triangle ๐‘จ๐‘ฉ๐‘ช, with usual notations, โˆ† ๐’”โˆ’๐’‚ = (a) ๐‘Ÿ (b) ๐‘… (c) โœ” ๐‘Ÿ1 (d) ๐‘Ÿ2 29. In any triangle ๐‘จ๐‘ฉ๐‘ช, with usual notations, โˆ† ๐’”โˆ’๐’ƒ = (a) ๐‘Ÿ (b) ๐‘… (c) ๐‘Ÿ1 (d) โœ” ๐‘Ÿ2 30. In any triangle ๐‘จ๐‘ฉ๐‘ช, with usual notations, โˆ† ๐’”โˆ’๐’„ = (a) โœ”๐‘Ÿ3 (b) ๐‘… (c) ๐‘Ÿ1 (d) ๐‘Ÿ2 31. In any triangle ๐‘จ๐‘ฉ๐‘ช, with usual notation , ๐’“: ๐‘น: ๐’“๐Ÿ = (a) 3:2:1 (b) 1:2:2 (c) โœ” 1:2:3 (d) 1:1:1 32. In any triangle ๐‘จ๐‘ฉ๐‘ช, with usual notation , ๐’“: ๐‘น: ๐’“๐Ÿ: ๐’“๐Ÿ: ๐’“๐Ÿ‘ = (a) 3:3:3:2:1 (b) 1:2:2:3:3 (c) โœ” 1:2:3:3:3 (d) 1:1:1:1:1 33. In a triangle ๐‘จ๐‘ฉ๐‘ช, if ๐œท = ๐Ÿ”๐ŸŽยฐ , ๐œธ = ๐Ÿ๐Ÿ“ยฐ then ๐œถ = (a) 90ยฐ (b) 180ยฐ (c) 150ยฐ (d) โœ”105ยฐ UNIT # 13 Inverse trigonometric functions Each question has four possible answer.Tick the correct answer. 1. If ๐’š = ๐‘บ๐’Š๐’๐’™, then Domain is : (a) โœ”โˆ’ ๐œ‹ 2 โ‰ค ๐‘ฅ โ‰ค ๐œ‹ 2 (b) 0 โ‰ค ๐‘ฅ โ‰ค ๐œ‹ (c) [0, ๐œ‹], ๐‘ฅ โ‰  ๐œ‹ 2 (d) [โˆ’ ๐œ‹ 2 , ๐œ‹ 2 ] , ๐‘ฅ โ‰  0 2. If ๐’š = ๐‘ช๐’๐’”๐’™, then Domain is : (a) โˆ’ ๐œ‹ 2 โ‰ค ๐‘ฅ โ‰ค ๐œ‹ 2 (b) โœ” 0 โ‰ค ๐‘ฅ โ‰ค ๐œ‹ (c) [0, ๐œ‹], ๐‘ฅ โ‰  ๐œ‹ 2 (d) [โˆ’ ๐œ‹ 2 , ๐œ‹ 2 ] , ๐‘ฅ โ‰  0 3. If ๐’š = ๐‘บ๐’†๐’„๐’™, then Domain is : (a) โˆ’ ๐œ‹ 2 โ‰ค ๐‘ฅ โ‰ค ๐œ‹ 2 (b) 0 โ‰ค ๐‘ฅ โ‰ค ๐œ‹ (c) โœ” [0, ๐œ‹], ๐‘ฅ โ‰  ๐œ‹ 2 (d) [โˆ’ ๐œ‹ 2 , ๐œ‹ 2 ] , ๐‘ฅ โ‰  0
  • 20. 20 | P a g e 4. If ๐’š = ๐‘ช๐’๐’”๐’†๐’„๐’™, then Domain is : (a) โˆ’ ๐œ‹ 2 โ‰ค ๐‘ฅ โ‰ค ๐œ‹ 2 (b) 0 โ‰ค ๐‘ฅ โ‰ค ๐œ‹ (c) [0, ๐œ‹], ๐‘ฅ โ‰  ๐œ‹ 2 (d) โœ” [โˆ’ ๐œ‹ 2 , ๐œ‹ 2 ] , ๐‘ฅ โ‰  0 5. If ๐’š = ๐‘ป๐’‚๐’๐’™, then Domain is : (a) โˆ’ ๐œ‹ 2 โ‰ค ๐‘ฅ โ‰ค ๐œ‹ 2 (b) โœ”0 โ‰ค ๐‘ฅ โ‰ค ๐œ‹ (c) [0, ๐œ‹], ๐‘ฅ โ‰  ๐œ‹ 2 (d) [โˆ’ ๐œ‹ 2 , ๐œ‹ 2 ] , ๐‘ฅ โ‰  0 6. If ๐’š = ๐‘ช๐’๐’•๐’™, then Domain is : (a) โˆ’ ๐œ‹ 2 โ‰ค ๐‘ฅ โ‰ค ๐œ‹ 2 (b) 0 โ‰ค ๐‘ฅ โ‰ค ๐œ‹ (c) โœ” [0, ๐œ‹], ๐‘ฅ โ‰  ๐œ‹ 2 (d) [โˆ’ ๐œ‹ 2 , ๐œ‹ 2 ] , ๐‘ฅ โ‰  0 7. If ๐’š = ๐‘บ๐’Š๐’๐’™, then range is : (a) โœ”โˆ’1 โ‰ค ๐‘ฅ โ‰ค 1 (b) (โˆ’โˆž, +โˆž)๐‘œ๐‘Ÿ ๐‘… (c) ๐‘ฆ โ‰ค โˆ’1 ๐‘œ๐‘Ÿ ๐‘ฆ โ‰ฅ 1 (d) ๐‘ฆ < โˆ’1๐‘œ๐‘Ÿ ๐‘ฆ > 1 8. If ๐’š = ๐‘ช๐’๐’”๐’™, then range is : (a) โœ”โˆ’1 โ‰ค ๐‘ฅ โ‰ค 1 (b) (โˆ’โˆž, +โˆž)๐‘œ๐‘Ÿ ๐‘… (c) ๐‘ฆ โ‰ค โˆ’1 ๐‘œ๐‘Ÿ ๐‘ฆ โ‰ฅ 1 (d) ๐‘ฆ < โˆ’1๐‘œ๐‘Ÿ ๐‘ฆ > 1 9. If ๐’š = ๐‘ป๐’‚๐’๐’™, then range is : (a) โˆ’1 โ‰ค ๐‘ฅ โ‰ค 1 (b) โœ” (โˆ’โˆž, +โˆž)๐‘œ๐‘Ÿ ๐‘… (c) ๐‘ฆ โ‰ค โˆ’1 ๐‘œ๐‘Ÿ ๐‘ฆ โ‰ฅ 1 (d) ๐‘ฆ < โˆ’1๐‘œ๐‘Ÿ ๐‘ฆ > 1 10. If ๐’š = ๐‘ช๐’๐’•๐’™, then range is : (a) โˆ’1 โ‰ค ๐‘ฅ โ‰ค 1 (b) โœ” (โˆ’โˆž, +โˆž)๐‘œ๐‘Ÿ ๐‘… (c) ๐‘ฆ โ‰ค โˆ’1 ๐‘œ๐‘Ÿ ๐‘ฆ โ‰ฅ 1 (d) ๐‘ฆ < โˆ’1๐‘œ๐‘Ÿ ๐‘ฆ > 1 11. If ๐’š = ๐‘ช๐’๐’”๐’†๐’„๐’™, then range is : (a) โˆ’1 โ‰ค ๐‘ฅ โ‰ค 1 (b) (โˆ’โˆž, +โˆž)๐‘œ๐‘Ÿ ๐‘… (c) โœ”๐‘ฆ โ‰ค โˆ’1 ๐‘œ๐‘Ÿ ๐‘ฆ โ‰ฅ 1 (d) ๐‘ฆ < โˆ’1๐‘œ๐‘Ÿ ๐‘ฆ > 1 12. If ๐’š = ๐‘บ๐’†๐’„๐’™, then range is : (a) โˆ’1 โ‰ค ๐‘ฅ โ‰ค 1 (b) (โˆ’โˆž, +โˆž)๐‘œ๐‘Ÿ ๐‘… (c) โœ”๐‘ฆ โ‰ค โˆ’1 ๐‘œ๐‘Ÿ ๐‘ฆ โ‰ฅ 1 (d) ๐‘ฆ < โˆ’1๐‘œ๐‘Ÿ ๐‘ฆ > 1 13. If ๐’š = ๐‘บ๐’Š๐’โˆ’๐Ÿ ๐’™, then domain is: (a) โœ” โˆ’ 1 โ‰ค ๐‘ฅ โ‰ค 1 (b) (โˆ’โˆž, +โˆž)๐‘œ๐‘Ÿ ๐‘… (c) ๐‘ฅ โ‰ฅ โˆ’1 ๐‘œ๐‘Ÿ ๐‘ฅ โ‰ค 1 (d) ๐‘ฅ โ‰ค โˆ’1 ๐‘œ๐‘Ÿ ๐‘ฅ โ‰ฅ 1 14. If ๐’š = ๐‘ช๐’๐’”โˆ’๐Ÿ ๐’™, then domain is: (a) โœ” โˆ’ 1 โ‰ค ๐‘ฅ โ‰ค 1 (b) (โˆ’โˆž, +โˆž)๐‘œ๐‘Ÿ ๐‘… (c) ๐‘ฅ โ‰ฅ โˆ’1 ๐‘œ๐‘Ÿ ๐‘ฅ โ‰ค 1 (d) ๐‘ฅ โ‰ค โˆ’1 ๐‘œ๐‘Ÿ ๐‘ฅ โ‰ฅ 1 15. If ๐’š = ๐‘ป๐’‚๐’โˆ’๐Ÿ ๐’™, then domain is: (a) โˆ’1 โ‰ค ๐‘ฅ โ‰ค 1 (b) โœ” (โˆ’โˆž, +โˆž)๐‘œ๐‘Ÿ ๐‘… (c) ๐‘ฅ โ‰ฅ โˆ’1 ๐‘œ๐‘Ÿ ๐‘ฅ โ‰ค 1 (d) ๐‘ฅ โ‰ค โˆ’1 ๐‘œ๐‘Ÿ ๐‘ฅ โ‰ฅ 1 16. If ๐’š = ๐‘ช๐’๐’•โˆ’๐Ÿ ๐’™, then domain is: (a) โˆ’1 โ‰ค ๐‘ฅ โ‰ค 1 (b) โœ” (โˆ’โˆž, +โˆž)๐‘œ๐‘Ÿ ๐‘… (c) ๐‘ฅ โ‰ฅ โˆ’1 ๐‘œ๐‘Ÿ ๐‘ฅ โ‰ค 1 (d) ๐‘ฅ โ‰ค โˆ’1 ๐‘œ๐‘Ÿ ๐‘ฅ โ‰ฅ 1 17. If ๐’š = ๐‘บ๐’†๐’„โˆ’๐Ÿ ๐’™, then domain is: (a) โˆ’1 โ‰ค ๐‘ฅ โ‰ค 1 (b) (โˆ’โˆž, +โˆž)๐‘œ๐‘Ÿ ๐‘… (c) โœ” ๐‘ฅ โ‰ฅ โˆ’1 ๐‘œ๐‘Ÿ ๐‘ฅ โ‰ค 1 (d) ๐‘ฅ โ‰ค โˆ’1 ๐‘œ๐‘Ÿ ๐‘ฅ โ‰ฅ 1 18. If ๐’š = ๐‘ช๐’๐’”๐’†๐’„โˆ’๐Ÿ ๐’™, then domain is: (a) โˆ’1 โ‰ค ๐‘ฅ โ‰ค 1 (b) (โˆ’โˆž, +โˆž)๐‘œ๐‘Ÿ ๐‘… (c) โœ” ๐‘ฅ โ‰ฅ โˆ’1 ๐‘œ๐‘Ÿ ๐‘ฅ โ‰ค 1 (d) ๐‘ฅ โ‰ค โˆ’1 ๐‘œ๐‘Ÿ ๐‘ฅ โ‰ฅ 1 19. If ๐’š = ๐‘บ๐’Š๐’โˆ’๐Ÿ ๐’™, then range is: (a) โœ” โˆ’ ๐œ‹ 2 โ‰ค ๐‘ฅ โ‰ค ๐œ‹ 2 (b) 0 โ‰ค ๐‘ฅ โ‰ค ๐œ‹ (c) โ€“ ๐œ‹ 2 < ๐‘ฅ < ๐œ‹ 2 (d) 0 < ๐‘ฅ < ๐œ‹ 20. If ๐’š = ๐‘ช๐’๐’”โˆ’๐Ÿ ๐’™, then range is: (a) โˆ’ ๐œ‹ 2 โ‰ค ๐‘ฅ โ‰ค ๐œ‹ 2 (b) โœ” 0 โ‰ค ๐‘ฅ โ‰ค ๐œ‹ (c) โ€“ ๐œ‹ 2 < ๐‘ฅ < ๐œ‹ 2 (d) 0 < ๐‘ฅ < ๐œ‹ 21. If ๐’š = ๐‘ป๐’‚๐’โˆ’๐Ÿ ๐’™, then range is: (a) โˆ’ ๐œ‹ 2 โ‰ค ๐‘ฅ โ‰ค ๐œ‹ 2 (b) 0 โ‰ค ๐‘ฅ โ‰ค ๐œ‹ (c) โœ” โ€“ ๐œ‹ 2 < ๐‘ฅ < ๐œ‹ 2 (d) 0 < ๐‘ฅ < ๐œ‹ 22. If ๐’š = ๐‘ช๐’๐’•โˆ’๐Ÿ ๐’™, then range is: (a) โˆ’ ๐œ‹ 2 โ‰ค ๐‘ฅ โ‰ค ๐œ‹ 2 (b) 0 โ‰ค ๐‘ฅ โ‰ค ๐œ‹ (c) โ€“ ๐œ‹ 2 < ๐‘ฅ < ๐œ‹ 2 (d) โœ” 0 < ๐‘ฅ < ๐œ‹ 23. If ๐’š = ๐’”๐’†๐’„โˆ’๐Ÿ ๐’™, then range is: (a) โœ” โˆ’ ๐œ‹ 2 โ‰ค ๐‘ฅ โ‰ค ๐œ‹ 2 (b) 0 โ‰ค ๐‘ฅ โ‰ค ๐œ‹ (c) โ€“ ๐œ‹ 2 < ๐‘ฅ < ๐œ‹ 2 (d) 0 < ๐‘ฅ < ๐œ‹ 24. If ๐’š = ๐‘ช๐’๐’”๐’†๐’„โˆ’๐Ÿ ๐’™, then range is: (a) โˆ’ ๐œ‹ 2 โ‰ค ๐‘ฅ โ‰ค ๐œ‹ 2 (b) โœ” 0 โ‰ค ๐‘ฅ โ‰ค ๐œ‹ (c) โ€“ ๐œ‹ 2 < ๐‘ฅ < ๐œ‹ 2 (d) 0 < ๐‘ฅ < ๐œ‹ 25. Inverse of a function exist only if it is: (a) Trigonometric function (b) โœ”(1 โˆ’ 1) function (c) onto function (d) an into function 26. ๐‘บ๐’Š๐’โˆ’๐Ÿ ๐’™ = (a) โœ” ๐œ‹ 2 โˆ’ cosโˆ’1 ๐‘ฅ (b) ๐œ‹ 2 โˆ’ sinโˆ’1 ๐‘ฅ (c) ๐œ‹ 2 + cosโˆ’1 ๐‘ฅ (d) ๐œ‹ 2 โˆ’ ๐‘๐‘œ๐‘ ๐‘’๐‘โˆ’1 ๐‘ฅ 27. ๐‘ช๐’๐’”โˆ’๐Ÿ ๐’™ = (a) ๐œ‹ 2 โˆ’ cosโˆ’1 ๐‘ฅ (b) โœ” ๐œ‹ 2 โˆ’ sinโˆ’1 ๐‘ฅ (c) ๐œ‹ 2 + cosโˆ’1 ๐‘ฅ (d) ๐œ‹ 2 โˆ’ ๐‘๐‘œ๐‘ ๐‘’๐‘โˆ’1 ๐‘ฅ 28. ๐‘ช๐’๐’”โˆ’๐Ÿ ๐’™ = (a) โœ” ๐œ‹ 2 โˆ’ secโˆ’1 ๐‘ฅ (b) ๐œ‹ 2 โˆ’ sinโˆ’1 ๐‘ฅ (c) ๐œ‹ 2 + secโˆ’1 ๐‘ฅ (d) ๐œ‹ 2 โˆ’ ๐‘๐‘œ๐‘ ๐‘’๐‘โˆ’1 ๐‘ฅ 29. ๐‘บ๐’†๐’„โˆ’๐Ÿ ๐’™ = (a) ๐œ‹ 2 โˆ’ secโˆ’1 ๐‘ฅ (b) ๐œ‹ 2 โˆ’ sinโˆ’1 ๐‘ฅ (c) ๐œ‹ 2 + secโˆ’1 ๐‘ฅ (d) โœ” ๐œ‹ 2 โˆ’ ๐‘๐‘œ๐‘ ๐‘’๐‘โˆ’1 ๐‘ฅ 30. ๐‘ป๐’‚๐’โˆ’๐Ÿ ๐’™ = (a) ๐œ‹ 2 โˆ’ secโˆ’1 ๐‘ฅ (b) ๐œ‹ 2 โˆ’ sinโˆ’1 ๐‘ฅ (c) โœ” ๐œ‹ 2 โˆ’cotโˆ’1 ๐‘ฅ (d) ๐œ‹ 2 โˆ’ ๐‘๐‘œ๐‘ ๐‘’๐‘โˆ’1 ๐‘ฅ 31. ๐‘ช๐’๐’•โˆ’๐Ÿ ๐’™ = (a) ๐œ‹ 2 โˆ’ secโˆ’1 ๐‘ฅ (b)โœ” ๐œ‹ 2 โˆ’ tanโˆ’1 ๐‘ฅ (c) ๐œ‹ 2 + secโˆ’1 ๐‘ฅ (d) ๐œ‹ 2 โˆ’ ๐‘๐‘œ๐‘ ๐‘’๐‘โˆ’1 ๐‘ฅ
  • 21. 21 | P a g e 32. ๐‘บ๐’Š๐’ (๐‘ช๐’๐’”โˆ’๐Ÿ โˆš๐Ÿ‘ ๐Ÿ ) = (a) ๐œ‹ 6 (b) โœ” 1 2 (c) โˆ’ 1 2 (d) โˆš3 2 33. ๐‘ป๐’‚๐’โˆ’๐Ÿ (โˆš๐Ÿ‘) = (a) ๐œ‹ 6 (b) โ€“ ๐œ‹ 6 (c) โ€“ ๐œ‹ 3 (d) โœ” ๐œ‹ 3 34. ๐‘บ๐’Š๐’ (๐‘บ๐’Š๐’โˆ’๐Ÿ ๐Ÿ ๐Ÿ ) = (a) โœ” 1 2 (b) 2 3 (c) 2 (d) 1 3 35. ๐‘บ๐’Š๐’โˆ’๐Ÿ ๐‘จ + ๐‘บ๐’Š๐’โˆ’๐Ÿ ๐‘ฉ = (a) โœ”๐‘†๐‘–๐‘›โˆ’1 (๐ดโˆš1 โˆ’ ๐ต2 + ๐ตโˆš1 โˆ’ ๐ด2 ) (b) ๐‘†๐‘–๐‘›โˆ’1 (๐ดโˆš1 โˆ’ ๐ด2 โˆ’ ๐ตโˆš1 โˆ’ ๐ต2) (c) ๐‘†๐‘–๐‘›โˆ’1 (๐ตโˆš1 โˆ’ ๐ด2 + ๐ดโˆš1 โˆ’ ๐ต2) (d) ๐‘†๐‘–๐‘›โˆ’1 (๐ด๐ตโˆš(1 โˆ’ ๐ด2)(1 โˆ’ ๐ต2)) 36. ๐‘บ๐’Š๐’โˆ’๐Ÿ ๐‘จ โˆ’ ๐‘บ๐’Š๐’โˆ’๐Ÿ ๐‘ฉ = (a) ๐‘†๐‘–๐‘›โˆ’1 (๐ดโˆš1 โˆ’ ๐ต2 + ๐ตโˆš1 โˆ’ ๐ด2) (b) โœ” ๐‘†๐‘–๐‘›โˆ’1 (๐ดโˆš1 โˆ’ ๐ด2 โˆ’ ๐ตโˆš1 โˆ’ ๐ต2) (c) ๐‘†๐‘–๐‘›โˆ’1 (๐ตโˆš1 โˆ’ ๐ด2 + ๐ดโˆš1 โˆ’ ๐ต2) (d) ๐‘†๐‘–๐‘›โˆ’1 (๐ด๐ตโˆš(1 โˆ’ ๐ด2)(1 โˆ’ ๐ต2)) 37. ๐‘ช๐’๐’”โˆ’๐Ÿ ๐‘จ + ๐‘ช๐’๐’”โˆ’๐Ÿ ๐‘ฉ = (a) ๐ถ๐‘œ๐‘ โˆ’1 (๐ด๐ต โˆ’ โˆš(1 โˆ’ ๐ด2)(1 โˆ’ ๐ต2)) (b) ๐ถ๐‘œ๐‘ โˆ’1 (๐ด๐ต + โˆš(1 โˆ’ ๐ด2)(1 โˆ’ ๐ต2)) (c) ๐ถ๐‘œ๐‘ โˆ’1 (๐ด๐ต โˆ’ โˆš(1 + ๐ด2)(1 + ๐ต2)) (d) ๐ถ๐‘œ๐‘ โˆ’1 (๐ด๐ต + โˆš(1 + ๐ด2)(1 + ๐ต2)) 38. ๐‘ช๐’๐’”โˆ’๐Ÿ ๐‘จ + ๐‘ช๐’๐’”โˆ’๐Ÿ ๐‘ฉ = (a) โœ” ๐ถ๐‘œ๐‘ โˆ’1 (๐ด๐ต โˆ’ โˆš(1 โˆ’ ๐ด2)(1 โˆ’ ๐ต2)) (b) โœ” ๐ถ๐‘œ๐‘ โˆ’1 (๐ด๐ต + โˆš(1 โˆ’ ๐ด2)(1 โˆ’ ๐ต2)) (c) ๐ถ๐‘œ๐‘ โˆ’1 (๐ด๐ต โˆ’ โˆš(1 + ๐ด2)(1 + ๐ต2)) (d) ๐ถ๐‘œ๐‘ โˆ’1 (๐ด๐ต + โˆš(1 + ๐ด2)(1 + ๐ต2)) 39. ๐‘ป๐’‚๐’โˆ’๐Ÿ ๐‘จ + ๐‘ป๐’‚๐’โˆ’๐Ÿ ๐‘ฉ = (a) โœ”๐‘‡๐‘Ž๐‘›โˆ’1 ( ๐ดโˆ’๐ต 1+๐ด๐ต ) (b) ๐‘‡๐‘Ž๐‘›โˆ’1 ( ๐ด+๐ต 1+๐ด๐ต ) (c) ๐‘‡๐‘Ž๐‘›โˆ’1 ( ๐ดโˆ’๐ต 1โˆ’๐ด๐ต ) (d) ๐‘‡๐‘Ž๐‘›โˆ’1 ( ๐ด+๐ต 1+๐ด๐ต ) 40. ๐‘ป๐’‚๐’โˆ’๐Ÿ ๐‘จ + ๐‘ป๐’‚๐’โˆ’๐Ÿ ๐‘ฉ = (a) ๐‘‡๐‘Ž๐‘›โˆ’1 ( ๐ดโˆ’๐ต 1+๐ด๐ต ) (b) โœ” ๐‘‡๐‘Ž๐‘›โˆ’1 ( ๐ด+๐ต 1+๐ด๐ต ) (c) ๐‘‡๐‘Ž๐‘›โˆ’1 ( ๐ดโˆ’๐ต 1โˆ’๐ด๐ต ) (d) ๐‘‡๐‘Ž๐‘›โˆ’1 ( ๐ด+๐ต 1+๐ด๐ต ) 41. ๐‘บ๐’Š๐’โˆ’๐Ÿ(โˆ’๐’™) = (a) โœ”โ€“ ๐‘†๐‘–๐‘›โˆ’1 ๐‘ฅ (b) ๐‘†๐‘–๐‘›โˆ’1 ๐‘ฅ (c) ๐œ‹ โˆ’ ๐‘†๐‘–๐‘›โˆ’1 ๐‘ฅ (d) ๐œ‹ โˆ’ ๐‘†๐‘–๐‘›๐‘ฅ 42. ๐‘ช๐’๐’”โˆ’๐Ÿ(โˆ’๐’™) = (a) โ€“ ๐ถ๐‘œ๐‘ โˆ’1 ๐‘ฅ (b) ๐ถ๐‘œ๐‘ โˆ’1 ๐‘ฅ (c) โœ” ๐œ‹ โˆ’ ๐ถ๐‘œ๐‘ โˆ’1 ๐‘ฅ (d) ๐œ‹ โˆ’ ๐ถ๐‘œ๐‘ ๐‘ฅ 43. ๐‘ป๐’‚๐’โˆ’๐Ÿ(โˆ’๐’™) = (a) โœ”โ€“ ๐‘‡๐‘Ž๐‘›โˆ’1 ๐‘ฅ (b) ๐‘‡๐‘Ž๐‘›โˆ’1 ๐‘ฅ (c) ๐œ‹ โˆ’ ๐‘‡๐‘Ž๐‘›โˆ’1 ๐‘ฅ (d) ๐œ‹ โˆ’ ๐‘‡๐‘Ž๐‘›๐‘ฅ 44. ๐‘ช๐’๐’”๐’†๐’„โˆ’๐Ÿ(โˆ’๐’™) = (a) โœ”โ€“ ๐ถ๐‘œ๐‘ ๐‘’๐‘โˆ’1 ๐‘ฅ (b) ๐ถ๐‘œ๐‘ ๐‘’๐‘โˆ’1 ๐‘ฅ (c) ๐œ‹ โˆ’ ๐ถ๐‘œ๐‘ ๐‘’๐‘โˆ’1 ๐‘ฅ (d) ๐œ‹ โˆ’ ๐ถ๐‘œ๐‘ ๐‘’๐‘๐‘ฅ 45. ๐‘บ๐’†๐’„โˆ’๐Ÿ(โˆ’๐’™) = (a) โ€“ ๐‘†๐‘’๐‘โˆ’1 ๐‘ฅ (b) ๐‘†๐‘’๐‘โˆ’1 ๐‘ฅ (c) โœ” ๐œ‹ โˆ’ ๐‘†๐‘’๐‘โˆ’1 ๐‘ฅ (d) ๐œ‹ โˆ’ ๐‘†๐‘’๐‘๐‘ฅ 46. ๐‘ช๐’๐’•โˆ’๐Ÿ(โˆ’๐’™) = (a) โˆ’๐ถ๐‘œ๐‘กโˆ’1 ๐‘ฅ (b) ๐ถ๐‘œ๐‘กโˆ’1 ๐‘ฅ (c) โœ” ๐œ‹ โˆ’ ๐ถ๐‘œ๐‘กโˆ’1 ๐‘ฅ (d) ๐œ‹ โˆ’ ๐ถ๐‘œ๐‘ก๐‘ฅ UNIT # 14 Solution of Trigonometric Equations Each question has four possible answer. Tick the correct answer. 1. An equation containing at least one trigonometric function is called: (a) Trigonometric function (b) โœ”Trigonometric equation (c) Trigonometric value (d) None 2. If ๐’”๐’Š๐’๐’™ = ๐Ÿ ๐Ÿ , then solution in the interval [๐ŸŽ, ๐Ÿ๐…] is: (a) โœ”{ ๐œ‹ 6 , 5๐œ‹ 6 } (b) { ๐œ‹ 6 , 7๐œ‹ 6 } (c) { ๐œ‹ 3 , 4๐œ‹ 6 } (d) { ๐œ‹ 3 , 2๐œ‹ 6 } 3. If ๐’„๐’๐’”๐’™ = ๐Ÿ ๐Ÿ , then the reference angle is: (a) โœ” ๐œ‹ 3 (b) โ€“ ๐œ‹ 3 (c) ๐œ‹ 6 (d) โ€“ ๐œ‹ 6 4. If ๐’”๐’Š๐’๐’™ = ๐Ÿ ๐Ÿ , then the reference angle is: (a) ๐œ‹ 3 (b) โ€“ ๐œ‹ 3 (c) โœ” ๐œ‹ 6 (d) โ€“ ๐œ‹ 6
  • 22. 22 | P a g e 5. General solution of ๐’•๐’‚๐’๐’™ = ๐Ÿ is: (a) โœ”{ ๐œ‹ 4 + ๐‘›๐œ‹, 5๐œ‹ 4 + ๐‘›๐œ‹} (b) { ๐œ‹ 4 + 2๐‘›๐œ‹, 5๐œ‹ 4 + 2๐‘›๐œ‹} (c) { ๐œ‹ 4 + ๐‘›๐œ‹, 3๐œ‹ 4 + ๐‘›๐œ‹} (d) { ๐œ‹ 4 + 2๐‘›๐œ‹, 3๐œ‹ 4 + 2๐‘›๐œ‹} 6. If ๐’•๐’‚๐’๐Ÿ๐’™ = โˆ’๐Ÿ, then solution in the interval [๐ŸŽ, ๐…]is: (a) โœ” ๐œ‹ 8 (b) ๐œ‹ 4 (c) 3๐œ‹ 8 (d) 3๐œ‹ 4 7. If ๐’”๐’Š๐’๐’™ + ๐’„๐’๐’”๐’™ = ๐ŸŽ then value of ๐’™ โˆˆ [๐ŸŽ, ๐Ÿ๐…] (a) { ๐œ‹ 4 , 3๐œ‹ 4 } (b) { ๐œ‹ 4 , 7๐œ‹ 4 } (c) โœ” { 3๐œ‹ 4 , 7๐œ‹ 4 } (d) { ๐œ‹ 4 , โˆ’๐œ‹ 4 } 8. General solution of ๐Ÿ’๐’”๐’Š๐’๐’™ โˆ’ ๐Ÿ– = ๐ŸŽ is: (a) {๐œ‹ + 2๐‘›๐œ‹} (b) {๐œ‹ + ๐‘›๐œ‹} (c) {โˆ’๐œ‹ + ๐‘›๐œ‹} (d) โœ”not possible 9. General solution of ๐Ÿ + ๐’„๐’๐’”๐’™ = ๐ŸŽ is: (a) โœ”{๐œ‹ + 2๐‘›๐œ‹} (b) {๐œ‹ + ๐‘›๐œ‹} (c) {โˆ’๐œ‹ + ๐‘›๐œ‹} (d) not possible 10. For the general solution , we first find the solution in the interval whose length is equal to its: (a) Range (b) domain (c) co-domain (d) โœ” period 11. All trigonometric functions are โ€ฆโ€ฆโ€ฆโ€ฆโ€ฆโ€ฆ.. functions. (a) โœ”Periodic (b) continues (c) injective (d) bijective 12. General solution of every trigonometric equation consists of : (a) One solution only (b) two solutions (c) โœ” infinitely many solutions (d) no real solution 13. Solution of the equation ๐Ÿ๐’”๐’Š๐’๐’™ + โˆš๐Ÿ‘ = ๐ŸŽ in the 4th quadrant is: (a) ๐œ‹ 2 (bโœ”) โˆ’๐œ‹ 3 (c) โˆ’๐œ‹ 6 (d) 11๐œ‹ 6 14. If ๐’”๐’Š๐’๐’™ = ๐’„๐’๐’”๐’™, then general solution is: (a) { ๐œ‹ 4 + ๐‘›๐œ‹, ๐‘› โˆˆ ๐‘} (b) { ๐œ‹ 4 + 2๐‘›๐œ‹, ๐‘› โˆˆ ๐‘} (c) โœ” { ๐œ‹ 4 + ๐‘›๐œ‹, 5๐œ‹ 4 + ๐‘›๐œ‹} (d){ ๐œ‹ 4 + ๐‘›๐œ‹, 5๐œ‹ 4 + ๐‘›๐œ‹} 15. In which quadrant is the solution of the equation ๐’”๐’Š๐’๐’™ + ๐Ÿ = ๐ŸŽ (a) 1st and 2nd (b) 2nd and 3rd (c) โœ” 3rd and 4th (d) Only 1st 16. If ๐’”๐’Š๐’๐’™ = ๐ŸŽ then ๐’™ = (a) โœ”๐‘›๐œ‹ , ๐‘› โˆˆ ๐‘ (b) ๐‘›๐œ‹ 2 , ๐‘› โˆˆ ๐‘ (c) 0 (d) ๐œ‹ 2 17. If ๐’•๐’‚๐’๐’™ = ๐ŸŽ then ๐’™ = (a) โœ”๐‘›๐œ‹ , ๐‘› โˆˆ ๐‘ (b) ๐‘›๐œ‹ 2 , ๐‘› โˆˆ ๐‘ (c) 0 (d) ๐œ‹ 2 18. The solution of the ๐’„๐’๐’•๐’™ = ๐Ÿ โˆš๐Ÿ‘ in [๐ŸŽ, ๐…] is (a) ๐œ‹ 6 (b) ๐œ‹ 4 (c) 2๐œ‹ 3 (d) โœ” ๐œ‹ 3 19. One solution of ๐’”๐’†๐’„๐’™ = โˆ’๐Ÿ is : (a) ๐œ‹ 6 (b) ๐œ‹ 4 (c) โœ” 2๐œ‹ 3 (d) ๐œ‹ 3 20. ๐‘บ๐’Š๐’๐Ÿ๐’™ = โˆš๐Ÿ‘ ๐Ÿ has two values of ๐’™ in the interval: (a) [0, ๐œ‹ 2 ] (b) [0,2๐œ‹] (c) [โ€“ ๐œ‹, ๐œ‹ 2 ] (d) [โˆ’ ๐œ‹ 2 , 0] ๏ƒŸ----------THE END--------๏ƒ  WITH BEST WISHES BY:- MUHAMMAD SALMAN SHERAZI M.Sc(Math) , B.ed 03337727666/03067856232